SlideShare uma empresa Scribd logo
1 de 199
Baixar para ler offline
Sumário
Aula 1: Integrais Duplas

11

1.1

Introdução . . . . . . . . . . . . . . . . . . . . . . .

12

1.2

Integral Dupla: Domínios Retangulares . . . . . . . .

12

1.3

Integral Dupla: Domínios Não Retangulares Limitados

14

1.4

Interpretação Geométrica . . . . . . . . . . . . . . .

15

1.5

Integrais Iteradas . . . . . . . . . . . . . . . . . . .

16

1.6

Propriedades das Integrais Duplas . . . . . . . . . .

19

1.7

Alguns Exemplos . . . . . . . . . . . . . . . . . . .

20

1.8

Conclusão . . . . . . . . . . . . . . . . . . . . . . .

25

RESUMO . . . . . . . . . . . . . . . . . . . . . . . . .

25

PRÓXIMA AULA . . . . . . . . . . . . . . . . . . . .

29

ATIVIDADES . . . . . . . . . . . . . . . . . . . . . .

29

LEITURA COMPLEMENTAR . . . . . . . . . . .

30

Aula 2: Mudança de Variáveis em Integrais Duplas

33

2.1

Introdução . . . . . . . . . . . . . . . . . . . . . . .

34

2.2

Mudança de Variáveis em Integrais Duplas

. . . . .

34

2.3

Alguns Exemplos . . . . . . . . . . . . . . . . . . .

39

2.4

Conclusão . . . . . . . . . . . . . . . . . . . . . . .

43

RESUMO . . . . . . . . . . . . . . . . . . . . . . . . .

43

PRÓXIMA AULA . . . . . . . . . . . . . . . . . . . .

45
ATIVIDADES . . . . . . . . . . . . . . . . . . . . . .

45

LEITURA COMPLEMENTAR . . . . . . . . . . .

46

Aula 3: Algumas Aplicações da Integral Dupla

47

3.1

Introdução . . . . . . . . . . . . . . . . . . . . . . .

48

3.2

Preliminares . . . . . . . . . . . . . . . . . . . . . .

48

3.3

Algumas Aplicações da Integral Dupla . . . . . . . .

52

3.4

Conclusão . . . . . . . . . . . . . . . . . . . . . . .

58

RESUMO . . . . . . . . . . . . . . . . . . . . . . . . .

58

PRÓXIMA AULA . . . . . . . . . . . . . . . . . . . .

59

ATIVIDADES . . . . . . . . . . . . . . . . . . . . . .

59

LEITURA COMPLEMENTAR . . . . . . . . . . .

60

Aula 4: Integrais triplas

63

4.1

Introdução . . . . . . . . . . . . . . . . . . . . . . .

64

4.2

Integração Tripla: Domínios Paralelepípedais . . . .

64

4.3

Integração Tripla: Domínios Não Paralelepípedais Limitados . . . . . . . . . . . . . . . . . . . . . . . .

66

4.4

Interpretação Geométrica . . . . . . . . . . . . . . .

67

4.5

Integrais Iteradas . . . . . . . . . . . . . . . . . . .

67

4.6

Propriedades das Integrais Triplas . . . . . . . . . .

68

4.7

Exemplos . . . . . . . . . . . . . . . . . . . . . . .

69

4.8

Conclusão . . . . . . . . . . . . . . . . . . . . . . .

74

RESUMO . . . . . . . . . . . . . . . . . . . . . . . . .

75

PRÓXIMA AULA . . . . . . . . . . . . . . . . . . . .

80

ATIVIDADES . . . . . . . . . . . . . . . . . . . . . .

80

LEITURA COMPLEMENTAR . . . . . . . . . . .

81
Aula 5: Mudança de Variáveis em Integrais tríplas

83

5.1

Introdução . . . . . . . . . . . . . . . . . . . . . . .

84

5.2

Mudança de Variáveis em Integrais Triplas . . . . . .

84

5.3

Alguns Exemplos . . . . . . . . . . . . . . . . . . .

87

5.4

Conclusão . . . . . . . . . . . . . . . . . . . . . . .

99

RESUMO . . . . . . . . . . . . . . . . . . . . . . . . .

99

PRÓXIMA AULA . . . . . . . . . . . . . . . . . . . . 103
ATIVIDADES . . . . . . . . . . . . . . . . . . . . . . 103
LEITURA COMPLEMENTAR . . . . . . . . . . . 104
Aula 6: Algumas Aplicações das Integrais tríplas

105

6.1

Introdução . . . . . . . . . . . . . . . . . . . . . . . 106

6.2

Preliminares . . . . . . . . . . . . . . . . . . . . . . 106

6.3

Algumas Aplicações da Integral Tripla . . . . . . . . 110

6.4

Conclusão . . . . . . . . . . . . . . . . . . . . . . . 117

RESUMO . . . . . . . . . . . . . . . . . . . . . . . . . 118
PRÓXIMA AULA . . . . . . . . . . . . . . . . . . . . 119
ATIVIDADES . . . . . . . . . . . . . . . . . . . . . . 120
LEITURA COMPLEMENTAR . . . . . . . . . . . 120
Aula 7: Integrais de Funções Vetoriais sobre Curvas em R3
123
7.1

Introdução . . . . . . . . . . . . . . . . . . . . . . . 124

7.2

Curvas em R3 . . . . . . . . . . . . . . . . . . . . . 124

7.3

Massa, Momento de Massa e Momento de Inércia de
Curvas em R3 . . . . . . . . . . . . . . . . . . . . . 126

7.4

Campos Vetoriais: Trabalho, Circulação e Fluxo . . . 128

7.5

Independência do Caminho . . . . . . . . . . . . . . 130

7.6

Algumas Aplicações das Integrais de Linha . . . . . 133
7.7

Conclusão . . . . . . . . . . . . . . . . . . . . . . . 138

RESUMO . . . . . . . . . . . . . . . . . . . . . . . . . 139
PRÓXIMA AULA . . . . . . . . . . . . . . . . . . . . 142
ATIVIDADES . . . . . . . . . . . . . . . . . . . . . . 142
LEITURA COMPLEMENTAR . . . . . . . . . . . 143
Aula 8: Integrais de Superfícies

145

8.1

Introdução . . . . . . . . . . . . . . . . . . . . . . . 146

8.2

Superfícies em R3 . . . . . . . . . . . . . . . . . . . 146

8.3

Área de Superfícies em R3 . . . . . . . . . . . . . . 147

8.4

Momento de massa e Momento de Inércia de Superfícies de Casca Fina em R3 . . . . . . . . . . . . . . 151

8.5

Superfícies Parametrizadas . . . . . . . . . . . . . . 155

8.6

Conclusão . . . . . . . . . . . . . . . . . . . . . . . 160

RESUMO . . . . . . . . . . . . . . . . . . . . . . . . . 160
PRÓXIMA AULA . . . . . . . . . . . . . . . . . . . . 164
ATIVIDADES . . . . . . . . . . . . . . . . . . . . . . 164
LEITURA COMPLEMENTAR . . . . . . . . . . . 165
Aula 9: Teorema de Green e Teorema de Stokes

167

9.1

Introdução . . . . . . . . . . . . . . . . . . . . . . . 168

9.2

Preliminares . . . . . . . . . . . . . . . . . . . . . . 168

9.3

Teorema de Green

9.4

Estendendo o Teorema de Green para Outras Regiões 175

9.5

Verificação do Teorema de Green . . . . . . . . . . 178

9.6

Teorema de Stokes . . . . . . . . . . . . . . . . . . 181

9.7

Aplicação do Teorema de Stokes . . . . . . . . . . . 183

9.8

Conclusão . . . . . . . . . . . . . . . . . . . . . . . 185

. . . . . . . . . . . . . . . . . . 171

RESUMO . . . . . . . . . . . . . . . . . . . . . . . . . 185
PRÓXIMA AULA . . . . . . . . . . . . . . . . . . . . 187
ATIVIDADES . . . . . . . . . . . . . . . . . . . . . . 187
LEITURA COMPLEMENTAR . . . . . . . . . . . 188
Aula 10: Teorema de Divergência

189

10.1 Introdução . . . . . . . . . . . . . . . . . . . . . . . 190
10.2 Preliminares . . . . . . . . . . . . . . . . . . . . . . 190
10.3 Teorema da Divergência . . . . . . . . . . . . . . . 191
10.4 Estendendo o Teorema da Divergência . . . . . . . . 194
10.5 Algumas Aplicações do Teorema da Divergência . . 196
10.6 Conclusão . . . . . . . . . . . . . . . . . . . . . . . 200
RESUMO . . . . . . . . . . . . . . . . . . . . . . . . . 200
ATIVIDADES . . . . . . . . . . . . . . . . . . . . . . 202
LEITURA COMPLEMENTAR . . . . . . . . . . . 203
AULA

Integrais Duplas
META:
Apresentar integrais duplas de funções de valores reais e domínio
em R2 .
OBJETIVOS:
Ao fim da aula os alunos deverão ser capazes de:
Definir a integral dupla de funções de valores reais e domínio em
R2 .
Calcular algumas integrais duplas de funções de valores reais e domínio em R2 .
PRÉ-REQUISITOS
Os conhecimentos de integrais de funções de valores reais com domínio em R, da disciplina Cálculo I.

1
Integrais Duplas

1.1

Introdução

Caros alunos iniciamos aqui nosso curso de Cálculo III com o
tema “Integrais Dupla”. A integração dupla, em essência, é uma
extensão natural da integral simples vista em Cálculo I e definida
como limite de somas de Riemann. Na prática, a integração dupla
é dada por duas integrações simples, cada uma efetuada sobre uma
variável e considerando as demais como constantes. É o que denominamos de integrais interadas. Suas características e detalhes
próprios serão vistas ao longo do nosso curso, nas próximas duas
aulas.

1.2

Integral Dupla: Domínios Retangulares

Começamos por considerar uma função f definida em um domínio retangular R = {(x, y) ∈ R2 |a ≤ x ≤ b∧c ≤ y ≤ d}. Formalmente f : [a, b]×[c, d] → R. Usando a imaginação, pensemos em R
coberta por uma rede de retas paralelas aos eixos coordenados e que
dividem R em pequenos retângulos (Fig. 1.1) . Oficialmente, consideraremos duas partições P [a, b] = {x0 = a, x1 , . . . , xj , xj+1 , . . . ,
xm = b} e P [c, d] = {y0 = c, y1 , . . . , yk , yk+1 , . . . , yn = d} onde
como visto em Cálculo I temos: x0 < x1 < · · · < xj < xj+1 <
· · · < xm e y0 < y1 < · · · < yk < yk+1 < · · · < yn .
Desta forma cada um dos Ij = [xj−1 , xj ] e Jk = [yk−1 , yk ] pequenos subintervalos têm comprimentos ∆xj = xj − xj−1 e ∆yk =
yk − yk−1 , respectivamente. Definimos, agora, a uma partição para
o retângulo R por P = P [R] = P [a, b] × P [c, d], o produto cartesiano das partições P [a, b] e P [c, d]. As retas retalham a região
R em uma série de retângulos Ajk = [xj−1 , xj ] × [yk−1 , yk ], 1 ≤

12
Cálculo III

AULA

1

Figura 1.1: Partição de R = [a, b] × [c, d]
j ≤ m, 1 ≤ k ≤ n. A area de cada pequeno retângulo é dada
por ∆Ajk = ∆xj ∆yk . Como tanto ∆xj quanto ∆yk são diferentes de zero, a área de cada pequeno retângulo é também diferente de zero. Podemos então definir a norma da partição por:
|P | = max (∆Ajk ), que corresponde a maior área entre todos os
1≤j≤m
1≤k≤n

pequeno retângulo.

Pausa para respirar que já vamos definir a integral dupla sobre
domínios retangulares. Para isto tomamos um ponto (ξj , ζk ) ∈
[xj−1 , xj ] × [yk−1 , yk ] em cada pequeno retângulo e definimos a seguinte soma de Riemann:
m

n

Smn =

f (ξj , ζk )∆Ajk
j=1 k=1

A integral dupla da função f (x, y) sobre o retângulo R, denotada
f (x, y)dxdy será então definida como o seguinte limite:
R
def

f (x, y)dxdy = lim Smn
R

BIOGRAFIA
Georg
Friedrich
Bernhard
Riemann
nasceu em Breselenz,
Reino de Hanôver,
17 de Setembro de
1826 e morreu em
Selasca, Itália, 20 de
Junho de 1866, foi um
matemático
alemão,
com
contribuições
fundamentais para a
análise e a geometria
diferencial. Wikipedia

|P |→0

13
Integrais Duplas

Figura 1.2: Soma de Riemann para f (x, y) em R = [a, b] × [c, d]

1.3

Integral Dupla: Domínios Não Retangulares Limitados

Para definir a integral dupla de uma função f : D ⊂ R2 → R
onde D é não é uma região retangular, porém é limitada, começamos por considerar uma função F definida em um domínio retangular R = {(x, y) ∈ R2 |a ≤ x ≤ b ∧ c ≤ y ≤ d} tal

 f (x, y) , (x, y) ∈ D
que D ⊂ R e F (x, y) =
. Formalmente
 0
, (x, y) ∈ D
/
F : [a, b] × [c, d] → R é uma extensão da função f (x, y). Usando
a imaginação, pensemos em R coberta por uma rede de retas paralelas aos eixos coordenados e que dividem R em pequenos retângulos e procedemos como na integral dupla sobre domínios retangulares, considerando a uma partição para o retângulo R por

14
Cálculo III

AULA

P = P [R] = P [a, b] × P [c, d], o produto cartesiano das partições

1

P [a, b] e P [c, d] onde P [a, b] = {x0 = a, x1 , . . . , xj , xj+1 , . . . , xm =
b} e P [c, d] = {y0 = c, y1 , . . . , yk , yk+1 , . . . , yn = d}. Do mesmo
modo definimos a norma da partição por: |P | = max (∆Ajk )
1≤j≤m
1≤k≤n

onde ∆Ajk = ∆xj ∆yk , ∆xj = xj − xj−1 e ∆yk = yk − yk−1 . Tomamos um ponto (ξj , ζk ) ∈ [xj−1 , xj ] × [yk−1 , yk ] em cada pequeno
retângulo e definimos a seguinte soma de Riemann para a função
estendida F (x, y):
m

n

Smn =

F (ξj , ζk )∆Ajk
j=1 k=1

A integral dupla da função f (x, y) sobre o domínio D ⊂ R2 , denof (x, y)dxdy será então definida como o seguinte limite:

tada
D

def

f (x, y)dxdy = lim Smn
D

|P |→0

Observem na partição (Fig. 1.3) que apenas os pequenos retângulos cinza claro contribuem para a soma de Riemann os demais
têm contribuição nula visto que o ponto escolhido dentro destes
estão fora de D ⊂ R2 e portanto F (ξj , ζk ) = 0.

1.4

Interpretação Geométrica

Quando a função f (x, y) é positiva na região R, como a da
(Fig. 1.2), vemos que a soma de Riemann aproxima o volume do
prisma sólido reto limitado inferiormente por R e superiormente
pela superfície z = f (x, y) e quanto maior for o refinamento da partição de R melhor será a aproximação. Podemos então, interpretar
f (x, y)dxdy como o volume do prisma sólido

a integral dupla
R

15
Integrais Duplas

Figura 1.3: Partição para F (x, y) em R = [a, b] × [c, d]
reto limitado inferiormente por R e superiormente pela superfície
z = f (x, y).

1.5

Integrais Iteradas

Do mesmo modo que para a integral simples, na integral dupla
a soma de Riemann não é um modo prático de se calcular uma
integral dupla. Vejamos agora um procedimento que facilitará o
cálculo de integrais duplas. Vamos exemplificar calculando o volume de um prisma reto de base retangular, limitado inferiormente
por [a, b] × [c, d] e superiormente pela função de valores positivos
f (x, y). para cada valor fixo de x no intervalo [a, b] consideremos
o perfil A(x) (área da seção transversal em x) (Fig. 1.4) fazemos
o produto por dx e integramos no intervalo [a, b]. Isto resulta no

16
Cálculo III

AULA

1

Figura 1.4: A(x), x fixo, integramos em relação a y
volume do citado prisma.
b

V =

A(x)dx
a

Por outro lado o perfil A(x) é dada pela área abaixo da curva
f (x, y), fixado o x, entre os valores de y no intervalo [c, d]. E como
d

f (x, y)dy.

vimos em Cálculo I A(x) =
c

O volume do prisma pode ser então escrito como:
b

d

V =

f (x, y)dy dx
a

c

.
Podemos alternativamente calcular o mesmo volume considerando
os perfis A(y) (área da seção transversal em y) (Fig. 1.5) fazemos
o produto por dy e integramos no intervalo [c, d]. Isto resulta no
volume do citado prisma.

17
Integrais Duplas

Figura 1.5: A(y), y fixo, integramos em relação a x

d

A(y)dy

V =
c

b

Da mesma forma como vimos em Cálculo I A(y) =

f (x, y)dx.
a

O volume do prisma pode ser então escrito como:
d

b

V =

f (x, y)dx dy
c

a

.
Como o volume dado pelas duas expressões é o mesmo temos que:
b

d

d

b

f (x, y)dx dy =
c

a

f (x, y)dy dx
a

c

ou seja a ordem em que as integrais simples são executadas não
altera o resultado final da integração dupla em domínios retangulares. Este procedimento e conhecido como integrais iteradas.

18
AULA

Cálculo III

1.6

Propriedades das Integrais Duplas

1

Como nosso curso é de Cálculo, apenas listaremos, sem demonstração, alguma das propriedades das integrais duplas. Caso
desejem conhecer a demonstração de algumas destas propriedades,
recomendo livros de Cálculo Avançado como os citados na bibliografia abaixo.
Propriedade 1.1. Sejam f : D ⊂ R2 → R uma função de valores
reais integrável em D e c ∈ R, então vale:

cf (x, y)dxdy = c

f (x, y)dxdy
D

D

Propriedade 1.2. Sejam f, g : D ⊂ R2 → R duas funções de
valores reais integráveis em D, então vale:

(f + g)(x, y)dxdy =
D

f (x, y)dxdy +
D

g(x, y)dxdy
D

Propriedade 1.3. Sejam f : D ⊂ R2 → R uma função de valores
reais integrável em D tal que f (x, y) ≥ 0, ∀(x, y) ∈ D, então vale:

f (x, y)dxdy ≥ 0
D

Propriedade 1.4. Sejam f, g : D ⊂ R2 → R duas funções de valores reais integráveis em D tais que f (x, y) ≥ g(x, y), ∀(x, y) ∈ D,
então vale:

f (x, y)dxdy ≥
D

g(x, y)dxdy
D

Propriedade 1.5. Seja f : D ⊂ R2 → R uma função de valores
reais integrável em D onde D = A ∪ B e A ∩ B é a união de um

19
Integrais Duplas
número finito de curvas em R2 , então vale:

f (x, y)dxdy =
D

f (x, y)dxdy +
A

f (x, y)dxdy
B

OBS 1.1. As duas primeiras propriedades diz respeito à “linearidade” do operador integral dupla. As terceira e quarta propriedades são denominadas “dominação” enquanto que a quinta
propriedade é denominada “aditividade”.

1.7

Alguns Exemplos

Nada mais natural que ilustrar um novo conceito com exemplos e, vamos aqui fazer exatamente isto, ilustrar o conceito de
integral dupla com dois exemplos. Antes porém, vale observar que
a na prática uma integral dupla equivale a duas integrais simples e
neste caso uma pergunta fica no ar e não deixaremos sem resposta.
Qual das duas variáveis x ou y integraremos primeiro? Muito bem,
a resposta é dada pela propria expressão da integral dupla. Isto
f (x, y)dxdy primeiramente integramos na va-

é, na integral
R

f (x, y)dydx

riável x e depois na variável y. Já na integral
R

primeiramente integramos na variável y e depois na variável x.

Vamos diretamente para o primeiro exemplo de integral dupla sobre domínios retangulares. A saber:
Exemplo 1.1. Considere a função f : [0, 1] × [0, 1] → R (Fig.
1.6) dada por f (x, y) = exp(−x − y) e determine a integral dupla
f (x, y)dxdy sobre a região R = {(x, y) ∈ R2 |0 ≤ x ≤

I =
R

1 ∧ 0 ≤ y ≤ 1}.

20
Cálculo III

AULA

1

Figura 1.6: Função f : [0, 1] × [0, 1] → R: f (x, y) = exp(−x − y)

SOLUÇÃO:
Passo 1 colocaremos os limites de integração que representam a
região R dada, segundo a ordem de integração:
1

1

exp(−x − y)dxdy

I=
0

0

Lembrando que: exp(−x − y) = exp(−x) exp(−y) temos:
1

1

exp(−x) exp(−y)dxdy

I=
0

0

Passo 2 integraremos na variável x considerando a variável y
como uma constante:
1

− exp(−x)

I=
0

1
0

exp(−y)dy

Substituindo os limites de integração temos:
1

(− exp(−1) − (− exp(−0))) exp(−y)dy

I=
0

Efetuando os cálculos temos:
1

(1 − exp(−1)) exp(−y)dy

I=
0

Passo 3 integraremos na variável y considerando a variável:
I = (1 − exp(−1)) − exp(−y)

1
0

Substituindo os limites de integração temos:
I = (1 − exp(−1)) (− exp(−1) − (− exp(−0)))
Efetuando os cálculos temos:

21
Integrais Duplas
I = (1 − exp(−1))2

OBS 1.2. Daremos aqui um método prático para determinar os
limites de integração em uma integral dupla sobre domínio não retangular da forma: D.

Passo 1 Fazer um desenho da região D. (Fig. 1.7) identificando
as curvas inferior a(x) e superior b(x) que limitam a região D.
Passo 2 Atravessar toda a região D e o eixo x com um segmento
de reta paralelo e orientado na direção positiva ao eixo y (segmento
AB na Fig. 1.7)
Passo 3 Deslocar o segmento de reta AB paralelo ao eixo y na

Figura 1.7: Determinação prática dos limites para D
direção negativa do eixo x até tocar o ponto mais à esquerda de D
marcando o limite inferior de x (ponto a na Fig. 1.7).
Passo 4 Deslocar o segmento de reta AB paralelo ao eixo y na
direção positiva do eixo x até tocar o ponto mais à direita de D
marcando o limite superior de x (ponto b na Fig. 1.7).
Passo 5 Tomando um ponto qualquer x ∈ (a, b) passamos o seg-

22
Cálculo III

AULA

mento de reta AB através da região D. O limite inferior para a

1

variável y será a função a(x), ponto da curva onde o segmento
entra na região D e o limite superior para a variável y será b(x),
ponto da curva onde o segmento de reta sai da região D.
Nossa integral será efetuada assim:
b

b(x)

f (x, y)dydx

f (x, y)dxdy =
D

a

a(x)

Exemplo 1.2. Considere a função f : [0, 1] × [0, 1] → R (Fig.
1.8) dada por f (x, y) = y(3x − x2 − y) e determine a integral duf (x, y)dxdy sobre a região D ∈ R2 interseção das

pla I =
R

curvas y = 0 e y = 3x − x2 .

Figura 1.8: Função f : [0, 1] × [0, 1] → R: f (x, y) = x.y
SOLUÇÃO:
Passo 1 faremos o desenho das duas curvas que determinam os
limites para a região D. A saber y = 0 e y = 3x − x2 (Fig. 1.9).
Passo 2 usando o processo prático exposto acima determinamos
os limites de integração. A saber: a = 0, b = 3, a(x) = 0 e
b(x) = 3x − x2 .

23
Integrais Duplas

Figura 1.9: Limites para o domínio D

A integral passa a ser escrita como:
3x−x2

3

I=

y(3x − x2 − y)dydx

f (x, y)dxdy =
0

0

Operando no integrando fazendo o produto por y temos:
3x−x2

3

(y(3x − x2 ) − y 2 )dydx

I=
0

0

Passo 3 efetuando a integração em y temos:
3 2
y
y 3 3x−x2
I=
( (3x − x2 ) − )
dx
2
3 0
0
Substituindo os limit3es de integração temos:
3
(3x − x2 )2
(3x − x2 )3
I=
(
(3x − x2 ) −
)dx
2
3
0
Efetuando as simplificações teremos:
3
(3x − x2 )3
I=
dx
6
0
Expandindo o binômio de Newton temos:
1 3
I=
(27x3 − 27x4 + 9x5 − x6 )dx
6 0
Passo 4 efetuando a integração em x temos:
1
x4
x5
x6 x7 3
I = (27 − 27 + 9 − )
6
4
5
6
7 0
Substituindo os limit3es de integração temos:
1
34
35
36 37
I = (27 − 27 + 9 − )
6
4
5
6
7
Efetuando os cálculos, garantido muito trabalho, temos:

24
Cálculo III

I=

1.8

AULA

1

729
280

Conclusão

Na aula de hoje, vimos que a integral dupla é uma extensão
natural do conceito de integral simples visto em Cálculo I. E se
por um lado a integral simples pode ser interpretada como a área
sob a curva descrita pela função a ser integrada, a integral dupla
pode ser vista como o volume sob a superfície descrita pela função
a ser duplamente integrada.

RESUMO

No nosso resumo da Aula 01 constam os seguintes tópicos:

Integração Dupla: Domínios retangulares
Considerando uma função f : R → R onde R = {(x, y) ∈ R2 |a ≤
x ≤ b ∧ c ≤ y ≤ d} é um retângulo em R2 . Podemos cobri-lo com
uma malha de retas formada pela partição: P = P [R] = P [a, b] ×
P [c, d] onde cada P [a, b] = {x0 = a, x1 , . . . , xj , xj+1 , . . . , xn = b}
e P [c, d] = {y0 = c, y1 , . . . , yk , yk+1 , . . . , ym = d} são partições
dos intervalos [a, b] em x e [c, d] em y respectivamente. A malha divide R nos retângulos Ajk = [xj−1 , xj ] × [yk−1 , yk ], 1 ≤ j ≤
n, 1 ≤ k ≤ m de área ∆Ajk = ∆xj ∆yk onde ∆xj = xj − xj−1
e ∆yk = yk − yk−1 são os comprimentos dos subintervalos Ij =
[xj−1 , xj ] e Jk = [yk−1 , yk ] respectivamente. Defini-se a norma da
partição por: |P | = max (∆Ajk ). Toma-se um ponto (ξj , ζk ) ∈
1≤j≤n
1≤k≤m

25
Integrais Duplas
[xj−1 , xj ] × [yk−1 , yk ] em cada retângulo Ajk e definimos a seguinte
soma de Riemann:
n

m

Snm =

f (ξj , ζk )∆Ajk
j=1 k=1

A integral dupla da função f (x, ) sobre o retângulo R, denotada
f (x, y)dxdy será então definida como o seguinte limite:
R
def

f (x, y)dxdy = lim Snm
|P |→0

R

Integração Dupla: Domínios não Retangulares
Para definir a integral dupla de uma função f : D ⊂ R2 → R
onde D é não é uma região retangular, porém é limitada, começamos por considerar uma função F definida em um domínio retangular R = {(x, y) ∈ R2 |a ≤ x ≤ b ∧ c ≤ y ≤ d} tal

 f (x, y) , (x, y) ∈ D
que D ⊂ R e F (x, y) =
. Formalmente
 0
, (x, y) ∈ D
/
F : [a, b] × [c, d] → R é uma extensão da função f (x, y). A partir
daqui todo o procedimento é semelhante ao da definição da integral dupla em domínios retangulares. Podemos definir a integral
dupla de uma função f (x, y) em um domínio não retangular D por:

def

f (x, y)dxdy = lim Smn
D

onde: Smn =

m
j=1

|P |→0

n
k=1 F (ξj , ζk )∆Ajk .

é a soma de Riemann

para F (x, y)

Integrais Iteradas
As integrais iteradas dizem que em um domínio retangular R =

26
Cálculo III

AULA

[a, b] × [c, d] a ordem de execução das integrais simples não alteram

1

o valor da integral dupla, que pode ser representada por:
d

b

b

d

f (x, y)dy dx

f (x, y)dx dy =
c

a

a

c

.
Propriedades das Integrais Duplas
As integrais duplas são de certo modo semelhantes às propriedades
das integrais simples que vimos em Cálculo I sendo quase que uma
extensão natural destas. As integrais duplas têm, entre outras, as
seguintes propriedades:
Propriedade 1 Sejam f : D ⊂ R2 → R uma função de valores
reais integrável em D e c ∈ R, então vale:

cf (x, y)dxdy = c
D

f (x, y)dxdy
D

Propriedade 2 Sejam f, g : D ⊂ R2 → R duas funções de valores
reais integráveis em D, então vale:

(f + g)(x, y)dxdy =
D

f (x, y)dxdy +
D

g(x, y)dxdy
D

Propriedade 3 Sejam f : D ⊂ R2 → R uma função de valores
reais integrável em D tal que f (x, y) ≥ 0, ∀(x, y) ∈ D, então vale:

f (x, y)dxdy ≥ 0
D

27
Integrais Duplas

Propriedade 4 Sejam f, g : D ⊂ R2 → R duas funções de valores reais integráveis em D tais que f (x, y) ≥ g(x, y), ∀(x, y) ∈ D,
então vale:

f (x, y)dxdy ≥
D

g(x, y)dxdy
D

Propriedade 5 Seja f : D ⊂ R2 → R uma função de valores
reais integrável em D onde D = A ∪ B e A ∩ B é a união de um
número finito de curvas em R2 , então vale:

f (x, y)dxdy =
D

f (x, y)dxdy +
A

f (x, y)dxdy
B

Determinação dos Limites de Integração
Para determinar os limites de integração em uma integral dupla
sobre domínio não retangular da forma: D seguimos os seguintes
passos:
Passo 1 Fazer um desenho da região D. (Fig. 1.7) identificando
as curvas inferior a(x) e superior b(x) que limitam a região D.
Passo 2 Atravessar toda a região D e o eixo x com um segmento
de reta paralelo e orientado na direção positiva ao eixo y (segmento
AB na Fig. 1.7)
Passo 3 Deslocar o segmento de reta AB paralelo ao eixo y na
direção negativa do eixo x até tocar o ponto mais à esquerda de D
marcando o limite inferior de x (ponto a na Fig. 1.7).

28
Cálculo III

AULA

Passo 4 Deslocar o segmento de reta AB paralelo ao eixo y na

1

direção positiva do eixo x até tocar o ponto mais à direita de D
marcando o limite superior de x (ponto b na Fig. 1.7).
Passo 5 Tomando um ponto qualquer x ∈ (a, b) passamos o segmento de reta AB através da região D. O limite inferior para a
variável y será a função a(x), ponto da curva onde o segmento
entra na região D e o limite superior para a variável y será b(x),
ponto da curva onde o segmento de reta sai da região D.
Nossa integral será efetuada assim:
b

b(x)

f (x, y)dxdy =
D

f (x, y)dydx
a

a(x)

PRÓXIMA AULA

Em nossa próxima aula veremos mudança de variáveis na integração dupla. O objetivo da mudança de variáveis em uma integral dupla será a de facilitar esta integração de uma de duas
formas. A primeira será tornando o integrando mais simples. A
segunda transformando o domínio D do integrando em um domínio
de forma geométrica mais simples.

ATIVIDADES

Deixamos como atividades o cálculo de algumas integrais duplas.

ATIV. 1.1. Seja f : [−1, +1] × [−1, +1] → R dada por f (x, y) =

29
Integrais Duplas
x2 + y 2 . Determine a integral dupla

f (x, y)dxdy.
R

Comentário:

Volte ao texto e reveja com calma e atenção o

cálculo de integrais duplas dos exemplos acima, elas lhe servirão
de guia.
ATIV. 1.2. Seja f : D ⊂ R2 → R dada por f (x, y) = x2 + y 2 ,
onde D = {(x, y) ∈ R2 |x ≥ 0 ∧ 0 ≤ y ≤ 1 − x2 }.

• Determine os limites da integral dupla

f (x, y)dxdy,
D

• esboce a região de integração e
• calcule a integral dupla

f (x, y)dxdy.
D

Comentário:

Volte ao texto e reveja com calma e atenção o

cálculo de integrais duplas dos exemplos acima, elas lhe servirão
de guia.

LEITURA COMPLEMENTAR

ÁVILA, Geraldo, Cálculo 3: Funções de Várias Variáveis, Livros
Técnicos e Científicos Editora, São Paulo, 3a edição, 1982.
LEITHOLD, Louis, O Cálculo com Geometria Analítica. Volume
2, Editora Harbra, 1994.
STEWART, James,Cálculo. Volume 3, 5a edição, Editora CENGAGE Learning, 2009.
SWOKOWSKI, Earl E., Cálculo com Geometria Analítica, Volume
2, 2a edição, Makron Books do Brásil SP, 1994.
THOMAS, George B., Cálculo, Volume 2, 10a, Addilson Wesley,

30
Cálculo III
2003.

AULA

1

KAPLAN, Wilfred, Cálculo Avançado Vol.1 e vol.2 Editora Edgard
Blücher 1991.// SPIEGEL, Murray R. Cálculo Avançado, Editora
McGraw-Hill do Brasil, 1971.
BOUCHARA, Jacques, Cálculo Integral Avançado, EDUSP, 2006.

31
AULA

Mudança de Variáveis em
Integrais Duplas
META:
Introduzir mudança de variáveis em integrais duplas de funções de
valores reais e domínio em R2 .
OBJETIVOS:
Ao fim da aula os alunos deverão ser capazes de:
Calcular o jacobiano de aplicações de R2 em R2 .
Calcular integrais duplas de funções de valores reais e domínio em
R2 utilizando mudança de variáveis.
Calcular integrais duplas de funções de valores reais e domínio em
R2 em coordenadas polares.
PRÉ-REQUISITOS
Os conhecimentos de integrais de funções de valores reais com domínio em R, da disciplina Cálculo I, curvas em R2 e coordenadas
polares da disciplina Cálculo II e integrais duplas aula 01.

2
Mudança de Variáveis em Integrais Duplas

2.1

Introdução

Caros alunos a segunda aula do nosso curso de Cálculo III
tem com o tema “Mudança de Variáveis em Integrais Duplas”. As
f (x, y)dxdy, dada a natureza ou de

vezes, na integral dupla
HISTÓRIA
O teorema de mudança de variáveis
em integrais duplas
foi primeiro proposto
por Euler quando ele
desenvolveu a noção
de integral dupla em
1769.
Usado por
Legendre, Laplace e
Gauss, foi primeiramente
generalizado
para n variáveis por
Mikhail Ostrogradski
em 1836, resistiu a
uma
demonstração
mais rigorosa por longo
tempo (cerca de 125
anos). E foi satisfatóriamente demonstrado
por Elie Cartan em
uma série de artigos
nos anos 1890.

D

f (x, y) ao do seu domínio D, fica mais fácil integrar se fizermos
uma mudança nas variáveis de integração, como quando D é uma
disco, um semi-disco, um setor circular ou mesmo uma faixa de
disco, usando-se o sistema de coordenadas polares de modo geral
a integral dupla é mais fácil de se determinar que em coordenadas
cartesianas.

2.2

Mudança de Variáveis em Integrais Duplas

Caros alunos começaremos revendo mudança de variáveis em
integrais simples. Considere uma função f : [a, b] → R. A idéia
é mudar a variável inicial x para uma nova variável ξ relacionadas por x = g(ξ), onde g(ξ) é uma função biunívoca estritamente
crescente ou estritamente decrescente em [a, b]. Isto garante que
podemos inverter a mudança de variáveis.
Seja F (x) uma anti-derivada de f (x) tal que F (x) = f (x). Então,
da regra da cadeia temos:
d
F (g(ξ)) = F (g(ξ))g (ξ) = f (g(ξ))g (ξ).
dξ
Integrando com respeito a ξ temos:
d
F (g(ξ))dξ = f (g(ξ))g (ξ)dξ
dξ
Das propriedades da integral temos:
F (g(ξ)) + C =

f (g(ξ))g (ξ)dξ

Como x = g(ξ) temos:
F (x) + C =

34

f (g(ξ))g (ξ)dξ
Cálculo III

AULA

Como F (x) é uma primitiva de f (x) a primeira expressão é a in-

2

tegral indefinida de f (x) com respeito a x e temos:
f (x)dx =

f (g(ξ))g (ξ)dξ

Que representa a mudança de variáveis em uma integral simples.
Para integrais definidas, se c = g(a) e d = g(b) então:
d

b

f (g(ξ))g (ξ)dξ

f (x)dx =
c

a

A expressão acima funciona bem quando g(ξ) é crescente neste
caso a < b e c < d. Porém, no caso de g(ξ) decrescente (g (ξ) < 0)
pois neste caso a < b e d < c e portanto o limite inferior da segunda integral não conhecide com o limite inferior do intervalo da
imagem de g(ξ) o mesmo acontecendo com o limite superior. Neste
caso, usando as propriedades da integral simples temos:
b

c

f (x)dx = −

f (g(ξ))g (ξ)dξ

a

d

De outra forma escrevemos:
b

c

f (x)dx =
a

f (g(ξ))|g (ξ)|dξ.
d

e operaremos os limites inferiores e superiors das integrais como os
limites inferiores e superiores dos domínios (intervalos) e a expressão acima vale tanto pra g(ξ) crescente quanto decrescente. Vamos
agora diretamente ao assunto dando uma argumentação heurística
para a expressão da mudança de variáveis em integrais duplas.
f (x, y)dxdy sobre

Para isto, consideremos a integral dupla
D

uma região D ∈ R2 do plano (x, y) e a transformação (x, y) =
T (u, v) tal que o domínio D do plano (x, y) seja a imagem do
domínio D do plano (u, v) (podemos expressar este fato como
D = T (D )). Mais especificamente podemos escrever: x = x(u, v)
ˆ
e y = y (u, v) tomando uma partição para o domínio D no plano
ˆ

OBSERVAÇÃO
heurística heu.rís.ti.ca
sf (gr heuristiké) 1
Ciência ou arte do procedimento heurístico.
2 Método de ensino
que consiste em que
o educando chegue à
verdade por seus próprios meios. 3 Ramo
da ciência histórica que
consiste na pesquisa
dos documentos do
passado.

(u, v) cobrindo-o com pequenos retângulos e usando a transformação T podemos levar o pequeno retângulo Ajk na pequena figura

35
Mudança de Variáveis em Integrais Duplas
plana Ajk = T (Ajk ) (ver Fig 2.1 e Fig 2.2). A área do pequeno
retângulo no plano (u, v) é ∆Ajk a área da pequena figura Ajk no
plano (x, j), e ai é que reside a argumentação heurística, será apro∂T
ximada pela área do paralelogramo formado pelos vetores
∆vk
∂v
∂T
e
∆uj e pelas linhas tracejadas (paralelas aos respectivos veto∂u
res). Do calculo vetorial temos:
∂T
∂x
ˆ
∂y
ˆ
∆uj =
∆uj i +
∆uj j + 0k.
∂u
∂u
∂u
∂T
∂x
ˆ
∂y
ˆ
∆vk =
∆vk i +
∆vk j + 0k
∂v
∂v
∂v
Vistos como vetores de R3 e a área do paralelogramo (ver Vetores
e Geometria Analítica) dada pelo módulo do seguinte produto vetorial:
∂T
∂T
∆uj ×
∆vk .
∂u
∂v
Fazendo o cálculo do produto vetorial temos:

∆Ajk =



i
j
k 

 ∂x

ˆ
∂y
ˆ
∂T
∂T

∆uj ×
∆vk = det 
 ∂u ∆uj ∂u ∆uj 0 
∂u
∂v
 ∂x

∂y
ˆ
ˆ
∆vk
∆vk 0
∂v
∂v
Fazendo os cálculos temos:
∂T
∂T
∂x ∂y ∂x ∂y
ˆ ˆ
ˆ ˆ
∆uj ×
∆vk =
−
∆uj ∆vk k.
∂u
∂v
∂u ∂v
∂v ∂u
Tomando o módulo da expressão acima, para a área de Ajk , temos:
ˆ ˆ
∂x ∂y ∂x ∂y
ˆ ˆ
−
∆uj ∆vk .
∆Ajk ≈
∂u ∂v
∂v ∂u
A expressão dentro do módulo é o determinante de uma matrix
2 × 2 conhecida como jacobiano da transformação x = x(u, v) e
ˆ
y = y (u, v) e denotado:
ˆ


∂x ∂y
ˆ
ˆ
ˆ ˆ
ˆ ˆ
∂(x, y)

 ∂x ∂y ∂x ∂y
= det  ∂u ∂u  =
−
.
∂x ∂y
ˆ
ˆ
∂(u, v)
∂u ∂v
∂v ∂u
∂v ∂v
Como a área do pequeno retângulo Ajk é dada por ∆Ajk = ∆uj ∆vk
temos:
∆Ajk ≈

36

∂(x, y)
∆Ajk .
∂(u, v)
Cálculo III

AULA

2

Figura 2.1: Plano (u, v)

Figura 2.2: Plano (x, y)

O que nos leva a considerar a seguinte fórmula para a mudança de
variáveis em integrais duplas:
∂(x, y)
dudv.
∂(u, v)
D
D
Que representa a mudança de variáveis na integral dupla pela
f (x, y)dxdy =

f (ˆ(u, v), y (u, v))
x
ˆ

transformação (x, y) = T (u, v).
OBS 2.1. Para o caso particular da mudança de variáveis do sistema de coordenadas cartesianas para o sistema de coordenadas
polares (x, y) = T (r, ϑ) = (r cos(ϑ), r sin(ϑ)) onde x = x(r, ϑ) =
ˆ
r cos(ϑ) e y = (r, ϑ) = r sin(ϑ), o jacobiano é dado por:
y
ˆ



∂x ∂y
ˆ
ˆ
cos(ϑ)
sin(ϑ)
∂(x, y)
 ∂r ∂r 
 = r.
= det  ∂ x ∂ y  = det 
ˆ
ˆ
∂(r, ϑ)
−r sin(ϑ) r cos(ϑ)
∂ϑ ∂ϑ
∂(x, y)
Portanto o jacobiano da transformação
= r a mudança de
∂(r, ϑ)
variáveis na integral dupla toma a forma:
f (x, y)dxdy =
D

f (r cos(ϑ), r sin(ϑ))rdrdϑ.
D

OBS 2.2. Daremos aqui um método prático para determinar os
limites de integração em uma integral dupla sobre domínio não retangular da forma: D em coordenadas polares.

37
Mudança de Variáveis em Integrais Duplas
Passo 1 Fazer um desenho da região D (Fig. 2.3), identificando
as curvas que limitam a região D.
Passo 2 Atravessar toda a região D com um raio r (ϑ) orientado
na direção positiva (Fig. 2.3)
Passo 3 Deslocar o raio r (ϑ) na direção negativa do ângulo ϑ (diNOTA

reção horária) até tocar o ponto mais à negativa de D marcando

Por
convenção
a
medida de ângulo tem
sinal positivo quando o
deslocamento é feito na
direção anti-horária,
direção contrária ao
movimento dos ponteiros do relógio e tem
sinal negativo quando
o deslocamento é feito
na direção horária,
direção do movimento
dos
ponteiros
do
relógio.

Figura 2.3: Determinação prática dos limites para D
o limite inferior de ϑ (ângulo α na Fig. 2.3).
Passo 4 Deslocar o raio r (ϑ) na direção positiva do ângulo ϑ
(direção anti-horária) até tocar o ponto mais à positiva de D marcando o limite inferior de ϑ (ângulo β na Fig. 2.3).
Passo 5 Tomando um ponto qualquer ϑ ∈ (α, β) passamos o raio
r (ϑ) através de D o limite inferior para a variável r será a função
α(ϑ), ponto da curva onde o raio r (ϑ) entra na região D e o limite
superior para a variável r será β(ϑ), ponto da curva onde o raio
r (ϑ) sai da região D.
Nossa integral será efetuada assim:
β

β(ϑ)

f (x, y)dxdy =
D

38

f (r cos(ϑ), r sin(ϑ))rdrdϑ
α

α(ϑ)
AULA

Cálculo III

2.3

2

Alguns Exemplos

Caros alunos, nesta seção ilustraremos, com dois exemplos, a
mudança de variáveis em integrais duplas. A rigor, trataremos
apenas de exemplos em coordenadas polares.

f (x, y)dxdy onde

Exemplo 2.1. Determinar a integral dupla
D

D = {(x, y) ∈ R2 |x ≥ 0 ∧ y ≥ 0 ∧ x2 + y 2 ≤ 1} e f (x, y) =
exp(−x2 − y 2 ). O domínio da função representa um quarto de
disco (Fig 2.4).

Figura 2.4: Gráfico do exemplo 1
SOLUÇÃO:
Passo 1 Como o domínio D é um quarto de disco, o mais adequado é utilizar o sistema de coordenadas polares. Podemos usar
o método prático de determinação dos limites da integral dupla
π
em coordenadas polares (Fig 2.5) e verificar que: α = 0, β = ,
2
α(ϑ) = 0 e β(ϑ) = 1.
Neste caso podemos descrever o domínio como: D = {(r, ϑ) ∈
R2 |0 ≤ r ≤ 1 ∧ 0 ≤ ϑ ≤ π/2}. E como x = r cos(ϑ) e y = r sin(ϑ) e

39
Mudança de Variáveis em Integrais Duplas

Figura 2.5: Gráfico do exemplo 1

∂(x, y)
= r.
∂(r, ϑ)
Quanto a variável r varia no intervalo [0, 1] independentemente de
o módulo do jacobiano da transformação é dado por:

ϑ e a variável ϑ varia no intervalo [0, π/2] ( a variação de ângulo no
primeiro quadrante). Podemos reescrever a integral dupla como:
1

I=

π/2

f (x, y)dxdy =
D

f (r cos(ϑ), r sin(ϑ))rdϑdr Subs0

0

tituindo f (x, y) temos:
1

π/2

exp(−(r cos(ϑ))2 − (r sin(ϑ))2 )rdϑdr

I=
0

0

Efetuando as simplificações temos:
1

π/2

exp(−r2 )rdϑdr

I=
0

0

Passo 2 Integrando primeiramente na variável ϑ e como o integrando não depende de ϑ temos:
1

exp(−r2 )ϑ

I=
0

π/2
0

rdr

Substituindo os limites de integração temos:
1

exp(−r2 )rdr

I = π/2
0

Passo 3 A última integral (variável r) podemos efetuar por mudança de variáveis pondo ξ = r2 
deste modo temos: dξ = 2rdr

 1
 1
1
ou seja rdr = − dξ e os limites r
eξ
. Daí, a integral
 0
 0
2

40
Cálculo III

2

passará a forma:
1

I = π/4

AULA

exp(−ξ)dξ
0

Cuja integração é fácil e da forma:
I = π/4 − exp(−ξ)

1
0

Efetuando os cálculo temos:
π
I = (1 − exp(−1))
4

Vamos agora, diretamente ao nosso segundo exemplo. Trata-se de
uma curva já conhecida de vocês (Cálculo II) a lemniscata.
Exemplo 2.2. Determinar a área da região D, a parte da lemniscata, r =

cos(2ϑ), que situa-se no primeiro quadrante. ver parte

cinza da (Fig 2.6).

Figura 2.6: Gráfico do exemplo 2
SOLUÇÃO:
Passo 1

Como o domínio D é um quarto de uma lemniscata, o

mais adequado é utilizar o sistema de coordenadas polares. Podemos usar o método prático de determinação dos limites da integral
dupla em coordenadas polares (Fig 2.7) e verificar que: α = 0,

41
Mudança de Variáveis em Integrais Duplas
β=

π
, α(ϑ) = 0 e β(ϑ) =
4

cos(2ϑ).

Figura 2.7: Gráfico do exemplo 2
Neste caso podemos descrever o domínio como: D = {(r, ϑ) ∈
R2 |0 ≤ ϑ ≤ π/4 ∧ 0 ≤ r ≤

cos(2ϑ)}. E como, neste exemplo,

queremos calcular área temos que f (x, y) = 1 e em coordenadas
polares podemos escrever na forma da seguinte integral dupla:
√
cos(2ϑ)

π/4

A=

rdrdϑ

dxdy =
D

0

0

Integrando em r temos:
√
π/4 2
cos(2ϑ)
r
A=
dϑ
2 0
0
Substituindo os limites de integração temos:
2
π/4
cos(2ϑ)
dϑ
A=
2
0
Simplificando o integrando temos:
π/4
cos(2ϑ)
A=
dϑ
2
0
Integrando na variável ϑ temos:
sin(2ϑ) π/4
A=
4
0
Substituindo os limites de integração temos:
sin(π/2) − sin(0)
A=
4
Portanto:
1
A=
4

42
Cálculo III

AULA

2
OBS 2.3. Caros alunos, é muito importante neste ponto uma revisão cuidadosa e detalhada dos dois exemplos dados acima. Efetuar
uma mudança de varáveis em integrais duplas não é tão simples
quanto efetuar uma mudança de variáveis em integrais simples.

2.4

Conclusão

Na aula de hoje, vimos que a mudança de variáveis em integrais
dupla, nos permite, facilitar o cálculo das ditas integrais quando
trabalhamos com domínios de integração de geometrias específicas,
como a induzida pelas coordenadas polares.

RESUMO

No nosso resumo da Aula 02 constam os seguintes tópicos:

Mudança de Variáveis em Integrais Duplas
Consideramos a transformação (x, y) = T (u, v) tal que o domínio
D do plano (x, y) seja transformado no domínio D do plano (u, v)
(D = T (D )) e mais especificamente x = x(u, v) e y = y (u, v).
ˆ
ˆ
∂(x, y)
Definindo o jacobiano da transformação, denotado
, por:
∂(u, v)


ˆ
∂x ∂y
ˆ
∂(x, y)
ˆ ˆ
ˆ ˆ

 ∂x ∂y ∂x ∂y
= det  ∂u ∂u  =
−
.
∂x ∂y
ˆ
ˆ
∂(u, v)
∂u ∂v
∂v ∂u
∂v ∂v
Vale então,a seguinte fórmula para a mudança de variáveis en integrais duplas:

43
Mudança de Variáveis em Integrais Duplas

f (ˆ(u, v), y (u, v))
x
ˆ

f (x, y)dxdy =
D

D

∂(x, y)
dudv.
∂(u, v)

Sistema de Coordenadas Polares
Para o caso particular da mudança de variáveis do sistema de coordenadas cartesianas para o sistema de coordenadas polares no
cálculo de integrais duplas temos:
(x, y) = T (r, ϑ) = (r cos(ϑ), r sin(ϑ)) onde x = x(r, ϑ) = r cos(ϑ) e
ˆ
y = y (r, ϑ) = r sin(ϑ).
ˆ
Vale a seguinte transformação de variáveis:

f (x, y)dxdy =
D

f (r cos(ϑ), r sin(ϑ))rdrdϑ.
D

Determinação dos Limites de Integração em Coordenadas Polares
Daremos aqui um método prático para determinar os limites de
integração em uma integral dupla sobre domínio não retangular
da forma: D em coordenadas polares.
Passo 1 Fazer um desenho da região D (Fig. 2.3), identificando
as curvas que limitam a região D.
Passo 2 Atravessar toda a região D com um raio r (ϑ) orientado
na direção positiva (Fig. 2.3)
Passo 3 Deslocar o raio r (ϑ) na direção negativa do ângulo ϑ (direção horária) até tocar o ponto mais à negativa de D marcando
o limite inferior de ϑ (ângulo α na Fig. 2.3).
Passo 4 Deslocar o raio r (ϑ) na direção positiva do ângulo ϑ
(direção anti-horária) até tocar o ponto mais à positiva de D marcando o limite inferior de ϑ (ângulo β na Fig. 2.3).
Passo 5 Tomando um ponto qualquer ϑ ∈ (α, β) passamos o raio

44
Cálculo III

AULA

r (ϑ) através de D o limite inferior para a variável r será a função

2

α(ϑ), ponto da curva onde o raio r (ϑ) entra na região D e o limite
superior para a variável r será β(ϑ), ponto da curva onde o raio
r (ϑ) sai da região D.
Nossa integral será efetuada assim:
β

β(ϑ)

f (x, y)dxdy =
D

f (r cos(ϑ), r sin(ϑ))rdrdϑ
α

α(ϑ)

PRÓXIMA AULA

Em nossa próxima aula veremos algumas das inúmeras aplicações da integral dupla. Nossa atenção estará voltada para o cálculo
do centro de massa de perfis planos bem como no cálculo de seus
momentos de inércia.

ATIVIDADES

Deixamos como atividades as seguintes questões.

ATIV. 2.1. Determine a área da parte da cardióide r(ϑ) = 1 +
cos(ϑ) que fica acima do eixo dos x (Fig 2.8) que está em cinza.
Comentário:

Volte ao texto e reveja com calma e atenção os

exemplos acima, eles lhe servirão de guia.
ATIV. 2.2. Determine a área entre a cardióide r(ϑ) = 1+cos(ϑ) e
o círculo r(ϑ) = 1 acima do eixo do x (Fig 2.9) que está em cinza.
Comentário:

Volte ao texto e reveja com calma e atenção os

exemplos acima, eles lhe servirão de guia.

45
Mudança de Variáveis em Integrais Duplas

Figura 2.8: Atividade 1

Figura 2.9: Atividade 2

LEITURA COMPLEMENTAR

ÁVILA, Geraldo, Cálculo 3: Funções de Várias Variáveis, Livros
Técnicos e Científicos Editora, São Paulo, 3a edição, 1982.
LEITHOLD, Louis, O Cálculo com Geometria Analítica. Volume
2, Editora Harbra, 1994.
STEWART, James,Cálculo. Volume 3, 5a edição, Editora CENGAGE Learning, 2009.
SWOKOWSKI, Earl E., Cálculo com Geometria Analítica, Volume
2, 2a edição, Makron Books do Brásil SP, 1994.
THOMAS, George B., Cálculo, Volume 2, 10a, Addilson Wesley,
2003.
KAPLAN, Wilfred, Cálculo Avançado Vol.1 e vol.2 Editora Edgard
Blücher 1991.// SPIEGEL, Murray R. Cálculo Avançado, Editora
McGraw-Hill do Brasil, 1971.
BOUCHARA, Jacques, Cálculo Integral Avançado, EDUSP, 2006.

46
AULA

Algumas Aplicações da
Integral Dupla
META:
Apresentar algumas aplicações das integrais duplas de funções de
valores reais e domínio em R2 .
OBJETIVOS:
Ao fim da aula os alunos deverão ser capazes de:
Determinar área, massa, centro de massa, momento de massa e
momento de inércia de figuras planas usando integrais duplas de
funções de valores reais e domínio em R2 .
PRÉ-REQUISITOS
Os conhecimentos de integrais de funções de valores reais com domínio em R, da disciplina Cálculo I, curvas em R2 e coordenadas
polares da disciplina Cálculo II e integrais duplas aula 01 e aula 02.

3
Algumas Aplicações da Integral Dupla

3.1

Introdução

Caros alunos nesta terceira aula do nosso curso de Cálculo III
com o tema “Algumas Aplicações das Integrais Duplas”. Dentre as
inúmeras aplicações da integral dupla, veremos apenas duas pelo
pouco tempo que dispomos. Veremos apenas como usar as integrais duplas para calcular a massa de uma região plana dada sua
distribuição de densidade e como calcular seu centro de gravidade.
Para outras aplicações recomendo uma busca na INTERNET

3.2

Preliminares

Consideraremos uma região D ⊂ R2 finita, com uma distribuição de densidade mássica superficial (massa por unidade de
superfície) (x, y), ∀(x, y) ∈ D.
Determinação da massa
Para determinar a massa consideremos uma função Φ definida
em um domínio retangular R = {(x, y)  R2 |a ≤ x ≤ b ∧ c ≤
∈
 (x, y) , (x, y) ∈ D
y ≤ d} tal que D ⊂ R e Φ(x, y) =
.
 0
, (x, y) ∈ D
/
Considerando a uma partição para o retângulo R dada por P =
P [R] = P [a, b] × P [c, d], o produto cartesiano das partições P [a, b]
e P [c, d] onde P [a, b] = {x0 = a, x1 , . . . , xj , xj+1 , . . . , xm = b}
e P [c, d] = {y0 = c, y1 , . . . , yk , yk+1 , . . . , yn = d}. Tomamos um
ponto (ξj , ζk ) ∈ [xj−1 , xj ] × [yk−1 , yk ] em cada pequeno retângulo
e definimos a seguinte soma de Riemann:
m

n

Smn =

Φ(ξj , ζk )∆Ajk .
j=1 k=1

48
Cálculo III

AULA

A massa da região D, denotada m(D), será a integral dupla da fun-

3

ção (x, y) sobre o domínio D ⊂ R2 , denotada

(x, y)dxdy
D

será então definida como o seguinte limite:
def

(x, y)dxdy = lim Smn

m(D) =

|P |→0

D

.

OBS 3.1. Para a determinação do peso da região D toma-se a
seguinte soma de Riemann:
m

n

Smn =

g(ξj , ζk )Φ(ξj , ζk )∆Ajk
j=1 k=1

onde g(ξj , ζk ) é a aceleração da gravidade no ponto (ξj , ζk ). E o
peso da região D, denotado p(D), será dado pela integral dupla:
def

p(D) =

g(x, y) (x, y)dxdy = lim Smn
|P |→0

D

.

Determinação do Momento de Massa
Usando as mesmas considerações acima para o cálculo da massa
de uma região D limitada com distribuição de densidade (x, y).
Para calcular o momento de massa de um pequeno retângulo com
relação ao eixo y tomamos o seguinte produto ξj Φ(ξj , ζk )∆Ajk . O
momento de massa total em relação ao eixo y para a região D será
aproximado pelo limite da soma de Riemann:
m

n

Smn =

ξj Φ(ξj , ζk )∆Ajk
j=1 k=1

.
O momento de massa da região D em relação ao eixo y será dada

49
Algumas Aplicações da Integral Dupla
pelo limite:
def

x (x, y)dxdy = lim Smn

My (D) =

|P |→0

D

.

De forma semelhante chega-se ao momento de massa da região D
em relação ao eixo x tomando-se a seguinte soma de Riemann:
m

n

Smn =

ζk Φ(ξj , ζk )∆Ajk
j=1 k=1

.
O momento de massa da região D em relação ao eixo x será dada
pelo limite:
def

Mx (D) =

y (x, y)dxdy = lim Smn
D

|P |→0

.

Determinação do Centro de Massa
O centro de massa de uma região plana D ⊂ R2 finita, com uma
distribuição de densidade mássica superficial (x, y), ∀(x, y) ∈ D,
é o ponto (¯, y ) definido por:
x ¯
x=
¯

My (D)
=
m(d)

x (x, y)dxdy
D

(x, y)dxdy
D

Mx (D)
y=
¯
=
m(d)

y (x, y)dxdy
D

(x, y)dxdy
D

Determinação do Momento de Inércia
Usando as mesmas considerações acima para o cálculo da massa

50
Cálculo III

AULA

de uma região D limitada com distribuição de densidade (x, y).

3

Para calcular o momento de inércia de um pequeno retângulo com
2
relação ao eixo y tomamos o seguinte produto ξj Φ(ξj , ζk )∆Ajk . O

momento de inércia total em relação ao eixo y para a região D será
aproximado pelo limite da soma de Riemann:
m

n
2
ξj Φ(ξj , ζk )∆Ajk

Smn =
j=1 k=1

.
O momento de inércia da região D em relação ao eixo y será dada
pelo limite:

def

x2 (x, y)dxdy = lim Smn

Iy (D) =

|P |→0

D

.

De forma semelhante chega-se ao momento de inércia da região D
em relação ao eixo x tomando-se a seguinte soma de Riemann:
m

n
2
ζk Φ(ξj , ζk )∆Ajk

Smn =
j=1 k=1

.
O momento da região D em relação ao eixo x será dada pelo limite:

def

y 2 (x, y)dxdy = lim Smn

Ix (D) =
D

|P |→0

.

O momento de inércia em relação a origem é dado pela seguinte
integral dupla:

51
Algumas Aplicações da Integral Dupla

(x2 + y 2 ) (x, y)dxdy

I0 (D) =
D

.

3.3

Algumas Aplicações da Integral Dupla

Faremos duas aplicações da integral dupla ao cálculo do centro
de massa de duas figuras planas. Na primeira usaremos o sistema
de coordenadas cartesiano. Na segunda usaremos uma mudança
de variáveis para o sistema de coordenadas polares.
Vamos aos nossos exemplos.

Exemplo 3.1. Para o primeiro exemplo desejamos determinar o
centro de massa de uma região triangular D dada pela interseção
das retas x = 0, y = 0 e a reta que passa pelos pontos (0, a) e
(b, 0) com a, b > 0 (Fig 3.1), cuja densidade superficial de massa
é constante (x, y) = .

Figura 3.1: Gráfico do exemplo 1

52
AULA

Cálculo III

3

SOLUÇÃO:
Começaremos por determinar os limites de integração inspeciox
nando a (Fig 3.1) e verificando que 0 ≤ x ≤ a e 0 ≤ y ≤ b 1 − .
a
Em segundo calcularemos a massa da região D, m(D) e os respectivos momentos de massa com relação ao eixo x e ao eixo y Mx (D)
e My (D) respectivamente.
Passo 1 determinar a massa m(D), dada pela integral dupla:
a

m(D) =

b(1−x/a)

(x, y)dxdy =

dydx
0

D

0

Integrando em y temos:
a

m(D) =

b(1−x/a)

y
0

0

dx

Substituindo os limites de integração temos:
a
x
m(D) =
b 1 − dx
a
0
Integrando em x temos:
x2 a
m(D) = b x −
2a 0
Substituindo os limites de integração temos:
a2
m(D) = b a −
2a
Simplificando temos:
ab
m(D) =
2
Passo 2 calcular o momento de massa Mx (D) dado pela integral
dupla:
Mx (D) =

(x, y)ydxdy
D

Substituindo os limites temos:
a

Mx (D) =

b(1−x/a)

(x, y)ydxdy =
D

ydydx
0

0

Integrando em y teremos:
a
y 2 b(1−x/a)
dx
Mx (D) =
2 0
0
Substituindo os limites de integração temos:
a
(b(1 − x/a))2
Mx (D) =
dx
2
0
Simplificando o integrando temos:

53
Algumas Aplicações da Integral Dupla
a

b2 b2 x b2 x2
dx
−
+
2
a
2a2
0
Integrando em x teremos:
b2 x b2 x2 b2 x3 a
Mx (D) =
−
+
2
2a
6a2 0
Substituindo os limites de integração temos:
b2 a b2 a2 b2 a3
Mx (D) =
−
+
2
2a
6a2
Simplificando as frações temos:
b2 a
Mx (D) =
6
Passo 3 calcular o momento de massa My (D) dado pela integral
Mx (D) =

dupla:
My (D) =

(x, y)xdxdy
D

My (D) =

(x, y)xdxdy
D

Substituindo os limites temos:
a

My (D) =

b(1−x/a)

(x, y)xdxdy =
D

xdydx
0

0

Integrando em y teremos:
a

b(1−x/a)

xy

My (D) =
0

0

dx

Substituindo os limites de integração temos:
a
x
My (D) =
bx 1 − dx
a
0
Integrando em x teremos:
x2 x3 a
My (D) = b
−
2
3a 0
Substituindo os limites de integração temos:
a3
a2
−
My (D) = b
2
3a
Simplificando as frações temos:
ba2
My (D) =
6
Passo 4 Determinar o centro de massa de D pelas fórmulas:
My (D)
Mx (D)
x=
¯
ey=
¯
.
m(D)
m(D)
Usando os resultados anteriores temos:

54
Cálculo III
ba2
b2 a
x= 6 ey= 6
¯
¯
ab
ab
2
2
Simplificando temos:
a
b
x= ey=
¯
¯
3
3

AULA

3

Como segundo exemplo usaremos uma região em que o sistema de
coordenadas polares facilita os cálculos.
Exemplo 3.2. Para o segundo exemplo desejamos determinar o
centro de massa de uma região D dada pelo quarto da coroa circular de raio interno a e raio externo b que situa-se no primeiro
quadrante (Fig 3.2), cuja densidade superficial de massa é constante (x, y) = .

Figura 3.2: Gráfico do exemplo 2
SOLUÇÃO:
Começaremos por determinar os limites de integração inspecionando a (Fig 3.2) e verificando que 0 ≤ ϑ ≤ π/2 e a ≤ r ≤ b.
Em segundo calcularemos a massa da região D, m(D) e os respectivos momentos de massa com relação ao eixo x e ao eixo y Mx (D)
e My (D) respectivamente.

55
Algumas Aplicações da Integral Dupla
Passo 1 determinar a massa m(D), dada pela integral dupla:
π/2

m(D) =

b

(x, y)dxdy =
D

rdrdϑ
0

a

Integrando em r temos:
π/2
r2 b
m(D) =
dϑ
2 a
0
Substituindo os limites de integração temos:
π/2
b2 a2
m(D) =
dϑ
−
2
2
0
Integrando em ϑ temos:
π/2
b2 a2
m(D) =
−
ϑ
2
2
0
Substituindo os limites de integração temos:
1
π(b2 − a2 )
m(D) =
4
Passo 2 calcular o momento de massa Mx (D) dado pela integral
dupla:
Mx (D) =

(x, y)ydxdy
D

Substituindo os limites em coordenadas polares e sabendo que
y = r sin(ϑ) temos:
π/2

Mx (D) =

b

(x, y)ydxdy =
D

r sin(ϑ)rdrdϑ
0

a

Integrando em r temos:
π/2

r3 b
dϑ
3 a
0
Substituindo os limites de integração temos:
π/2
b3 a3
−
dϑ
Mx (D) =
sin(ϑ)
3
3
0
Integrando em ϑ temos:
π/2
b3 a3
Mx (D) =
−
(− cos(ϑ))
3
3
0
Substituindo os limites de integração temos:
b3 a3
Mx (D) =
−
(− cos(π/2) − − cos(0))
3
3
Simplificando temos:
1 3
Mx (D) =
(b − a3 )
3
Passo 3 calcular o momento de massa My (D) dado pela integral
Mx (D) =

dupla:

56

sin(ϑ)
AULA

Cálculo III

My (D) =

3

(x, y)xdxdy
D

My (D) =

(x, y)xdxdy
D

Substituindo os limites em coordenadas polares e sabendo que
x = r cos(ϑ) temos:
π/2

b

r cos(ϑ)rdrdϑ

(x, y)ydxdy =

Mx (D) =

0

D

a

Integrando em r temos:
π/2

r3 b
dϑ
3 a
0
Substituindo os limites de integração temos:
π/2
b3 a3
Mx (D) =
cos(ϑ)
−
dϑ
3
3
0
Integrando em ϑ temos:
π/2
b3 a3
−
(sin(ϑ))
Mx (D) =
3
3
0
Substituindo os limites de integração temos:
b3 a3
−
(sin(π/2) − sin(0))
Mx (D) =
3
3
Simplificando temos:
1 3
Mx (D) =
(b − a3 )
3
Passo 4 Determinar o centro de massa de D pelas fórmulas:
My (D)
Mx (D)
x=
¯
ey=
¯
.
m(D)
m(D)
Usando os resultados anteriores temos:
1 3
(b − a3 )
3
x=y=
¯ ¯
1
π(b2 − a2 )
4
Levando em conta que b3 − a3 = (b − a)(b2 + ba + a2 ) e b2 − a2 =
Mx (D) =

cos(ϑ)

(b − a)(b + a) temos:
1
(b − a)(b2 + ba + a2 )
3
x=y=
¯ ¯
1
π(b − a)(b + a)
4
Simplificando temos:
4 b2 + ba + a2
x=y=
¯ ¯
.
3π
b+a

57
Algumas Aplicações da Integral Dupla

3.4

Conclusão

Na aula de hoje, vimos que dentre as inúmeras aplicações da
integral dupla, dentro da área da física destacamos, entre outras,
algumas das mais importantes que são: a determinação da massa
de uma região plana limitada por curvas, dada sua distribuição de
densidade, o cálculo do momento de massa de uma região plana
limitada por curvas, dada sua distribuição de densidade, o momento de inércia de uma região plana limitada por curvas, dada
sua distribuição de densidade e o cálculo do centro de massa de
uma região plana limitada por curvas, dada sua distribuição de
densidade.

RESUMO

Massa, Momento de Massa e Momento de Inércia
Dada uma região D ∈ R2 plana limitada com distribuição de densidade superficial (x, y) podemos calcular a massa de D, o momento
de massa em relação ao eixo x, o momento de massa relativo ao
eixo y, o momento de inércia em relação ao eixo x, o momento de
inércia relativo ao eixo y e momento de inércia relativo a origem,
denotados respectivamente m(D), Mx (D), My (D), Ix (D), Iy (D)
e I0 (D), pelas integrais duplas:
m(D) =

(x, y)dxdy
D

Mx (D) =

(x, y)ydxdy
D

My (D) =

(x, y)xdxdy
D

(x, y)y 2 dxdy

Ix (D) =
D

58
Cálculo III
(x, y)x2 dxdy e

Iy (D) =

AULA

3

D

(x, y)(x2 + y 2 )dxdy

I0 (D) =
D

Centro de Massa
Podemos também calcular o centro de massa, denotado (¯, y ) usando
x ¯
as seguintes fórmulas:
My (D)
x=
¯
=
m(d)

x (x, y)dxdy
D

(x, y)dxdy
D

y=
¯

Mx (D)
=
m(d)

y (x, y)dxdy
D

(x, y)dxdy
D

PRÓXIMA AULA

Em nossa próxima aula veremos as integrais triplas. Primeiramente definindo-as para funções de domínios retangulares através
do limite de somas de riemann estendendo a definição para funções
definidas em domínios não retangulares porém limitados.

ATIVIDADES

Deixamos como atividades dois problemas de determinação do
centro de massa.

ATIV. 3.1. Determine o centro de massa da região D dada pela
interseção das retas y = 0, x = 1 e y = ax2 (Fig 3.3) região em
cinza.
Comentário:

Volte ao texto e reveja com calma e atenção as

59
Algumas Aplicações da Integral Dupla

Figura 3.3: Atividade 1

Figura 3.4: Atividade 2

demonstrações acima, elas lhe servirão de guia. Use para este caso
coordenadas cartesianas.
ATIV. 3.2. Determine o centro de massa da região D dada pelo
semi-círculo superior x2 + y 2 = a2 (Fig 3.4) região em cinza.
Comentário:

Volte ao texto e reveja com calma e atenção as

demonstrações acima, elas lhe servirão de guia. Use para este caso
coordenadas polares.

LEITURA COMPLEMENTAR

ÁVILA, Geraldo, Cálculo 3: Funções de Várias Variáveis, Livros
Técnicos e Científicos Editora, São Paulo, 3a edição, 1982.
LEITHOLD, Louis, O Cálculo com Geometria Analítica. Volume
2, Editora Harbra, 1994.
STEWART, James,Cálculo. Volume 3, 5a edição, Editora CENGAGE Learning, 2009.
SWOKOWSKI, Earl E., Cálculo com Geometria Analítica, Volume
2, 2a edição, Makron Books do Brásil SP, 1994.

60
Cálculo III

AULA

THOMAS, George B., Cálculo, Volume 2, 10a, Addilson Wesley,

3

2003.
KAPLAN, Wilfred, Cálculo Avançado Vol.1 e vol.2 Editora Edgard
Blücher 1991.// SPIEGEL, Murray R. Cálculo Avançado, Editora
McGraw-Hill do Brasil, 1971.
BOUCHARA, Jacques, Cálculo Integral Avançado, EDUSP, 2006.

61
AULA

Integrais triplas
META:
Apresentar integrais triplas de funções de valores reais e domínio
em R3 .
OBJETIVOS:
Ao fim da aula os alunos deverão ser capazes de:
Definir integral tripla e calcular algumas integrais triplas de funções de valores reais e domínio em R3 .
PRÉ-REQUISITOS
Os conhecimentos de integrais de funções de valores reais com domínio em R, da disciplina Cálculo I.

4
Integrais triplas

4.1

Introdução

Caros alunos a quarta aula do nosso curso de Cálculo III com o
tema “Integrais Triplas”. Bem como a integral dupla, vista na nossa
primeira aula, a integração tripla, em essência, é uma extensão
natural da integral simples vista em Cálculo I e definida como
limite de somas de Riemann. Na prática, a integração tripla é dada
HISTÓRIA
A primeira técnica
sistemática documentada para o cálculo
de integrais triplas no
cálculo de volume foi
o método da exaustão
de
Eudoxus
cerca
de 370AC. O maior
avanço
no
cálculo
de integrais triplas
veio do Iraque, no
século 11, na figura
de Ibn AL-Haythan
(conhecido na Europa
por Alhazen ).
Enquanto resolvia o que
ficou conhecido como
“Problema de Alhazen”
(um problema de ótica)
ele calculou o volume
de um parabolsóide
usando um método de
indução. Wikipédia.

por três integrações simples, cada uma efetuada sobre uma variável
e considerando as demais como constantes. É o que denominamos
de integrais interadas. As características e detalhes próprios das
integrais triplas serão vistas ao longo do nosso curso, nas próximas
três aulas.

4.2

Integração Tripla: Domínios Paralelepípedais

Começamos por considerar uma função φ definida em um domínio paralelepipedal R = {(x, y, z) ∈ R3 |a ≤ x ≤ b ∧ c ≤ y ≤
d ∧ e ≤ z ≤ f }. Formalmente φ : [a, b] × [c, d] × [e, f ] → R.
Usando a imaginação, pensemos em R retalhada por uma rede
de planos paralelos aos planos coordenados e que dividem R em
pequenos paralelepípedos. Oficialmente, consideraremos três partições P [a, b] = {x0 = a, x1 , . . . , xi , xi+1 , . . . , xl = b}, P [c, d] =
{y0 = c, y1 , . . . , yj , yj+1 , . . . , ym = d} e P [e, f ] = {z0 = e, z1 ,
. . . , zk , zk+1 , . . . , zn = f } onde como visto em Cálculo I temos:
x0 < x1 < · · · < xi < xi+1 < · · · < xl , y0 < y1 < · · · < yj <
yj+1 < · · · < ym e z0 < z1 < · · · < zk < zk+1 < · · · < zn . Desta
forma cada um dos pequenos subintervalos Ii = [xi−1 , xi ], Jj =

64
Cálculo III

AULA

[yj−1 , yj ] e Kk = [zk−1 , zk ] têm comprimentos ∆xi = xi − xi−1 ,

4

∆yj = yj − yj−1 e ∆zk = zk − zk−1 , respectivamente. Definimos,
agora, a uma partição para o paralelepípedo R por P = P [R] =
P [a, b]×P [c, d]×P [e, f ], o produto cartesiano das partições P [a, b],
P [c, d] e P [e, f ]. Os planos retalham a região R em uma série de pequenos paralelepípedos Vijk = [xi−1 , xi ] × [yj−1 , yj ] × [zk−1 , zk ], 1 ≤
i ≤ l, 1 ≤ j ≤ m, 1 ≤ k ≤ n. O volume de cada pequeno paralelepípedo é dado por ∆Vijk = ∆xi ∆yj ∆zk . Como tanto ∆xi quanto
∆yj quanto ∆zk são diferentes de zero, o volume de cada pequeno
paralelepípedo é também diferente de zero. Podemos então definir
a norma da partição por: |P | = max (∆Vijk ), que corresponde
1≤i≤l
1≤j≤m
1≤k≤n

ao maior volume entre todos os pequenos paralelepípedos.
Pausa para respirar que já vamos definir a integral tripla sobre domínios paralelepípedais. Para isto tomemos um ponto (ξi , ζj , ηk ) ∈
[xi−1 , xi ] × [yj−1 , yj ] × [zk−1 , zk ] em cada pequeno paralelepípedo
e definimos a seguinte soma de Riemann:

l

m

n

Slmn =

φ(ξi , ζj , ηk )∆Vijk
i=1 j=1 k=1

A integral tripla da função φ(x, y, z) sobre o paralelepípedo R,
φ(x, y, z)dxdydz será então definida como o se-

denotada
R

guinte limite:

def

φ(x, y, z)dxdydz = lim Slmn
R

|P |→0

65
Integrais triplas

4.3

Integração Tripla: Domínios Não Paralelepípedais Limitados

Para definir a integral tripla de uma função φ : D ⊂ R3 →
R onde D é limitado não paralelepipedal, começamos por considerar uma função Φ definida em um domínio paralelepipedal
R = {(x, y, z) ∈ R3 |a ≤ x ≤ b ∧ c ≤ y ≤ d ∧ e ≤ z ≤ f } tal

 φ(x, y, z) , (x, y, z) ∈ D
. Formalque D ⊂ R e Φ(x, y, z) =
 0
, (x, y, z) ∈ D
/
mente Φ : [a, b] × [c, d] × [e, f ] → R é uma extensão da função
φ(x, y, z). Usando a imaginação, pensemos em R coberta por uma
rede de planos paralelos aos planos coordenados e que dividem
R em pequenos paralelepípedos e procedemos como na integral
tripla sobre domínios paralelepípedais, considerando a uma partição para o paralelepípedo R por P = P [R] = P [a, b] × P [c, d] ×
P [e, f ], o produto cartesiano das partições P [a, b], P [c, d] e P [e, f ]
onde P [a, b] = {x0 = a, x1 , . . . , xi , xi+1 , . . . , xl = b}, P [c, d] =
{y0 = c, y1 , . . . , yj , yj+1 , . . . , ym = d} e P [e, f ] = {z0 = e, z1 ,
. . . , zk , zk+1 , . . . , zn = f }. Do mesmo modo definimos a norma
da partição por: |P | = max (∆Vijk ) onde ∆Vijk = ∆xi ∆yj ∆zk ,
1≤i≤l
1≤j≤m
1≤k≤n

∆xi = xi − xi−1 , ∆yj = yj − yj−1 e ∆zk = zk − zk−1 . Tomamos
um ponto (ξi , ζj , ηk ) ∈ [xi−1 , xi ] × [yj−1 , yj ] × [zk−1 , zk ] em cada
pequeno paralelepípedo e definimos a seguinte soma de Riemann
para a função estendida Φ(x, y, z):

l

m

n

Slmn =

Φ(ξi , ζj , ηk )∆Vijk
i=1 j=1 k=1

66
Cálculo III

AULA

A integral tripla da função φ(x, y, z) sobre o domínio D ⊂ R3 ,

4

φ(x, y, z)dxdydz será então definida como o se-

denotada
D

guinte limite:
def

φ(x, y, z)dxdydz = lim Slmn .
|P |→0

D

Observem que, semelhante ao caso das integrais duplas, apenas os
pequenos paralelepípedos cujo ponto escolhido pertence ao domínio
D ⊂ R3 , contribuem para a soma de Riemann os demais têm
contribuição nula visto que o ponto escolhido dentro destes estão
fora de D ⊂ R2 e portanto Φ(ξi , ζj , ηk ) = 0.

4.4

Interpretação Geométrica

Quando a função φ : D ⊂ R3 → R é constante e igual a um
(φ(x, y, z) = 1, ∀(x, y, z) ∈ D) e a região domínio D é limitada,
vemos que a soma de Riemann aproxima o volume da região D e
quanto maior for o refinamento da partição de R3 ⊃ R ⊃ D melhor
será a aproximação. Podemos então, interpretar a integral tripla
dxdydz como o volume da região D ⊂ R3 .
D

4.5

Integrais Iteradas

Dada uma função φ : R → R onde R = [a, b] × [c, d] × [e, f ], do
mesmo modo que na integral dupla, valem as integrais interadas:

b

d

f

φ(x, y, z)dxdydz =

1.
R

φ(x, y, z)dz dy dx
a

c
b

e
f

d

φ(x, y, z)dxdydz =

2.
R

φ(x, y, z)dy dz dx
a

e

c

67
Integrais triplas
d

b

f

φ(x, y, z)dxdydz =

3.
R

φ(x, y, z)dz dx dy
c

a
d

e
f

b

φ(x, y, z)dxdydz =

4.
R

φ(x, y, z)dx dz dy
c

e
f

a
d

b

φ(x, y, z)dx dy dz

φ(x, y, z)dxdydz =

5.

e

R

c
f

a
b

d

φ(x, y, z)dy dx dz

φ(x, y, z)dxdydz =

6.
R

e

a

c

Em outras palavras, quando o domínio da integral tripla é paralelepipedal a ordem de integração não importa.

4.6

Propriedades das Integrais Triplas

Como nosso curso é de Cálculo, apenas listaremos, sem demonstração, alguma das propriedades das integrais triplas. Caso
desejem conhecer a demonstração de algumas destas propriedades,
recomendo livros de Cálculo Avançado como os citados na bibliografia abaixo.
Propriedade 4.1. Sejam f : D ⊂ R3 → R uma função de valores
reais integrável em D e c ∈ R, então vale:

cf (x, y, z)dxdydz = c
D

f (x, y, z)dxdydz
D

Propriedade 4.2. Sejam f, g : D ⊂ R3 → R duas funções de
valores reais integráveis em D, então vale:

(f + g)(x, y, z)dxdydz =
D

f (x, y, z)dxdydz
D

+

g(x, y, z)dxdydz
D

68
Cálculo III

AULA

Propriedade 4.3. Sejam f : D ⊂ R3 → R uma função de valores

4

reais integrável em D tal que f (x, y, z) ≥ 0, ∀(x, y, z) ∈ D, então
vale:

f (x, y, z)dxdydz ≥ 0
D

Propriedade 4.4. Sejam f, g : D ⊂ R3 → R duas funções de valores reais integráveis em D tais que f (x, y, z) ≥ g(x, y, z), ∀(x, y, z) ∈
D, então vale:

f (x, y, z)dxdydz ≥

g(x, y, z)dxdydz
D

D

Propriedade 4.5. Seja f : D ⊂ R3 → R uma função de valores
reais integrável em D onde D = A ∪ B e A ∩ B é a união de um
número finito de superfícies em R3 , então vale:

f (x, y, z)dxdydz =
D

f (x, y, z)dxdydz
A

+

f (x, y, z)dxdydz
B

OBS 4.1. As duas primeiras propriedades diz respeito à “linearidade” do operador integral tripla. As terceira e quarta propriedades são denominadas “dominação” enquanto que a quinta
propriedade é denominada “aditividade”.

4.7

Exemplos

Nada mais natural que ilustrar um novo conceito com exemplos e, vamos aqui fazer exatamente isto. Ilustrar o conceito de
integral tripla com dois exemplos. Antes porém, vale observar

69
Integrais triplas
que a na prática uma integral tripla equivale a três integrais simples e neste caso uma pergunta fica no ar. Qual das duas variáveis x, y ou z integraremos primeiro? Muito bem, a resposta é
dada pela propria expressão da integral tripla. Isto é, na integral
f (x, y, z)dxdydz primeiramente integramos na variável x,
R

depois na variável y e por último na variável z. Já na integral
f (x, y, z)dzdydx primeiramente integramos na variável z,
R

depois na variável y e por último na variável x.
Exemplo 4.1. Considere a função f : [0, 1] × [0, 1] × [0, 1] → R
dada por f (x, y) = x2 + y 2 + z 2 e determine a integral tripla
f (x, y, z)dxdydz sobre a região R = {(x, y, z) ∈ R3 |0 ≤

I=
R

x ≤ 1 ∧ 0 ≤ y ≤ 1 ∧ 0 ≤ z ≤ 1}.

SOLUÇÃO:
Passo 1 colocaremos os limites de integração que representam a
região R dada, segundo a ordem de integração:
1

1

1

(x2 + y 2 + z 2 )dxdydz

I=
0

0

0

Passo 2 integraremos na variável x considerando as variáveis y e
z como constantes:
1
1
x3
I=
+ y 2 x + z 2 x dydz
3
0
0
Substituindo os limites de integração temos:
1
1
13 03
I=
−
+ y 2 (1 − 0) + z 2 (1 − 0) dydz
3
3
0
0
Efetuando os cálculos temos:
1
1
1
I=
+ y 2 + z 2 dy
3
0
0
Passo 3 integraremos na variável y considerando a variável x
como constante:

70
Cálculo III
1

1
1
y3
y+
+ z2y
dz
3
3
0
0
Substituindo os limites de integração temos:
1
1
13 03
I=
(1 − 0) +
−
+ z 2 (1 − 0) dz
3
3
3
0
Efetuando os cálculos temos:
1
1 1
+ + z 2 dz
I=
3 3
0
Passo 4 último passo, integraremos na variável z:
1
1
z3 1
I=
z+ z+
3
3
3
0
Substituindo os limites de integração temos:
1
1
13 03
I=
(1 − 0) + (1 − 0) +
−
3
3
3
3
Efetuando os cálculos temos:
1 1 1
I= + + =1
3 3 3

I=

AULA

4

Figura 4.1: Determinação prática dos limites para D
OBS 4.2. Daremos aqui um método prático para determinar os
limites de integração em uma integral tripla sobre domínio não retangular da forma: D.

Passo 1 Fazer um desenho da região D. (Fig. 4.1) identificando
as superfícies inferior a(x, y) e superior b(x, y) que limitam a região

71
Integrais triplas
D, bem como a sombra projetada no plano xy por D, denotada
D∗ e identificar as curvas limites da região D∗ a(x) curva inferior
e b(x) curva superior, como na AULA01.
Passo 2 Atravessar toda a região D∗ e o eixo x com um segmento
de reta paralelo e orientado na direção positiva ao eixo y (segmento
r na Fig. 4.1)
Passo 3 Deslocar o segmento de reta r paralelo ao eixo y na direção negativa do eixo x até tocar o ponto mais à esquerda de D∗
marcando o limite inferior de x (ponto a na Fig. 4.1).
Passo 4 Deslocar o segmento de reta r paralelo ao eixo y na direção positiva do eixo x até tocar o ponto mais à direita de D∗
marcando o limite superior de x (ponto b na Fig. 4.1).
Passo 5 Tomando um ponto qualquer x ∈ (a, b) passamos o segmento de reta r através da região D∗ paralelo ao eixo y na direção
positiva do eixo x. O limite inferior para a variável y será a função a(x), ponto da curva onde o segmento entra na região D∗ e o
limite superior para a variável y será b(x), ponto da curva onde o
segmento de reta sai da região D∗ .
Passo 6 Tomando um ponto qualquer (x, y) ∈ D∗ passamos o
segmento de reta s através da região D, paralelo ao eixo z orientado na direção positiva de z. O limite inferior para a variável z
será a função a(x, y), ponto da superfície onde o segmento entra
na região D e o limite superior para a variável z será b(x, y), ponto
da superfície onde o segmento de reta sai da região D.
Nossa integral será efetuada assim:
b

b(x)

b(x,y)

f (x, y)dxdy =
D

72

f (x, y, z)dzdydx
a

a(x)

a(x,y)
AULA

Cálculo III

4
Vamos diretamente para um segundo exemplo de integral dupla
sobre domínios não retangulares. A saber:
Exemplo 4.2. Considere a função f : D ⊂ R3 → R dada por
f (x, y, z)dxdydz

f (x, y) = xyz e determine a integral dupla
D

sobre a região D = {(x, y, z) ∈ R3 |0 ≤ x ≤ 1 ∧ 0 ≤ y ≤ x2 ∧ 0 ≤
z ≤ 1}, (Fig. 4.2).

Figura 4.2: Domínio D para o exemplo 2
SOLUÇÃO:
Passo 1 faremos o desenho das superfícies que determinam os
limites para a região D. A saber x = 0, x = 1, y = x2 , x = 0 e
z = 1 (Fig. 4.2).
Usando o processo prático exposto acima determinamos os limites de integração. A saber: a = 0, b = 1, a(x) = 0, b(x) = x2 ,
a(x, y) = 0 e b(x, y) = 1.
x2

1

1

I=

xyzdzdydx
0

0

0

Passo 2 integraremos na variável z considerando a variável y

73
Integrais triplas
e x como uma constante:
1
x2
z2 1
dydx
I=
xy
2 0
0
0
Substituindo os limites de integração temos:
1
x2
12
02
xy − xy ) dydx
I=
2
2
0
0
Efetuando os cálculos temos:
2
1 1 x
xydydx
I=
2 0 0
Passo 3 integraremos na variável y considerando a variável x
constante temos:
1
y 2 x2
I=
x
dx
2 0
0
Substituindo os limites de integração temos:
1 1
(x2 )2
02
I=
x
−x
dx
2 0
2
2
Efetuando os cálculos temos:
1 1 5
x dx
I=
4 0
Integrando , finalmente , na variável x temos:
1 x6 1
I=
4 6 0
Substituindo os limites de integração temos:
1 16 06
−
I=
4 6
6
1
Efetuando os cálculos temos: I =
24

4.8

Conclusão

Na aula de hoje, vimos que a integral tripla é uma extensão
natural do conceito de integral simples visto em Cálculo I e também
uma extensão natural do conceito de integral dupla, vista em nossa
primeira aula do curso de Cálculo III. E se por um lado a integral
simples pode ser interpretada como a área sob a curva descrita por

74
Cálculo III

AULA

função positiva f (x) em um domínio [a, b] e a integral dupla pode

4

ser vista como o volume de um prisma reto limitado superiormente
pela a superfície descrita por uma função positiva f (x, y) e limitado
inferiormente pelo domínio [a, b] × [c, d], a integral tripla só tem
interpretação geométrica no caso simples em que f (x, y, z) = 1.
Neste caso a integral tripla representa o volume da região limitada
D ⊂ R3 .

RESUMO

Integração Tripla: Domínios Paralelepipedais
Considerando uma função φ definida em um domínio paralelepipedal R = {(x, y, z) ∈ R3 |a ≤ x ≤ b ∧ c ≤ y ≤ d ∧ e ≤ z ≤ f }.
Podemos dividir R em pequenos paralelepípedos considerando os
planos paralelos ao planos cartesianos gerados pela partição P =
P [R] = P [a, b]×P [c, d]×P [e, f ], o produto cartesiano das partições
P [a, b], P [c, d] e P [e, f ] onde P [a, b] = {x0 = a, x1 , . . . , xi , xi+1 , . . . ,
xl = b}, P [c, d] = {y0 = c, y1 , . . . , yj , yj+1 , . . . , ym = d} e P [e, f ] =
{z0 = e, z1 , . . . , zk , zk+1 , . . . , zn = f }. Os planos retalham a região
R em uma série de pequenos paralelepípedos Vijk = [xi−1 , xi ] ×
[yj−1 , yj ] × [zk−1 , zk ], 1 ≤ i ≤ l, 1 ≤ j ≤ m, 1 ≤ k ≤ n. O volume
de cada pequeno paralelepípedo é dado por ∆Vijk = ∆xi ∆yj ∆zk .
A norma da partição fica estabelecida como: |P | = max (∆Vijk ).
1≤i≤l
1≤j≤m
1≤k≤n

Toma-se um ponto (ξi , ζj , ηk ) ∈ [xi−1 , xi ] × [yj−1 , yj ] × [zk−1 , zk ]
em cada pequeno paralelepípedo e definimos a seguinte soma de
Riemann:

75
Integrais triplas

l

m

n

Slmn =

φ(ξi , ζj , ηk )∆Vijk
i=1 j=1 k=1

A integral tripla da função φ(x, y, z) sobre o paralelepípedo R,
φ(x, y, z)dxdydz será então definida como o se-

denotada
R

guinte limite:
def

φ(x, y, z)dxdydz = lim Slmn
|P |→0

R

Integração Tripla: Domínios Não Paralelepípedais Limitados
Para definir a integral tripla de uma função φ : D ⊂ R3 → R
onde D é não paralelepipedal limitado, começamos por considerar uma função Φ definida em um domínio paralelepipedal R =
{(x, y, z) ∈ R3 |a ≤ x b ∧ c ≤ y ≤ d ∧ e ≤ z ≤ f } tal que
≤
 φ(x, y, z) , (x, y, z) ∈ D
D ⊂ R e Φ(x, y, z) =
. Formalmente
 0
, (x, y, z) ∈ D
/
Φ : [a, b] × [c, d] × [e, f ] → R é uma extensão da função φ(x, y, z).
A partir daqui todo o procedimento é semelhante ao da definição
da integral tripla em domínios paralelepipedais. Podemos definir
a integral tripla de uma função φ(x, y, z) em um domínio não retangular D por:
def

φ(x, y, z)dxdydz = lim Slmn
D

onde Slmn =

l
i=1

m
j=1

|P |→0

n
k=1 Φ(ξi , ζj , ηk )∆Vijk

é a soma de Rie-

mann para Φ(x, y, z.
Integrais Iteradas
As integrais iteradas dizem que em um domínio retangular R =
[a, b] × [c, d] × [e, f ] a ordem de execução das integrais simples não

76
Cálculo III

AULA

alteram o valor da integral tripla, que pode ser representada por:

4

b

d

f

φ(x, y, z)dz dy dx

φ(x, y, z)dxdydz =

1.

a

R

c
b

e
f

d

φ(x, y, z)dy dz dx

φ(x, y, z)dxdydz =

2.

a

R

e
d

c
b

f

φ(x, y, z)dxdydz =

3.
R

φ(x, y, z)dz dx dy
c

a
d

e
f

b

φ(x, y, z)dxdydz =

4.
R

φ(x, y, z)dx dz dy
c

e
f

a
d

b

φ(x, y, z)dxdydz =

5.
R

φ(x, y, z)dx dy dz
e

c
f

a
b

d

φ(x, y, z)dxdydz =

6.
R

φ(x, y, z)dy dx dz
e

a

c

Propriedades das Integrais triplas
As integrais triplas são, de certo modo, semelhantes às propriedades das integrais simples que vimos em Cálculo I sendo quase
que uma extensão natural destas. As integrais triplas têm, entre
outras, as seguintes propriedades:
Propriedade 1 Sejam f : D ⊂ R3 → R uma função de valores
reais integrável em D e c ∈ R, então vale:

cf (x, y, z)dxdydz = c
D

f (x, y, z)dxdydz
D

Propriedade 2 Sejam f, g : D ⊂ R3 → R duas funções de valores
reais integráveis em D, então vale:

(f + g)(x, y, z)dxdydz =
D

f (x, y, z)dxdydz
D

+

g(x, y, z)dxdydz
D

77
Integrais triplas
Propriedade 3 Sejam f : D ⊂ R3 → R uma função de valores
reais integrável em D tal que f (x, y, z) ≥ 0, ∀(x, y, z) ∈ D, então
vale:

f (x, y, z)dxdydz ≥ 0
D

Propriedade 4 Sejam f, g : D ⊂ R3 → R duas funções de valores
reais integráveis em D tais que f (x, y, z) ≥ g(x, y, z), ∀(x, y, z) ∈
D, então vale:

f (x, y, z)dxdydz ≥
D

g(x, y, z)dxdydz
D

Propriedade 5 Seja f : D ⊂ R3 → R uma função de valores
reais integrável em D onde D = A ∪ B e A ∩ B é a união de um
número finito de superfícies em R3 , então vale:

f (x, y, z)dxdydz =
D

f (x, y, z)dxdydz
A

+

f (x, y, z)dxdydz
B

Determinação dos Limites de Integração para Integrais
Triplas
Daremos aqui um método prático para determinar os limites de
integração em uma integral tripla sobre domínio não retangular da
forma: D.

Passo 1 Fazer um desenho da região D. (Fig. 4.1) identificando
as superfícies inferior a(x, y) e superior b(x, y) que limitam a região
D, bem como a sombra projetada no plano xy por D, denotada

78
Cálculo III

AULA

D∗ e identificar as curvas limites da região D∗ a(x) curva inferior

4

e b(x) curva superior, como na AULA01.

Passo 2 Atravessar toda a região D∗ e o eixo x com um segmento de reta paralelo e orientado na direção positiva ao eixo y
(segmento r na Fig. 4.1)

Passo 3 Deslocar o segmento de reta r paralelo ao eixo y na
direção negativa do eixo x até tocar o ponto mais à esquerda de
D∗ marcando o limite inferior de x (ponto a na Fig. 4.1).

Passo 4 Deslocar o segmento de reta r paralelo ao eixo y na
direção positiva do eixo x até tocar o ponto mais à direita de D∗
marcando o limite superior de x (ponto b na Fig. 4.1).

Passo 5 Tomando um ponto qualquer x ∈ (a, b) passamos o
segmento de reta r através da região D∗ paralelo ao eixo y na direção positiva do eixo x. O limite inferior para a variável y será a
função a(x), ponto da curva onde o segmento entra na região D∗ e
o limite superior para a variável y será b(x), ponto da curva onde
o segmento de reta sai da região D∗ .

Passo 6 Tomando um ponto qualquer (x, y) ∈ D∗ passamos o
segmento de reta s através da região D, paralelo ao eixo z orientado na direção positiva de z. O limite inferior para a variável z
será a função a(x, y), ponto da superfície onde o segmento entra
na região D e o limite superior para a variável z será b(x, y), ponto
da superfície onde o segmento de reta sai da região D.

79
Integrais triplas
Nossa integral será efetuada assim:
b

b(x)

b(x,y)

f (x, y)dxdy =
D

f (x, y, z)dzdydx
a

a(x)

a(x,y)

PRÓXIMA AULA

Em nossa próxima aula veremos mudança de variáveis na integração tripla. O objetivo da mudança de variáveis em uma integral tripla será a de facilitar esta integração de uma de duas
formas. A primeira será tornando o integrando mais simples. A
segunda transformando o domínio D do integrando em um domínio
de forma geométrica mais simples.

ATIVIDADES

Deixamos como atividades o cálculo de algumas integrais tríplas.

ATIV. 4.1. Seja f : [−1, +1] × [−1, +1] × [−1, +1] → R dada por
f (x, y, z) = x2 + y 2 + z 2 . Determine a integral tripla:
f (x, y, z)dxdydz.
R

Comentário:

Volte ao texto e reveja com calma e atenção o

cálculo de integrais duplas dos exemplos acima, elas lhe servirão
de guia.
ATIV. 4.2. Seja f : D ⊂ R3 → R dada por f (x, y, z) = 1, onde
D = {(x, y, z) ∈ R3 |x ≥ 0 ∧ 0 ≤ y ≤ 1 − x2 ∧ 0 ≤ z ≤ 1 − x2 }.

80
Cálculo III

AULA

4

• Esboce a região de integração
• Determine os limites da integral tripla:
f (x, y, z)dxdydz
D

• Calcule a integral tripla

f (x, y, z)dxdydz.
D

Comentário:

Volte ao texto e reveja com calma e atenção o

cálculo de integrais duplas dos exemplos acima, elas lhe servirão
de guia.

LEITURA COMPLEMENTAR

ÁVILA, Geraldo, Cálculo 3: Funções de Várias Variáveis, Livros
Técnicos e Científicos Editora, São Paulo, 3a edição, 1982.
LEITHOLD, Louis, O Cálculo com Geometria Analítica. Volume
2, Editora Harbra, 1994.
STEWART, James,Cálculo. Volume 3, 5a edição, Editora CENGAGE Learning, 2009.
SWOKOWSKI, Earl E., Cálculo com Geometria Analítica, Volume
2, 2a edição, Makron Books do Brásil SP, 1994.
THOMAS, George B., Cálculo, Volume 2, 10a, Addilson Wesley,
2003.
KAPLAN, Wilfred, Cálculo Avançado Vol.1 e vol.2 Editora Edgard
Blücher 1991.// SPIEGEL, Murray R. Cálculo Avançado, Editora
McGraw-Hill do Brasil, 1971.
BOUCHARA, Jacques, Cálculo Integral Avançado, EDUSP, 2006.

81
AULA

Mudança de Variáveis
em Integrais tríplas
META:
Introduzir mudança de variáveis em integrais triplas de funções de
valores reais e domínio em R3 .
OBJETIVOS:
Ao fim da aula os alunos deverão ser capazes de:
Calcular integrais triplas de funções de valores reais e domínio em
R3 utilizando mudança de variáveis.
PRÉ-REQUISITOS
Os conhecimentos de integrais de funções de valores reais com domínio em R, da disciplina Cálculo I, superfícies em R3 , de coordenadas polares da disciplina Cálculo II e integrais triplas aula 04.

5
Mudança de Variáveis em Integrais tríplas

5.1

Introdução

Caros alunos o problema da mudança de variáveis em integrais
triplas é inteiramente análogo ao problema de mudança de variáveis em integrais duplas. Analogias a parte, o fato de do espaço R3
ter uma dimensão a mais que o R2 , traz um esforço algébrico adicional ao tratamento geral da mudança de variáveis em integrais
HISTÓRIA
O teorema de mudança de variáveis em
integrais tríplas foi
primeiro proposto por
Lagrange em 1773 e
usado por Legendre,
Laplace e Gauss, e
primeiramente
generalizado
para
n
variáveis por Mikhail
Ostrogradski em 1836,
resistiu a uma demonstração mais rigorosa
por longo tempo (cerca
de 125 anos).
E
foi
satisfatóriamente
demonstrado por Elie
Cartan em uma série
de artigos nos anos
1890.

triplas. Veremos dois casos particulares de mudança de variáveis
em integrais tripla que correspondem aos: sistemas de coordenadas
cilíndricos e sistema de coordenadas esféricos.

5.2

Mudança de Variáveis em Integrais Triplas

Vamos considerar a integração de uma função f : D ⊂ R3 →
R onde (x, y, z) ∈ D e conseqüentemente, ∀(x, y, z) ∈ D temos
f (x, y, z) ∈ R. Consideraremos também, uma transformação T :
D ⊂ R3 → D ⊂ R3 , biunívoca de modo que D = T −1 (D ),
∀(u, v, w) ∈ D , (x, y, z) = T −1 (u, v, w) ∈ D. Trocando em miúdos: x = x(u, v, w), y = y (u, v, w) e z = z (u, v, w). E suponhamos
ˆ
ˆ
ˆ
as funções contínuas e deriváveis e seu
∂(x, y, z)
x, y, z
ou
:
finido por: J
u, v, w
∂(u, v, w)

∂x
ˆ
 ∂u
 ∂x
∂(x, y, z)
x, y, z
ˆ
J
=
= det 
 ∂v
u, v, w
∂(u, v, w)
 ∂x
ˆ
∂w

jacobiano, denotado J, de-

∂y
ˆ
∂u
∂y
ˆ
∂v
∂y
ˆ
∂w

∂z
ˆ
∂u
∂z
ˆ
∂v
∂z
ˆ
∂w




.



Suponhamos uma partição de D feita partindo de planos paralelos aos planos coordenados vw (u constante), uw (v constante) e
uv (w constante). Denotando ui+1 = ui + ∆ui , vj+1 = vj + ∆vj e

84
Cálculo III

AULA

wk+1 = wk + ∆wk , destacamos o pequeno paralelepípedo indexado

5

por ijk, (Fig 5.1). Suponhamos que

Figura 5.1: Elemento de vo-

Figura 5.2: Elemento de vo-

lume em D

lume em D

este pequeno paralelepípedo de volume ∆Vijk = ∆ui ∆vj ∆wk seja
mapeado por T −1 em um subdomínio em D de volume ∆Vijk (Fig
5.2). Seja: P = P (u, v, w) = (ˆ(u, v, w), y (u, v, w), z (u, v, w)).
x
ˆ
ˆ
Os segmentos de reta (u, vj , wk ), (ui , v, wk ) e (ui , vj , w) começando no ponto (ui , vj , wk ) são mapeados por T −1 em P (u, vj , wk )
P (ui , v, wk ) P (ui , vj , w) ver (Fig 5.2).
∂P
No subdomínio Vijk ⊂ D traçamos os vetores tangentes
∆ui ,
∂u
∂P
∂P
∆vj e
∆wk , ver (Fig 5.3). Em seguida traçamos segmentos
∂v
∂w
de reta paralelos aos vetores tangentes completando um paralelepípedo em D, ver (Fig 5.4), cujo volume admitiremos aproximadamente igual ao ∆Vijk (esta é a argumentação heurística). Este
volume é dado por:

∆Vijk ≈

∂P
∂P
∂P
∆ui ×
∆vj •
∆wk .
∂u
∂v
∂w

85
Mudança de Variáveis em Integrais tríplas
Levando em conta que:
∂P
∂x
ˆ
∂y
ˆ
∂z
ˆ
=
i+
j+
k
∂u
∂u
∂u
∂u
∂P
∂x
ˆ
∂y
ˆ
∂z
ˆ
=
i+
j+
k
∂v
∂v
∂v
∂v
∂P
∂x
ˆ
∂y
ˆ
∂z
ˆ
=
i+
j+
k
∂w
∂w
∂w
∂w
e calculando o produto vetorial mixto teremos:

Figura 5.3: Elemento de vo-

Figura 5.4: Elemento de vo-

lume em D

lume em D




∂P
∂P
∂P
∆ui ×
∆vj •
∆wk = det 

∂u
∂v
∂w


∂x
ˆ
∂u
∂x
ˆ
∂v
∂x
ˆ
∂w

∂y
ˆ
∂u
∂y
ˆ
∂v
∂y
ˆ
∂w

∂z
ˆ
∂u
∂z
ˆ
∂v
∂z
ˆ
∂w




 ∆ui ∆vj ∆wk



Daí, levando em conta a expressão do jacobiano em R3 , dada
acima, temos:

∆Vijk ≈

∂(ˆ, y , z )
x ˆ ˆ
∆ui ∆vj ∆wk
∂(u, v, w)

O que nos leva à seguinte expressão para a mudança de variáveis
em integrais triplas:
F (u, v, w) |J| dudvdw

f (x, y, z)dxdydz =
D

86

D
Cálculo III
onde: F (u, v, w) = f (ˆ(u, v, w), y (u, v, w), z (u, v, w)) e J é o jacox
ˆ
ˆ
x, y , z
ˆ ˆ ˆ
.
biano J = J
u, v, w

5.3

AULA

5

Alguns Exemplos

Nesta seção veremos dois exemplos de integrais triplas com
mudança de variáveis. No primeiro aplicaremos a mudança de variáveis dada pelo sistema de coordenadas cilíndricas e no segundo
o sistema de coordenadas esféricas

Primeiramente veremos um exemplo em coordenadas cilíndricas.
Antes porém, veremos como determinar os limites de integração
em coordenadas cilíndricas.

Figura 5.5: Coordenadas ci-

Figura 5.6: Coordenadas ci-

líndricas 1

líndricas 2

Passo 1 Esboçar o domínio D bem como sua projeção D∗ no
plano xy (ver Fig. 5.5).
Passo 2 Identificar as curvas que limitam a região D∗ . Atravessar
a região D∗ com uma reta r começando na origem (ver Fig. 5.6).

87
Mudança de Variáveis em Integrais tríplas
À medida em que a reta r percorre a região D∗ o ângulo ϑ que ela
forma com o eixo x positivo varia do mínimo α que será o limite
inferior da variável ϑ ao máximo β que será o limite superior da
variável ϑ. Daí, a variável ϑ ∈ [α, β].
Passo 3 Para cada valor fixo da variável ϑ ∈ [α, β] atravessar a
região D∗ com a reta r (ver Fig. 5.7). O ponto onde a reta r
entra na região D∗ é o limite inferior α(ϑ) para a variável r e o
ponto onde a reta r sai da região D∗ é o limite inferior β(ϑ) para
a variável r. Daí, r ∈ [α(ϑ), β(ϑ)].
Passo 4 Para cada valor fixo da variável ϑ ∈ [α, β] e da variável
r ∈ [α(ϑ), β(ϑ)] tomar o ponto (r, ϑ) ∈ D∗ em coordenadas polares e levantar a reta s atravessando a região D (ver Fig. 5.8). O
ponto onde a reta s entra na região D é o limite inferior α(r, ϑ)
para a variável z e o ponto onde a reta s sai da região D é o limite
superior β(r, ϑ) para a variável z. Daí, z ∈ [α(r, ϑ), β(r, ϑ)].

Figura 5.7: Coordenadas ci-

Figura 5.8: Coordenadas ci-

líndricas 3

líndricas 4

Podemos agora encarar o nosso primeiro exemplo onde colocaremos

88
Cálculo III

AULA

em prática a determinação dos limites de integração em coordena-

5

das cilíndricas.

Exemplo 5.1. Considere o sólido gerado pela intersecção das superfícies: z = y + a, (plano) x2 + y 2 − 2ay = 0, (cilindro) e z = 0,
(plano) (Fig 5.9) e determine seu volume.

Figura 5.9:
exemplo 1

Superfícies do

Figura 5.10: Interseção das
superfícies do exemplo 1

SOLUÇÃO: Para uma melhor compreensão mostramos na (Fig
5.10) o sólido gerado pela interseção das superfícies dadas e na
(Fig 5.11) as superfícies que compõem o sólido separadas no espaço.

Usaremos para o caso o sistema de coordenadas cilíndricas, dada
pela transformação: (x, y, z) → (r, ϑ, z) onde x = r cos(ϑ), y =
r sin(ϑ) e z = z. O jacobiano da transformação é dado por:

89
Mudança de Variáveis em Integrais tríplas

Figura 5.11: Domínio D para o exemplo 2

∂x
ˆ
 ∂r
 ∂x
x, y, z
ˆ
= det 
J =J
 ∂ϑ
r, ϑ, z
 ∂x
ˆ
∂z
Efetuando as derivadas parciais temos:




cos(ϑ)

∂y
ˆ
∂r
∂y
ˆ
∂ϑ
∂y
ˆ
∂z

sin(ϑ)

∂z
ˆ
∂r
∂z
ˆ
∂ϑ
∂z
ˆ
∂z

0














J = det  −r sin(ϑ) r cos(ϑ) 0 


0
0
1
Fazendo as contas do determinante temos:

J =r
Aproveitaremos o exemplo para aplicar os passos, vistos acima,
para determinação dos limites de integração de uma integral tripla
no sistema de coordenadas cilíndricas.
Passo 1:

Esboçar a interseção das superfícies (sólido D), bem

como sua projeção sobre o plano xy (superfície D∗ ), (ver Fig 5.10).
A projeção sobre o plano xy (superfície D∗ ), conhecide com a superfície inferior do sólido, sendo o disco dado por x2 +y 2 −2ay ≤ 0.

90
Cálculo III

AULA

Passo 2: Os limites para r e ϑ são determinados em D∗ do mesmo

5

modo que para coordenadas polares em R2 . Neste caso 0 ≤ ϑ ≤ 2π
e para r temos: que r vai de zero até a borda de D∗ que é dada
por x2 + y 2 − 2ay = (r cos(ϑ))2 + (r sin(ϑ))2 − 2ar sin(ϑ) = 0.
Daí, r2 − 2ar sin(ϑ) = r(r − 2a sin(ϑ)) = 0 Simplificando temos:
0 ≤ r ≤ 2a sin(ϑ).
Passo 3:

Para determinar os limites para z. Por cada par

(r, ϑ) ∈ D∗ , traçamos uma reta paralela ao eixo z orientada no
sentido positivo do eixo z atravessando o sólido. O limite inferior
de z é o ponto onde a reta entra no sólido e o limite superior o
ponto onde a reta sai do sólido. Neste caso: 0 ≤ z ≤ a + x ou
como x = r cos(ϑ) temos: 0 ≤ z ≤ a + r cos(ϑ).
Daí, o cálculo do volume de D será dado pela integral:
2π

2a sin(ϑ)

a+r sin(ϑ)

V ol(D) =

rdzdrdϑ
0

0

0

Integrando na variável z temos:
2π

2a sin(ϑ)

V ol(D) =

a+r sin(ϑ)

rz
0

0

drdϑ

0

Substituindo os limites de integração temos:
2π

2a sin(ϑ)

r(a + r sin(ϑ) − 0)drdϑ

V ol(D) =
0

0

Fazendo as contas temos:
2π

2a sin(ϑ)

(ar + r2 sin(ϑ))drdϑ

V ol(D) =
0

0

Integrando em na variável r temos:
2π

V ol(D) =

(a
0

r2 r3
+
sin(ϑ))
2
3

2a sin(ϑ)
0

dϑ

91
Mudança de Variáveis em Integrais tríplas
Substituindo o limite superior pois, o limite inferior por ser r = 0
não contribui, temos:
2π

V ol(D) =

(a
0

(2a sin(ϑ))2 (2a sin(ϑ))3
+
sin(ϑ))dϑ
2
3

Simplificando temos:
2π

(2a3 sin2 (ϑ) +

V ol(D) =
0

8a3 sin4 (ϑ)
)dϑ
3

Reescrevendo temos:
2π

sin2 (ϑ)dϑ +

V ol(D) = 2a3
0

8a3
3

2π

sin4 (ϑ)dϑ
0

Das tabelas de integrais temos:
α sinn−1 (αu) cos(αu)
an
n−1
+
sinn−2 (αu)du
n
u sin(2αu)
−
sin2 (αu)du =
2
4α

sinn (αu)du = −

Dai, temos:
ϑ sin(2ϑ)
−
2
4
sin3 (ϑ) cos(ϑ) 3
sin4 (ϑ)dϑ = −
+
4
4
3
sin (ϑ) cos(ϑ) 3
= −
+
4
4
sin2 (ϑ)dϑ =

sin2 (ϑ)dϑ
ϑ sin(2ϑ)
−
2
4

Podemos agora calcular as integrais. Para a integral de sin2 (ϑ)
temos:
2π

ϑ sin(2ϑ) 2π
−
2
4
0
2π sin(4π)
= +
−
2
4
0 sin(0)
− +
2
4
= π

sin2 (ϑ)dϑ =
0

92
AULA

Cálculo III

5

Para a integral de sin4 (ϑ) temos:
2π

2π
sin3 (ϑ) cos(ϑ) 3 ϑ sin(2ϑ)
+
−
4
4 2
4
0
3
sin (2π) cos(2π) 3 2π sin(4π)
= + −
+
−
4
4 2
4
3
sin (0) cos(0) 3 0 sin(0)
− −
+
−
4
4 2
4
3π
=
4

sin4 (ϑ)dϑ =
0

−

Substituindo no cálculo de V ol(D) temos:
V ol(D) = 2πa3 +

8a3 3π
3 4

= 4πa3
Em nosso segundo exemplo utilizaremos coordenadas esféricas, Antes porém, veremos como determinar os limites de integração em
coordenadas esféricas.

Figura 5.12: Coordenadas es-

Figura 5.13: Coordenadas es-

féricas 1

féricas 2

Passo 1 Esboçar o domínio D bem como sua projeção D∗ no
plano xy (ver Fig. 5.12).

93
Mudança de Variáveis em Integrais tríplas
Passo 2 Identificar as curvas que limitam a região D∗ . Atravessar
a região D∗ com uma reta r começando na origem (ver Fig. 5.13).
Á medida em que a reta r percorre a região D∗ o ângulo ϑ que ela
forma com o eixo x positivo varia do mínimo α que será o limite
inferior da variável ϑ ao máximo β que será o limite superior da
variável ϑ. Daí, a variável ϑ ∈ [α, β].
Passo 3 Para cada valor fixo da variável ϑ ∈ [α, β] atravessar

Figura 5.14: Coordenadas es-

Figura 5.15: Coordenadas es-

féricas 3

féricas 4

a região D com o plano P que contem o eixo z e forma ângulo ϑ
com o eixo x positivo (ver Fig. 5.14). Traçamos uma reta r que
começa na origem e está contida no plano que corta D. À medida
em que a reta r percorre a região D o ângulo ϕ que ela forma com
o eixo z positivo varia do mínimo α(ϑ) que será o limite inferior da
variável ϕ ao máximo β(ϑ) que será o limite superior da variável
ϕ. Daí, a variável ϕ ∈ [α(ϑ), β(ϑ)].
Passo 4 Para cada valor fixo da variável ϑ ∈ [α, β] e da variável
r ∈ [α(ϑ), β(ϑ)] plano P que contem o eixo z e forma ângulo ϑ com
o eixo x positivo. No plano P traçar a reta s que forma ângulo ϕ

94
Cálculo III

AULA

com o eixo z positivo atravessando a região D (ver Fig. 5.15). O

5

ponto onde a reta s entra na região D é o limite inferior α(ϑ, ϕ)
para a variável r e o ponto onde a reta s sai da região D é o limite
superior β(ϑ, ϕ) para a variável r. Daí, r ∈ [α(ϑ, ϕ), β(ϑ, ϕ)].

Podemos agora encarar o nosso segundo exemplo onde colocaremos
em prática a determinação dos limites de integração em coordenadas esféricas.

Exemplo 5.2. Considere o sólido gerado pela interseção das superfícies: z =

x2 + y 2 (cone), z =

a2 − x2 − y 2 (esfera)(Fig

5.16) e determine seu volume.

Figura 5.16:
exemplo 2

Superfícies de

Figura 5.17: Interseção das
superfícies de exemplo 2

SOLUÇÃO: Para uma melhor compreensão mostramos na (Fig
5.17) o sólido gerado pela interseção das superfícies dadas e na
(Fig 5.18) as superfícies que compõem o sólido separadas no espaço.

95
Mudança de Variáveis em Integrais tríplas

Figura 5.18: Domínio D para o exemplo 2
Usaremos para o caso o sistema de coordenadas esféricas, dada
pela transformação: (x, y, z) → (r, ϑ, ϕ) onde x = r cos(ϑ) cos(ϕ),
y = r sin(ϑ) cos(ϕ) e z = r sin(ϕ). O jacobiano da transformação
é dado por:

J =J

x, y, z
r, ϑ, z

 ∂x
ˆ
 ∂r
 ∂x
ˆ
= det 
 ∂ϑ
 ∂x
ˆ
∂ϕ

∂y
ˆ
∂r
∂y
ˆ
∂ϑ
∂y
ˆ
∂ϕ

∂z
ˆ
∂r
∂z
ˆ
∂ϑ
∂z
ˆ
∂ϕ








Efetuando as derivadas parciais temos:


cos(ϑ) cos(ϕ)

sin(ϑ) cos(ϕ)

sin(ϕ)



J = det  −r sin(ϑ) cos(ϕ) r cos(ϑ) cos(ϕ)
0

−r cos(ϑ) sin(ϕ) −r sin(ϑ) sin(ϕ) r cos(ϕ)







Fazendo as contas do determinante temos:

J = r2 sin(ϕ)
Aproveitaremos o exemplo para aplicar os passos na determinação dos limites de integração de uma integral tripla no sistema de

96
Cálculo III

AULA

5

Figura 5.19: Domínio D para o exemplo 2

coordenadas esféricas expostos acima.
Passo 1:

Esboçar a interseção das superfícies (sólido D), bem

como sua projeção sobre o plano xy (superfície D∗ ), ver (Fig
5.19). A projeção sobre o plano xy (superfície D∗ ), é dada por
x2 + y 2 ≤ a2 .
Passo 2:

Os limites para a variável ϑ são determinados em D∗

como em um sistema de coordenadas polares. No caso como D∗ é
√
a
temos que: 0 ≤ ϑ ≤ 2π.
um disco de raio
2
Passo 3: Os limites para a variável ϕ são determinados em D
do seguinte modo: para cada valor fixo de ϑ, em D∗ , cortamos o
domínio D por um plano que passa no eixo z e forma ângulo ϑ com
o eixo x. Traçamos uma reta M que passa na origem, pertence ao
plano ϑ e atravessa o domínio D. O ângulo ϕ é o ângulo formado
por M e o eixo z positivo. Para o caso o menor valor é ϕ = 0,
quando M conhecide com o eixo Z e o maior valor de ϕ em D
é quando M conhecide com a geratriz do cone z = x2 + y 2 e
π
ϕ= .
4
Passo 4: Os limites para a variável r são determinados em D

97
Mudança de Variáveis em Integrais tríplas
do seguinte modo: para cada par fixo ϑ, ϕ) percorremos a reta
M partindo da origem. O limite inferior de r é o ponto onde a
reta entra em D e o limite superior o ponto onde M sai de D.
Para o nosso caso: 0 ≤ r ≤ a (a reta sai na superfície da esfera
z=

a2 − x2 − y 2 .

Podemos determinar o volume de D pela integral tripla:
2π

π/4

a

r2 sin(ϕ)drdϕdϑ

V ol(D) =
0

0

0

Integrando primeiramente na variável r temos:
2π

π/4

V ol(D) =
0

0

r3
3

a
0

sin(ϕ)dϕdϑ

Substituindo os limites de integração temos:
2π

π/4

a3 03
−
3
3

V ol(D) =
0

0

sin(ϕ)dϕdϑ

Simplificando temos:
V ol(D) =

a3
3

2π

π/4

sin(ϕ)dϕdϑ
0

0

Integrando na variável ϕ temos:
V ol(D) =

a3
3

2π

− cos(ϕ)
0

π/4
0

dϑ

Substituindo os limites de integração temos:
V ol(D) =

a3
3

2π

(− cos(π/4) + cos(0)) dϑ
0

Simplificando temos:
√
a3 2 − 2
V ol(D) =
3
2

2π

dϑ
0

Integrando na variável ϑ temos:
√
a3 2 − 2
V ol(D) =
ϑ
3
2

98

2π
0
AULA

Cálculo III
Substituindo os limites de integração temos:

5

√
a3 2 − 2
V ol(D) =
(2π − 0)
3
2
Finalmente, simplificando temos:
πa3 (2 −
V ol(D) =
3

5.4

√

2)

Conclusão

Na aula de hoje, vimos que algumas vezes é conveniente fazer
uma mudança nas variáveis de integração em uma integral tripla,
para facilitar o cálculo da mesma. Vimos em particularmente duas
mudanças de variáveis são muito importantes e correspondem aos:
sistema de coordenadas cilíndricas e sistema de coordenadas esféricas.

RESUMO

Para o nosso resumo da aula 05 necessitamos algumas considerações iniciais para tratar da mudança de variáveis em integrais
triplas. A saber:
Consideramos a transformação (x, y, z) = T (u, v, w) tal que o domínio um ponto do domínio D ⊂ R3 , (x, y, z) seja transformado
no ponto (u, v, w) do domínio D ⊂ R3 , (D = T (D )) e mais
especificamente x = x(u, v, w), y = y (u, v, w) e z = z (u, v, w). Deˆ
ˆ
ˆ
x, y, z
ou
finindo o jacobiano da transformação, denotado J, J
u, v, w
∂(x, y, z)
, por:
∂(u, v, w)

99
Integrais Duplas
Integrais Duplas
Integrais Duplas
Integrais Duplas
Integrais Duplas
Integrais Duplas
Integrais Duplas
Integrais Duplas
Integrais Duplas
Integrais Duplas
Integrais Duplas
Integrais Duplas
Integrais Duplas
Integrais Duplas
Integrais Duplas
Integrais Duplas
Integrais Duplas
Integrais Duplas
Integrais Duplas
Integrais Duplas
Integrais Duplas
Integrais Duplas
Integrais Duplas
Integrais Duplas
Integrais Duplas
Integrais Duplas
Integrais Duplas
Integrais Duplas
Integrais Duplas
Integrais Duplas
Integrais Duplas
Integrais Duplas
Integrais Duplas
Integrais Duplas
Integrais Duplas
Integrais Duplas
Integrais Duplas
Integrais Duplas
Integrais Duplas
Integrais Duplas
Integrais Duplas
Integrais Duplas
Integrais Duplas
Integrais Duplas
Integrais Duplas
Integrais Duplas
Integrais Duplas
Integrais Duplas
Integrais Duplas
Integrais Duplas
Integrais Duplas
Integrais Duplas
Integrais Duplas
Integrais Duplas
Integrais Duplas
Integrais Duplas
Integrais Duplas
Integrais Duplas
Integrais Duplas
Integrais Duplas
Integrais Duplas
Integrais Duplas
Integrais Duplas
Integrais Duplas
Integrais Duplas
Integrais Duplas
Integrais Duplas
Integrais Duplas
Integrais Duplas
Integrais Duplas
Integrais Duplas
Integrais Duplas
Integrais Duplas
Integrais Duplas
Integrais Duplas
Integrais Duplas
Integrais Duplas
Integrais Duplas
Integrais Duplas
Integrais Duplas
Integrais Duplas
Integrais Duplas
Integrais Duplas
Integrais Duplas
Integrais Duplas
Integrais Duplas
Integrais Duplas
Integrais Duplas
Integrais Duplas
Integrais Duplas
Integrais Duplas
Integrais Duplas
Integrais Duplas
Integrais Duplas
Integrais Duplas
Integrais Duplas
Integrais Duplas
Integrais Duplas
Integrais Duplas
Integrais Duplas
Integrais Duplas
Integrais Duplas
Integrais Duplas
Integrais Duplas

Mais conteúdo relacionado

Mais procurados

Apostila engenharia economica financeira i
Apostila engenharia economica financeira iApostila engenharia economica financeira i
Apostila engenharia economica financeira icustos contabil
 
Aula 14 balanço de energia em processos químicos - 06.05.11
Aula 14   balanço de energia em processos químicos - 06.05.11Aula 14   balanço de energia em processos químicos - 06.05.11
Aula 14 balanço de energia em processos químicos - 06.05.11Nelson Virgilio Carvalho Filho
 
Caderno de Exercícios – Estatística com solução
Caderno de Exercícios – Estatística com soluçãoCaderno de Exercícios – Estatística com solução
Caderno de Exercícios – Estatística com soluçãoOutliers Academy
 
Exercícios Resolvidos: Taxa relacionada
Exercícios Resolvidos: Taxa relacionadaExercícios Resolvidos: Taxa relacionada
Exercícios Resolvidos: Taxa relacionadaDiego Oliveira
 
Relatório de física sobre a lei de hooke
Relatório de física sobre a lei de hookeRelatório de física sobre a lei de hooke
Relatório de física sobre a lei de hookeKarine D'Assunção
 
Matematica volume unico gelson iezzi
Matematica volume unico gelson iezziMatematica volume unico gelson iezzi
Matematica volume unico gelson iezzirosefarias123
 
Gincana Matemática(revisão geral-Ensino Médio) Professor Dimas
Gincana Matemática(revisão geral-Ensino Médio) Professor DimasGincana Matemática(revisão geral-Ensino Médio) Professor Dimas
Gincana Matemática(revisão geral-Ensino Médio) Professor DimasEdimar Santos
 
TRANSFERÊNCIA DE CALOR E MASSA
TRANSFERÊNCIA DE CALOR E MASSATRANSFERÊNCIA DE CALOR E MASSA
TRANSFERÊNCIA DE CALOR E MASSAThomas Willams
 
Apostila compressor centrifugo
Apostila compressor centrifugoApostila compressor centrifugo
Apostila compressor centrifugoMarcelo Araujo
 
Resolução do livro de estática hibbeler 10ª ed - cap 4-6
Resolução do livro de estática   hibbeler 10ª ed - cap 4-6Resolução do livro de estática   hibbeler 10ª ed - cap 4-6
Resolução do livro de estática hibbeler 10ª ed - cap 4-6Jefferson_Melo
 
Exercícios Resolvidos: Equação da reta tangente
Exercícios Resolvidos: Equação da reta tangenteExercícios Resolvidos: Equação da reta tangente
Exercícios Resolvidos: Equação da reta tangenteDiego Oliveira
 
Exercícios Resolvidos: Volume dos sólidos de revolução
Exercícios Resolvidos: Volume dos sólidos de revoluçãoExercícios Resolvidos: Volume dos sólidos de revolução
Exercícios Resolvidos: Volume dos sólidos de revoluçãoDiego Oliveira
 
Mecânica vetorial para engenheiros (estática) 7ª edição beer
Mecânica vetorial para engenheiros (estática) 7ª edição beerMecânica vetorial para engenheiros (estática) 7ª edição beer
Mecânica vetorial para engenheiros (estática) 7ª edição beerAnderson Carvalho
 

Mais procurados (20)

Apostila engenharia economica financeira i
Apostila engenharia economica financeira iApostila engenharia economica financeira i
Apostila engenharia economica financeira i
 
Fatoração
FatoraçãoFatoração
Fatoração
 
Aula 14 balanço de energia em processos químicos - 06.05.11
Aula 14   balanço de energia em processos químicos - 06.05.11Aula 14   balanço de energia em processos químicos - 06.05.11
Aula 14 balanço de energia em processos químicos - 06.05.11
 
Caderno de Exercícios – Estatística com solução
Caderno de Exercícios – Estatística com soluçãoCaderno de Exercícios – Estatística com solução
Caderno de Exercícios – Estatística com solução
 
Exercícios Resolvidos: Taxa relacionada
Exercícios Resolvidos: Taxa relacionadaExercícios Resolvidos: Taxa relacionada
Exercícios Resolvidos: Taxa relacionada
 
Relatório de física sobre a lei de hooke
Relatório de física sobre a lei de hookeRelatório de física sobre a lei de hooke
Relatório de física sobre a lei de hooke
 
Matematica volume unico gelson iezzi
Matematica volume unico gelson iezziMatematica volume unico gelson iezzi
Matematica volume unico gelson iezzi
 
4 cinematica dos fluidos exercícios
4 cinematica dos fluidos exercícios4 cinematica dos fluidos exercícios
4 cinematica dos fluidos exercícios
 
Gincana Matemática(revisão geral-Ensino Médio) Professor Dimas
Gincana Matemática(revisão geral-Ensino Médio) Professor DimasGincana Matemática(revisão geral-Ensino Médio) Professor Dimas
Gincana Matemática(revisão geral-Ensino Médio) Professor Dimas
 
Programacao linear aula 3 metodo grafico
Programacao linear   aula 3 metodo graficoProgramacao linear   aula 3 metodo grafico
Programacao linear aula 3 metodo grafico
 
TRANSFERÊNCIA DE CALOR E MASSA
TRANSFERÊNCIA DE CALOR E MASSATRANSFERÊNCIA DE CALOR E MASSA
TRANSFERÊNCIA DE CALOR E MASSA
 
Aula 07 derivadas - regras de derivação - parte 1
Aula 07   derivadas - regras de derivação - parte 1Aula 07   derivadas - regras de derivação - parte 1
Aula 07 derivadas - regras de derivação - parte 1
 
Equação do 1º grau
Equação do 1º grauEquação do 1º grau
Equação do 1º grau
 
Apostila compressor centrifugo
Apostila compressor centrifugoApostila compressor centrifugo
Apostila compressor centrifugo
 
Resolução do livro de estática hibbeler 10ª ed - cap 4-6
Resolução do livro de estática   hibbeler 10ª ed - cap 4-6Resolução do livro de estática   hibbeler 10ª ed - cap 4-6
Resolução do livro de estática hibbeler 10ª ed - cap 4-6
 
Exercícios Resolvidos: Equação da reta tangente
Exercícios Resolvidos: Equação da reta tangenteExercícios Resolvidos: Equação da reta tangente
Exercícios Resolvidos: Equação da reta tangente
 
Exercícios Resolvidos: Volume dos sólidos de revolução
Exercícios Resolvidos: Volume dos sólidos de revoluçãoExercícios Resolvidos: Volume dos sólidos de revolução
Exercícios Resolvidos: Volume dos sólidos de revolução
 
Mecânica vetorial para engenheiros (estática) 7ª edição beer
Mecânica vetorial para engenheiros (estática) 7ª edição beerMecânica vetorial para engenheiros (estática) 7ª edição beer
Mecânica vetorial para engenheiros (estática) 7ª edição beer
 
Didática da física
Didática da físicaDidática da física
Didática da física
 
Ciclo de Brayton
Ciclo de BraytonCiclo de Brayton
Ciclo de Brayton
 

Destaque

Calculo.vol 3.guidorizzi.pdf
Calculo.vol 3.guidorizzi.pdfCalculo.vol 3.guidorizzi.pdf
Calculo.vol 3.guidorizzi.pdfGustavo Padovany
 
Equações diferenciais dennis g. zill vol 01
Equações diferenciais   dennis g. zill vol 01Equações diferenciais   dennis g. zill vol 01
Equações diferenciais dennis g. zill vol 01ricardoehumasiladabino
 
Inequações modulares
Inequações modularesInequações modulares
Inequações modularesjvcastromattos
 
Equações diferenciais 1990
Equações diferenciais   1990Equações diferenciais   1990
Equações diferenciais 1990Romeu da Silveira
 
Introdução ao estudo de Arquitetura de Computadores - (baseado em Stallings, ...
Introdução ao estudo de Arquitetura de Computadores - (baseado em Stallings, ...Introdução ao estudo de Arquitetura de Computadores - (baseado em Stallings, ...
Introdução ao estudo de Arquitetura de Computadores - (baseado em Stallings, ...Isabel Mendes Pedrosa
 
[Solução] cálculo com geometria leithold
[Solução] cálculo com geometria   leithold[Solução] cálculo com geometria   leithold
[Solução] cálculo com geometria leitholdAnderson Carvalho
 
Livro proprietario calculo diferencia e integral iii
Livro proprietario   calculo diferencia e integral iiiLivro proprietario   calculo diferencia e integral iii
Livro proprietario calculo diferencia e integral iiiAndré Pinto
 
28817084 equacoes-diferenciais-dennis-g-zill-7ed-resolvido
28817084 equacoes-diferenciais-dennis-g-zill-7ed-resolvido28817084 equacoes-diferenciais-dennis-g-zill-7ed-resolvido
28817084 equacoes-diferenciais-dennis-g-zill-7ed-resolvidoakhenatoni
 
(Aula 3) equações logaritmicas
(Aula 3) equações logaritmicas(Aula 3) equações logaritmicas
(Aula 3) equações logaritmicasnetaulasifpaitb
 
Adjetivos pátrios - estados e localidades brasileiras
Adjetivos pátrios - estados e localidades brasileirasAdjetivos pátrios - estados e localidades brasileiras
Adjetivos pátrios - estados e localidades brasileirasAndriane Cursino
 
Resolução leithold - vol. 01 e 02
Resolução   leithold - vol. 01 e 02Resolução   leithold - vol. 01 e 02
Resolução leithold - vol. 01 e 02Claudia Sá de Moura
 
Logaritmos
LogaritmosLogaritmos
LogaritmosIsabel
 
Fundamentos geometria i
Fundamentos geometria iFundamentos geometria i
Fundamentos geometria icon_seguir
 
FIGURAS PLANAS
FIGURAS PLANASFIGURAS PLANAS
FIGURAS PLANASedmildo
 

Destaque (19)

Calculo.vol 3.guidorizzi.pdf
Calculo.vol 3.guidorizzi.pdfCalculo.vol 3.guidorizzi.pdf
Calculo.vol 3.guidorizzi.pdf
 
Equações diferenciais dennis g. zill vol 01
Equações diferenciais   dennis g. zill vol 01Equações diferenciais   dennis g. zill vol 01
Equações diferenciais dennis g. zill vol 01
 
Inequações modulares
Inequações modularesInequações modulares
Inequações modulares
 
Equações diferenciais 1990
Equações diferenciais   1990Equações diferenciais   1990
Equações diferenciais 1990
 
Equações diferenciais ordinárias
Equações diferenciais ordináriasEquações diferenciais ordinárias
Equações diferenciais ordinárias
 
Séries e Seqüências
Séries e SeqüênciasSéries e Seqüências
Séries e Seqüências
 
Introdução ao estudo de Arquitetura de Computadores - (baseado em Stallings, ...
Introdução ao estudo de Arquitetura de Computadores - (baseado em Stallings, ...Introdução ao estudo de Arquitetura de Computadores - (baseado em Stallings, ...
Introdução ao estudo de Arquitetura de Computadores - (baseado em Stallings, ...
 
[Solução] cálculo com geometria leithold
[Solução] cálculo com geometria   leithold[Solução] cálculo com geometria   leithold
[Solução] cálculo com geometria leithold
 
Livro proprietario calculo diferencia e integral iii
Livro proprietario   calculo diferencia e integral iiiLivro proprietario   calculo diferencia e integral iii
Livro proprietario calculo diferencia e integral iii
 
28817084 equacoes-diferenciais-dennis-g-zill-7ed-resolvido
28817084 equacoes-diferenciais-dennis-g-zill-7ed-resolvido28817084 equacoes-diferenciais-dennis-g-zill-7ed-resolvido
28817084 equacoes-diferenciais-dennis-g-zill-7ed-resolvido
 
(Aula 3) equações logaritmicas
(Aula 3) equações logaritmicas(Aula 3) equações logaritmicas
(Aula 3) equações logaritmicas
 
Adjetivos pátrios - estados e localidades brasileiras
Adjetivos pátrios - estados e localidades brasileirasAdjetivos pátrios - estados e localidades brasileiras
Adjetivos pátrios - estados e localidades brasileiras
 
Resolução leithold - vol. 01 e 02
Resolução   leithold - vol. 01 e 02Resolução   leithold - vol. 01 e 02
Resolução leithold - vol. 01 e 02
 
Logaritmos
LogaritmosLogaritmos
Logaritmos
 
Calculo vol 2, 11va ed thomas
Calculo vol 2, 11va ed   thomasCalculo vol 2, 11va ed   thomas
Calculo vol 2, 11va ed thomas
 
Fundamentos geometria i
Fundamentos geometria iFundamentos geometria i
Fundamentos geometria i
 
Sequencias e series calculo
Sequencias e series   calculoSequencias e series   calculo
Sequencias e series calculo
 
Aula de LOGARITMOS
Aula de LOGARITMOSAula de LOGARITMOS
Aula de LOGARITMOS
 
FIGURAS PLANAS
FIGURAS PLANASFIGURAS PLANAS
FIGURAS PLANAS
 

Semelhante a Integrais Duplas

Semelhante a Integrais Duplas (20)

Geometria euclidiana plana
Geometria euclidiana planaGeometria euclidiana plana
Geometria euclidiana plana
 
Ap ami v8
Ap ami v8Ap ami v8
Ap ami v8
 
Fismat apostila
Fismat apostilaFismat apostila
Fismat apostila
 
Residuos
ResiduosResiduos
Residuos
 
Algebra linear
Algebra linearAlgebra linear
Algebra linear
 
Intro teoria dos numerros i
Intro teoria dos numerros iIntro teoria dos numerros i
Intro teoria dos numerros i
 
Lista01
Lista01Lista01
Lista01
 
Apostila Calculo II - UDESC.pdf
Apostila Calculo II - UDESC.pdfApostila Calculo II - UDESC.pdf
Apostila Calculo II - UDESC.pdf
 
Livro Completo sobre Maple
Livro Completo sobre MapleLivro Completo sobre Maple
Livro Completo sobre Maple
 
Ap amiii
Ap amiiiAp amiii
Ap amiii
 
Funcões de uma variável
Funcões de uma variávelFuncões de uma variável
Funcões de uma variável
 
05 calculo ii imp
05 calculo ii imp05 calculo ii imp
05 calculo ii imp
 
Livrocalcii
LivrocalciiLivrocalcii
Livrocalcii
 
Principio de Ánalise
Principio de Ánalise Principio de Ánalise
Principio de Ánalise
 
Friedli, s. cálculo 1. 1ª ed. belo horizonte, imprensa universitária da ufmg,...
Friedli, s. cálculo 1. 1ª ed. belo horizonte, imprensa universitária da ufmg,...Friedli, s. cálculo 1. 1ª ed. belo horizonte, imprensa universitária da ufmg,...
Friedli, s. cálculo 1. 1ª ed. belo horizonte, imprensa universitária da ufmg,...
 
Aa booklet
Aa bookletAa booklet
Aa booklet
 
Calculo 2p
Calculo 2pCalculo 2p
Calculo 2p
 
Csharp e orientacao a objetos
Csharp e orientacao a objetosCsharp e orientacao a objetos
Csharp e orientacao a objetos
 
Qm31
Qm31Qm31
Qm31
 
Qm31
Qm31Qm31
Qm31
 

Integrais Duplas

  • 1. Sumário Aula 1: Integrais Duplas 11 1.1 Introdução . . . . . . . . . . . . . . . . . . . . . . . 12 1.2 Integral Dupla: Domínios Retangulares . . . . . . . . 12 1.3 Integral Dupla: Domínios Não Retangulares Limitados 14 1.4 Interpretação Geométrica . . . . . . . . . . . . . . . 15 1.5 Integrais Iteradas . . . . . . . . . . . . . . . . . . . 16 1.6 Propriedades das Integrais Duplas . . . . . . . . . . 19 1.7 Alguns Exemplos . . . . . . . . . . . . . . . . . . . 20 1.8 Conclusão . . . . . . . . . . . . . . . . . . . . . . . 25 RESUMO . . . . . . . . . . . . . . . . . . . . . . . . . 25 PRÓXIMA AULA . . . . . . . . . . . . . . . . . . . . 29 ATIVIDADES . . . . . . . . . . . . . . . . . . . . . . 29 LEITURA COMPLEMENTAR . . . . . . . . . . . 30 Aula 2: Mudança de Variáveis em Integrais Duplas 33 2.1 Introdução . . . . . . . . . . . . . . . . . . . . . . . 34 2.2 Mudança de Variáveis em Integrais Duplas . . . . . 34 2.3 Alguns Exemplos . . . . . . . . . . . . . . . . . . . 39 2.4 Conclusão . . . . . . . . . . . . . . . . . . . . . . . 43 RESUMO . . . . . . . . . . . . . . . . . . . . . . . . . 43 PRÓXIMA AULA . . . . . . . . . . . . . . . . . . . . 45
  • 2. ATIVIDADES . . . . . . . . . . . . . . . . . . . . . . 45 LEITURA COMPLEMENTAR . . . . . . . . . . . 46 Aula 3: Algumas Aplicações da Integral Dupla 47 3.1 Introdução . . . . . . . . . . . . . . . . . . . . . . . 48 3.2 Preliminares . . . . . . . . . . . . . . . . . . . . . . 48 3.3 Algumas Aplicações da Integral Dupla . . . . . . . . 52 3.4 Conclusão . . . . . . . . . . . . . . . . . . . . . . . 58 RESUMO . . . . . . . . . . . . . . . . . . . . . . . . . 58 PRÓXIMA AULA . . . . . . . . . . . . . . . . . . . . 59 ATIVIDADES . . . . . . . . . . . . . . . . . . . . . . 59 LEITURA COMPLEMENTAR . . . . . . . . . . . 60 Aula 4: Integrais triplas 63 4.1 Introdução . . . . . . . . . . . . . . . . . . . . . . . 64 4.2 Integração Tripla: Domínios Paralelepípedais . . . . 64 4.3 Integração Tripla: Domínios Não Paralelepípedais Limitados . . . . . . . . . . . . . . . . . . . . . . . . 66 4.4 Interpretação Geométrica . . . . . . . . . . . . . . . 67 4.5 Integrais Iteradas . . . . . . . . . . . . . . . . . . . 67 4.6 Propriedades das Integrais Triplas . . . . . . . . . . 68 4.7 Exemplos . . . . . . . . . . . . . . . . . . . . . . . 69 4.8 Conclusão . . . . . . . . . . . . . . . . . . . . . . . 74 RESUMO . . . . . . . . . . . . . . . . . . . . . . . . . 75 PRÓXIMA AULA . . . . . . . . . . . . . . . . . . . . 80 ATIVIDADES . . . . . . . . . . . . . . . . . . . . . . 80 LEITURA COMPLEMENTAR . . . . . . . . . . . 81
  • 3. Aula 5: Mudança de Variáveis em Integrais tríplas 83 5.1 Introdução . . . . . . . . . . . . . . . . . . . . . . . 84 5.2 Mudança de Variáveis em Integrais Triplas . . . . . . 84 5.3 Alguns Exemplos . . . . . . . . . . . . . . . . . . . 87 5.4 Conclusão . . . . . . . . . . . . . . . . . . . . . . . 99 RESUMO . . . . . . . . . . . . . . . . . . . . . . . . . 99 PRÓXIMA AULA . . . . . . . . . . . . . . . . . . . . 103 ATIVIDADES . . . . . . . . . . . . . . . . . . . . . . 103 LEITURA COMPLEMENTAR . . . . . . . . . . . 104 Aula 6: Algumas Aplicações das Integrais tríplas 105 6.1 Introdução . . . . . . . . . . . . . . . . . . . . . . . 106 6.2 Preliminares . . . . . . . . . . . . . . . . . . . . . . 106 6.3 Algumas Aplicações da Integral Tripla . . . . . . . . 110 6.4 Conclusão . . . . . . . . . . . . . . . . . . . . . . . 117 RESUMO . . . . . . . . . . . . . . . . . . . . . . . . . 118 PRÓXIMA AULA . . . . . . . . . . . . . . . . . . . . 119 ATIVIDADES . . . . . . . . . . . . . . . . . . . . . . 120 LEITURA COMPLEMENTAR . . . . . . . . . . . 120 Aula 7: Integrais de Funções Vetoriais sobre Curvas em R3 123 7.1 Introdução . . . . . . . . . . . . . . . . . . . . . . . 124 7.2 Curvas em R3 . . . . . . . . . . . . . . . . . . . . . 124 7.3 Massa, Momento de Massa e Momento de Inércia de Curvas em R3 . . . . . . . . . . . . . . . . . . . . . 126 7.4 Campos Vetoriais: Trabalho, Circulação e Fluxo . . . 128 7.5 Independência do Caminho . . . . . . . . . . . . . . 130 7.6 Algumas Aplicações das Integrais de Linha . . . . . 133
  • 4. 7.7 Conclusão . . . . . . . . . . . . . . . . . . . . . . . 138 RESUMO . . . . . . . . . . . . . . . . . . . . . . . . . 139 PRÓXIMA AULA . . . . . . . . . . . . . . . . . . . . 142 ATIVIDADES . . . . . . . . . . . . . . . . . . . . . . 142 LEITURA COMPLEMENTAR . . . . . . . . . . . 143 Aula 8: Integrais de Superfícies 145 8.1 Introdução . . . . . . . . . . . . . . . . . . . . . . . 146 8.2 Superfícies em R3 . . . . . . . . . . . . . . . . . . . 146 8.3 Área de Superfícies em R3 . . . . . . . . . . . . . . 147 8.4 Momento de massa e Momento de Inércia de Superfícies de Casca Fina em R3 . . . . . . . . . . . . . . 151 8.5 Superfícies Parametrizadas . . . . . . . . . . . . . . 155 8.6 Conclusão . . . . . . . . . . . . . . . . . . . . . . . 160 RESUMO . . . . . . . . . . . . . . . . . . . . . . . . . 160 PRÓXIMA AULA . . . . . . . . . . . . . . . . . . . . 164 ATIVIDADES . . . . . . . . . . . . . . . . . . . . . . 164 LEITURA COMPLEMENTAR . . . . . . . . . . . 165 Aula 9: Teorema de Green e Teorema de Stokes 167 9.1 Introdução . . . . . . . . . . . . . . . . . . . . . . . 168 9.2 Preliminares . . . . . . . . . . . . . . . . . . . . . . 168 9.3 Teorema de Green 9.4 Estendendo o Teorema de Green para Outras Regiões 175 9.5 Verificação do Teorema de Green . . . . . . . . . . 178 9.6 Teorema de Stokes . . . . . . . . . . . . . . . . . . 181 9.7 Aplicação do Teorema de Stokes . . . . . . . . . . . 183 9.8 Conclusão . . . . . . . . . . . . . . . . . . . . . . . 185 . . . . . . . . . . . . . . . . . . 171 RESUMO . . . . . . . . . . . . . . . . . . . . . . . . . 185
  • 5. PRÓXIMA AULA . . . . . . . . . . . . . . . . . . . . 187 ATIVIDADES . . . . . . . . . . . . . . . . . . . . . . 187 LEITURA COMPLEMENTAR . . . . . . . . . . . 188 Aula 10: Teorema de Divergência 189 10.1 Introdução . . . . . . . . . . . . . . . . . . . . . . . 190 10.2 Preliminares . . . . . . . . . . . . . . . . . . . . . . 190 10.3 Teorema da Divergência . . . . . . . . . . . . . . . 191 10.4 Estendendo o Teorema da Divergência . . . . . . . . 194 10.5 Algumas Aplicações do Teorema da Divergência . . 196 10.6 Conclusão . . . . . . . . . . . . . . . . . . . . . . . 200 RESUMO . . . . . . . . . . . . . . . . . . . . . . . . . 200 ATIVIDADES . . . . . . . . . . . . . . . . . . . . . . 202 LEITURA COMPLEMENTAR . . . . . . . . . . . 203
  • 6.
  • 7. AULA Integrais Duplas META: Apresentar integrais duplas de funções de valores reais e domínio em R2 . OBJETIVOS: Ao fim da aula os alunos deverão ser capazes de: Definir a integral dupla de funções de valores reais e domínio em R2 . Calcular algumas integrais duplas de funções de valores reais e domínio em R2 . PRÉ-REQUISITOS Os conhecimentos de integrais de funções de valores reais com domínio em R, da disciplina Cálculo I. 1
  • 8. Integrais Duplas 1.1 Introdução Caros alunos iniciamos aqui nosso curso de Cálculo III com o tema “Integrais Dupla”. A integração dupla, em essência, é uma extensão natural da integral simples vista em Cálculo I e definida como limite de somas de Riemann. Na prática, a integração dupla é dada por duas integrações simples, cada uma efetuada sobre uma variável e considerando as demais como constantes. É o que denominamos de integrais interadas. Suas características e detalhes próprios serão vistas ao longo do nosso curso, nas próximas duas aulas. 1.2 Integral Dupla: Domínios Retangulares Começamos por considerar uma função f definida em um domínio retangular R = {(x, y) ∈ R2 |a ≤ x ≤ b∧c ≤ y ≤ d}. Formalmente f : [a, b]×[c, d] → R. Usando a imaginação, pensemos em R coberta por uma rede de retas paralelas aos eixos coordenados e que dividem R em pequenos retângulos (Fig. 1.1) . Oficialmente, consideraremos duas partições P [a, b] = {x0 = a, x1 , . . . , xj , xj+1 , . . . , xm = b} e P [c, d] = {y0 = c, y1 , . . . , yk , yk+1 , . . . , yn = d} onde como visto em Cálculo I temos: x0 < x1 < · · · < xj < xj+1 < · · · < xm e y0 < y1 < · · · < yk < yk+1 < · · · < yn . Desta forma cada um dos Ij = [xj−1 , xj ] e Jk = [yk−1 , yk ] pequenos subintervalos têm comprimentos ∆xj = xj − xj−1 e ∆yk = yk − yk−1 , respectivamente. Definimos, agora, a uma partição para o retângulo R por P = P [R] = P [a, b] × P [c, d], o produto cartesiano das partições P [a, b] e P [c, d]. As retas retalham a região R em uma série de retângulos Ajk = [xj−1 , xj ] × [yk−1 , yk ], 1 ≤ 12
  • 9. Cálculo III AULA 1 Figura 1.1: Partição de R = [a, b] × [c, d] j ≤ m, 1 ≤ k ≤ n. A area de cada pequeno retângulo é dada por ∆Ajk = ∆xj ∆yk . Como tanto ∆xj quanto ∆yk são diferentes de zero, a área de cada pequeno retângulo é também diferente de zero. Podemos então definir a norma da partição por: |P | = max (∆Ajk ), que corresponde a maior área entre todos os 1≤j≤m 1≤k≤n pequeno retângulo. Pausa para respirar que já vamos definir a integral dupla sobre domínios retangulares. Para isto tomamos um ponto (ξj , ζk ) ∈ [xj−1 , xj ] × [yk−1 , yk ] em cada pequeno retângulo e definimos a seguinte soma de Riemann: m n Smn = f (ξj , ζk )∆Ajk j=1 k=1 A integral dupla da função f (x, y) sobre o retângulo R, denotada f (x, y)dxdy será então definida como o seguinte limite: R def f (x, y)dxdy = lim Smn R BIOGRAFIA Georg Friedrich Bernhard Riemann nasceu em Breselenz, Reino de Hanôver, 17 de Setembro de 1826 e morreu em Selasca, Itália, 20 de Junho de 1866, foi um matemático alemão, com contribuições fundamentais para a análise e a geometria diferencial. Wikipedia |P |→0 13
  • 10. Integrais Duplas Figura 1.2: Soma de Riemann para f (x, y) em R = [a, b] × [c, d] 1.3 Integral Dupla: Domínios Não Retangulares Limitados Para definir a integral dupla de uma função f : D ⊂ R2 → R onde D é não é uma região retangular, porém é limitada, começamos por considerar uma função F definida em um domínio retangular R = {(x, y) ∈ R2 |a ≤ x ≤ b ∧ c ≤ y ≤ d} tal   f (x, y) , (x, y) ∈ D que D ⊂ R e F (x, y) = . Formalmente  0 , (x, y) ∈ D / F : [a, b] × [c, d] → R é uma extensão da função f (x, y). Usando a imaginação, pensemos em R coberta por uma rede de retas paralelas aos eixos coordenados e que dividem R em pequenos retângulos e procedemos como na integral dupla sobre domínios retangulares, considerando a uma partição para o retângulo R por 14
  • 11. Cálculo III AULA P = P [R] = P [a, b] × P [c, d], o produto cartesiano das partições 1 P [a, b] e P [c, d] onde P [a, b] = {x0 = a, x1 , . . . , xj , xj+1 , . . . , xm = b} e P [c, d] = {y0 = c, y1 , . . . , yk , yk+1 , . . . , yn = d}. Do mesmo modo definimos a norma da partição por: |P | = max (∆Ajk ) 1≤j≤m 1≤k≤n onde ∆Ajk = ∆xj ∆yk , ∆xj = xj − xj−1 e ∆yk = yk − yk−1 . Tomamos um ponto (ξj , ζk ) ∈ [xj−1 , xj ] × [yk−1 , yk ] em cada pequeno retângulo e definimos a seguinte soma de Riemann para a função estendida F (x, y): m n Smn = F (ξj , ζk )∆Ajk j=1 k=1 A integral dupla da função f (x, y) sobre o domínio D ⊂ R2 , denof (x, y)dxdy será então definida como o seguinte limite: tada D def f (x, y)dxdy = lim Smn D |P |→0 Observem na partição (Fig. 1.3) que apenas os pequenos retângulos cinza claro contribuem para a soma de Riemann os demais têm contribuição nula visto que o ponto escolhido dentro destes estão fora de D ⊂ R2 e portanto F (ξj , ζk ) = 0. 1.4 Interpretação Geométrica Quando a função f (x, y) é positiva na região R, como a da (Fig. 1.2), vemos que a soma de Riemann aproxima o volume do prisma sólido reto limitado inferiormente por R e superiormente pela superfície z = f (x, y) e quanto maior for o refinamento da partição de R melhor será a aproximação. Podemos então, interpretar f (x, y)dxdy como o volume do prisma sólido a integral dupla R 15
  • 12. Integrais Duplas Figura 1.3: Partição para F (x, y) em R = [a, b] × [c, d] reto limitado inferiormente por R e superiormente pela superfície z = f (x, y). 1.5 Integrais Iteradas Do mesmo modo que para a integral simples, na integral dupla a soma de Riemann não é um modo prático de se calcular uma integral dupla. Vejamos agora um procedimento que facilitará o cálculo de integrais duplas. Vamos exemplificar calculando o volume de um prisma reto de base retangular, limitado inferiormente por [a, b] × [c, d] e superiormente pela função de valores positivos f (x, y). para cada valor fixo de x no intervalo [a, b] consideremos o perfil A(x) (área da seção transversal em x) (Fig. 1.4) fazemos o produto por dx e integramos no intervalo [a, b]. Isto resulta no 16
  • 13. Cálculo III AULA 1 Figura 1.4: A(x), x fixo, integramos em relação a y volume do citado prisma. b V = A(x)dx a Por outro lado o perfil A(x) é dada pela área abaixo da curva f (x, y), fixado o x, entre os valores de y no intervalo [c, d]. E como d f (x, y)dy. vimos em Cálculo I A(x) = c O volume do prisma pode ser então escrito como: b d V = f (x, y)dy dx a c . Podemos alternativamente calcular o mesmo volume considerando os perfis A(y) (área da seção transversal em y) (Fig. 1.5) fazemos o produto por dy e integramos no intervalo [c, d]. Isto resulta no volume do citado prisma. 17
  • 14. Integrais Duplas Figura 1.5: A(y), y fixo, integramos em relação a x d A(y)dy V = c b Da mesma forma como vimos em Cálculo I A(y) = f (x, y)dx. a O volume do prisma pode ser então escrito como: d b V = f (x, y)dx dy c a . Como o volume dado pelas duas expressões é o mesmo temos que: b d d b f (x, y)dx dy = c a f (x, y)dy dx a c ou seja a ordem em que as integrais simples são executadas não altera o resultado final da integração dupla em domínios retangulares. Este procedimento e conhecido como integrais iteradas. 18
  • 15. AULA Cálculo III 1.6 Propriedades das Integrais Duplas 1 Como nosso curso é de Cálculo, apenas listaremos, sem demonstração, alguma das propriedades das integrais duplas. Caso desejem conhecer a demonstração de algumas destas propriedades, recomendo livros de Cálculo Avançado como os citados na bibliografia abaixo. Propriedade 1.1. Sejam f : D ⊂ R2 → R uma função de valores reais integrável em D e c ∈ R, então vale: cf (x, y)dxdy = c f (x, y)dxdy D D Propriedade 1.2. Sejam f, g : D ⊂ R2 → R duas funções de valores reais integráveis em D, então vale: (f + g)(x, y)dxdy = D f (x, y)dxdy + D g(x, y)dxdy D Propriedade 1.3. Sejam f : D ⊂ R2 → R uma função de valores reais integrável em D tal que f (x, y) ≥ 0, ∀(x, y) ∈ D, então vale: f (x, y)dxdy ≥ 0 D Propriedade 1.4. Sejam f, g : D ⊂ R2 → R duas funções de valores reais integráveis em D tais que f (x, y) ≥ g(x, y), ∀(x, y) ∈ D, então vale: f (x, y)dxdy ≥ D g(x, y)dxdy D Propriedade 1.5. Seja f : D ⊂ R2 → R uma função de valores reais integrável em D onde D = A ∪ B e A ∩ B é a união de um 19
  • 16. Integrais Duplas número finito de curvas em R2 , então vale: f (x, y)dxdy = D f (x, y)dxdy + A f (x, y)dxdy B OBS 1.1. As duas primeiras propriedades diz respeito à “linearidade” do operador integral dupla. As terceira e quarta propriedades são denominadas “dominação” enquanto que a quinta propriedade é denominada “aditividade”. 1.7 Alguns Exemplos Nada mais natural que ilustrar um novo conceito com exemplos e, vamos aqui fazer exatamente isto, ilustrar o conceito de integral dupla com dois exemplos. Antes porém, vale observar que a na prática uma integral dupla equivale a duas integrais simples e neste caso uma pergunta fica no ar e não deixaremos sem resposta. Qual das duas variáveis x ou y integraremos primeiro? Muito bem, a resposta é dada pela propria expressão da integral dupla. Isto f (x, y)dxdy primeiramente integramos na va- é, na integral R f (x, y)dydx riável x e depois na variável y. Já na integral R primeiramente integramos na variável y e depois na variável x. Vamos diretamente para o primeiro exemplo de integral dupla sobre domínios retangulares. A saber: Exemplo 1.1. Considere a função f : [0, 1] × [0, 1] → R (Fig. 1.6) dada por f (x, y) = exp(−x − y) e determine a integral dupla f (x, y)dxdy sobre a região R = {(x, y) ∈ R2 |0 ≤ x ≤ I = R 1 ∧ 0 ≤ y ≤ 1}. 20
  • 17. Cálculo III AULA 1 Figura 1.6: Função f : [0, 1] × [0, 1] → R: f (x, y) = exp(−x − y) SOLUÇÃO: Passo 1 colocaremos os limites de integração que representam a região R dada, segundo a ordem de integração: 1 1 exp(−x − y)dxdy I= 0 0 Lembrando que: exp(−x − y) = exp(−x) exp(−y) temos: 1 1 exp(−x) exp(−y)dxdy I= 0 0 Passo 2 integraremos na variável x considerando a variável y como uma constante: 1 − exp(−x) I= 0 1 0 exp(−y)dy Substituindo os limites de integração temos: 1 (− exp(−1) − (− exp(−0))) exp(−y)dy I= 0 Efetuando os cálculos temos: 1 (1 − exp(−1)) exp(−y)dy I= 0 Passo 3 integraremos na variável y considerando a variável: I = (1 − exp(−1)) − exp(−y) 1 0 Substituindo os limites de integração temos: I = (1 − exp(−1)) (− exp(−1) − (− exp(−0))) Efetuando os cálculos temos: 21
  • 18. Integrais Duplas I = (1 − exp(−1))2 OBS 1.2. Daremos aqui um método prático para determinar os limites de integração em uma integral dupla sobre domínio não retangular da forma: D. Passo 1 Fazer um desenho da região D. (Fig. 1.7) identificando as curvas inferior a(x) e superior b(x) que limitam a região D. Passo 2 Atravessar toda a região D e o eixo x com um segmento de reta paralelo e orientado na direção positiva ao eixo y (segmento AB na Fig. 1.7) Passo 3 Deslocar o segmento de reta AB paralelo ao eixo y na Figura 1.7: Determinação prática dos limites para D direção negativa do eixo x até tocar o ponto mais à esquerda de D marcando o limite inferior de x (ponto a na Fig. 1.7). Passo 4 Deslocar o segmento de reta AB paralelo ao eixo y na direção positiva do eixo x até tocar o ponto mais à direita de D marcando o limite superior de x (ponto b na Fig. 1.7). Passo 5 Tomando um ponto qualquer x ∈ (a, b) passamos o seg- 22
  • 19. Cálculo III AULA mento de reta AB através da região D. O limite inferior para a 1 variável y será a função a(x), ponto da curva onde o segmento entra na região D e o limite superior para a variável y será b(x), ponto da curva onde o segmento de reta sai da região D. Nossa integral será efetuada assim: b b(x) f (x, y)dydx f (x, y)dxdy = D a a(x) Exemplo 1.2. Considere a função f : [0, 1] × [0, 1] → R (Fig. 1.8) dada por f (x, y) = y(3x − x2 − y) e determine a integral duf (x, y)dxdy sobre a região D ∈ R2 interseção das pla I = R curvas y = 0 e y = 3x − x2 . Figura 1.8: Função f : [0, 1] × [0, 1] → R: f (x, y) = x.y SOLUÇÃO: Passo 1 faremos o desenho das duas curvas que determinam os limites para a região D. A saber y = 0 e y = 3x − x2 (Fig. 1.9). Passo 2 usando o processo prático exposto acima determinamos os limites de integração. A saber: a = 0, b = 3, a(x) = 0 e b(x) = 3x − x2 . 23
  • 20. Integrais Duplas Figura 1.9: Limites para o domínio D A integral passa a ser escrita como: 3x−x2 3 I= y(3x − x2 − y)dydx f (x, y)dxdy = 0 0 Operando no integrando fazendo o produto por y temos: 3x−x2 3 (y(3x − x2 ) − y 2 )dydx I= 0 0 Passo 3 efetuando a integração em y temos: 3 2 y y 3 3x−x2 I= ( (3x − x2 ) − ) dx 2 3 0 0 Substituindo os limit3es de integração temos: 3 (3x − x2 )2 (3x − x2 )3 I= ( (3x − x2 ) − )dx 2 3 0 Efetuando as simplificações teremos: 3 (3x − x2 )3 I= dx 6 0 Expandindo o binômio de Newton temos: 1 3 I= (27x3 − 27x4 + 9x5 − x6 )dx 6 0 Passo 4 efetuando a integração em x temos: 1 x4 x5 x6 x7 3 I = (27 − 27 + 9 − ) 6 4 5 6 7 0 Substituindo os limit3es de integração temos: 1 34 35 36 37 I = (27 − 27 + 9 − ) 6 4 5 6 7 Efetuando os cálculos, garantido muito trabalho, temos: 24
  • 21. Cálculo III I= 1.8 AULA 1 729 280 Conclusão Na aula de hoje, vimos que a integral dupla é uma extensão natural do conceito de integral simples visto em Cálculo I. E se por um lado a integral simples pode ser interpretada como a área sob a curva descrita pela função a ser integrada, a integral dupla pode ser vista como o volume sob a superfície descrita pela função a ser duplamente integrada. RESUMO No nosso resumo da Aula 01 constam os seguintes tópicos: Integração Dupla: Domínios retangulares Considerando uma função f : R → R onde R = {(x, y) ∈ R2 |a ≤ x ≤ b ∧ c ≤ y ≤ d} é um retângulo em R2 . Podemos cobri-lo com uma malha de retas formada pela partição: P = P [R] = P [a, b] × P [c, d] onde cada P [a, b] = {x0 = a, x1 , . . . , xj , xj+1 , . . . , xn = b} e P [c, d] = {y0 = c, y1 , . . . , yk , yk+1 , . . . , ym = d} são partições dos intervalos [a, b] em x e [c, d] em y respectivamente. A malha divide R nos retângulos Ajk = [xj−1 , xj ] × [yk−1 , yk ], 1 ≤ j ≤ n, 1 ≤ k ≤ m de área ∆Ajk = ∆xj ∆yk onde ∆xj = xj − xj−1 e ∆yk = yk − yk−1 são os comprimentos dos subintervalos Ij = [xj−1 , xj ] e Jk = [yk−1 , yk ] respectivamente. Defini-se a norma da partição por: |P | = max (∆Ajk ). Toma-se um ponto (ξj , ζk ) ∈ 1≤j≤n 1≤k≤m 25
  • 22. Integrais Duplas [xj−1 , xj ] × [yk−1 , yk ] em cada retângulo Ajk e definimos a seguinte soma de Riemann: n m Snm = f (ξj , ζk )∆Ajk j=1 k=1 A integral dupla da função f (x, ) sobre o retângulo R, denotada f (x, y)dxdy será então definida como o seguinte limite: R def f (x, y)dxdy = lim Snm |P |→0 R Integração Dupla: Domínios não Retangulares Para definir a integral dupla de uma função f : D ⊂ R2 → R onde D é não é uma região retangular, porém é limitada, começamos por considerar uma função F definida em um domínio retangular R = {(x, y) ∈ R2 |a ≤ x ≤ b ∧ c ≤ y ≤ d} tal   f (x, y) , (x, y) ∈ D que D ⊂ R e F (x, y) = . Formalmente  0 , (x, y) ∈ D / F : [a, b] × [c, d] → R é uma extensão da função f (x, y). A partir daqui todo o procedimento é semelhante ao da definição da integral dupla em domínios retangulares. Podemos definir a integral dupla de uma função f (x, y) em um domínio não retangular D por: def f (x, y)dxdy = lim Smn D onde: Smn = m j=1 |P |→0 n k=1 F (ξj , ζk )∆Ajk . é a soma de Riemann para F (x, y) Integrais Iteradas As integrais iteradas dizem que em um domínio retangular R = 26
  • 23. Cálculo III AULA [a, b] × [c, d] a ordem de execução das integrais simples não alteram 1 o valor da integral dupla, que pode ser representada por: d b b d f (x, y)dy dx f (x, y)dx dy = c a a c . Propriedades das Integrais Duplas As integrais duplas são de certo modo semelhantes às propriedades das integrais simples que vimos em Cálculo I sendo quase que uma extensão natural destas. As integrais duplas têm, entre outras, as seguintes propriedades: Propriedade 1 Sejam f : D ⊂ R2 → R uma função de valores reais integrável em D e c ∈ R, então vale: cf (x, y)dxdy = c D f (x, y)dxdy D Propriedade 2 Sejam f, g : D ⊂ R2 → R duas funções de valores reais integráveis em D, então vale: (f + g)(x, y)dxdy = D f (x, y)dxdy + D g(x, y)dxdy D Propriedade 3 Sejam f : D ⊂ R2 → R uma função de valores reais integrável em D tal que f (x, y) ≥ 0, ∀(x, y) ∈ D, então vale: f (x, y)dxdy ≥ 0 D 27
  • 24. Integrais Duplas Propriedade 4 Sejam f, g : D ⊂ R2 → R duas funções de valores reais integráveis em D tais que f (x, y) ≥ g(x, y), ∀(x, y) ∈ D, então vale: f (x, y)dxdy ≥ D g(x, y)dxdy D Propriedade 5 Seja f : D ⊂ R2 → R uma função de valores reais integrável em D onde D = A ∪ B e A ∩ B é a união de um número finito de curvas em R2 , então vale: f (x, y)dxdy = D f (x, y)dxdy + A f (x, y)dxdy B Determinação dos Limites de Integração Para determinar os limites de integração em uma integral dupla sobre domínio não retangular da forma: D seguimos os seguintes passos: Passo 1 Fazer um desenho da região D. (Fig. 1.7) identificando as curvas inferior a(x) e superior b(x) que limitam a região D. Passo 2 Atravessar toda a região D e o eixo x com um segmento de reta paralelo e orientado na direção positiva ao eixo y (segmento AB na Fig. 1.7) Passo 3 Deslocar o segmento de reta AB paralelo ao eixo y na direção negativa do eixo x até tocar o ponto mais à esquerda de D marcando o limite inferior de x (ponto a na Fig. 1.7). 28
  • 25. Cálculo III AULA Passo 4 Deslocar o segmento de reta AB paralelo ao eixo y na 1 direção positiva do eixo x até tocar o ponto mais à direita de D marcando o limite superior de x (ponto b na Fig. 1.7). Passo 5 Tomando um ponto qualquer x ∈ (a, b) passamos o segmento de reta AB através da região D. O limite inferior para a variável y será a função a(x), ponto da curva onde o segmento entra na região D e o limite superior para a variável y será b(x), ponto da curva onde o segmento de reta sai da região D. Nossa integral será efetuada assim: b b(x) f (x, y)dxdy = D f (x, y)dydx a a(x) PRÓXIMA AULA Em nossa próxima aula veremos mudança de variáveis na integração dupla. O objetivo da mudança de variáveis em uma integral dupla será a de facilitar esta integração de uma de duas formas. A primeira será tornando o integrando mais simples. A segunda transformando o domínio D do integrando em um domínio de forma geométrica mais simples. ATIVIDADES Deixamos como atividades o cálculo de algumas integrais duplas. ATIV. 1.1. Seja f : [−1, +1] × [−1, +1] → R dada por f (x, y) = 29
  • 26. Integrais Duplas x2 + y 2 . Determine a integral dupla f (x, y)dxdy. R Comentário: Volte ao texto e reveja com calma e atenção o cálculo de integrais duplas dos exemplos acima, elas lhe servirão de guia. ATIV. 1.2. Seja f : D ⊂ R2 → R dada por f (x, y) = x2 + y 2 , onde D = {(x, y) ∈ R2 |x ≥ 0 ∧ 0 ≤ y ≤ 1 − x2 }. • Determine os limites da integral dupla f (x, y)dxdy, D • esboce a região de integração e • calcule a integral dupla f (x, y)dxdy. D Comentário: Volte ao texto e reveja com calma e atenção o cálculo de integrais duplas dos exemplos acima, elas lhe servirão de guia. LEITURA COMPLEMENTAR ÁVILA, Geraldo, Cálculo 3: Funções de Várias Variáveis, Livros Técnicos e Científicos Editora, São Paulo, 3a edição, 1982. LEITHOLD, Louis, O Cálculo com Geometria Analítica. Volume 2, Editora Harbra, 1994. STEWART, James,Cálculo. Volume 3, 5a edição, Editora CENGAGE Learning, 2009. SWOKOWSKI, Earl E., Cálculo com Geometria Analítica, Volume 2, 2a edição, Makron Books do Brásil SP, 1994. THOMAS, George B., Cálculo, Volume 2, 10a, Addilson Wesley, 30
  • 27. Cálculo III 2003. AULA 1 KAPLAN, Wilfred, Cálculo Avançado Vol.1 e vol.2 Editora Edgard Blücher 1991.// SPIEGEL, Murray R. Cálculo Avançado, Editora McGraw-Hill do Brasil, 1971. BOUCHARA, Jacques, Cálculo Integral Avançado, EDUSP, 2006. 31
  • 28.
  • 29. AULA Mudança de Variáveis em Integrais Duplas META: Introduzir mudança de variáveis em integrais duplas de funções de valores reais e domínio em R2 . OBJETIVOS: Ao fim da aula os alunos deverão ser capazes de: Calcular o jacobiano de aplicações de R2 em R2 . Calcular integrais duplas de funções de valores reais e domínio em R2 utilizando mudança de variáveis. Calcular integrais duplas de funções de valores reais e domínio em R2 em coordenadas polares. PRÉ-REQUISITOS Os conhecimentos de integrais de funções de valores reais com domínio em R, da disciplina Cálculo I, curvas em R2 e coordenadas polares da disciplina Cálculo II e integrais duplas aula 01. 2
  • 30. Mudança de Variáveis em Integrais Duplas 2.1 Introdução Caros alunos a segunda aula do nosso curso de Cálculo III tem com o tema “Mudança de Variáveis em Integrais Duplas”. As f (x, y)dxdy, dada a natureza ou de vezes, na integral dupla HISTÓRIA O teorema de mudança de variáveis em integrais duplas foi primeiro proposto por Euler quando ele desenvolveu a noção de integral dupla em 1769. Usado por Legendre, Laplace e Gauss, foi primeiramente generalizado para n variáveis por Mikhail Ostrogradski em 1836, resistiu a uma demonstração mais rigorosa por longo tempo (cerca de 125 anos). E foi satisfatóriamente demonstrado por Elie Cartan em uma série de artigos nos anos 1890. D f (x, y) ao do seu domínio D, fica mais fácil integrar se fizermos uma mudança nas variáveis de integração, como quando D é uma disco, um semi-disco, um setor circular ou mesmo uma faixa de disco, usando-se o sistema de coordenadas polares de modo geral a integral dupla é mais fácil de se determinar que em coordenadas cartesianas. 2.2 Mudança de Variáveis em Integrais Duplas Caros alunos começaremos revendo mudança de variáveis em integrais simples. Considere uma função f : [a, b] → R. A idéia é mudar a variável inicial x para uma nova variável ξ relacionadas por x = g(ξ), onde g(ξ) é uma função biunívoca estritamente crescente ou estritamente decrescente em [a, b]. Isto garante que podemos inverter a mudança de variáveis. Seja F (x) uma anti-derivada de f (x) tal que F (x) = f (x). Então, da regra da cadeia temos: d F (g(ξ)) = F (g(ξ))g (ξ) = f (g(ξ))g (ξ). dξ Integrando com respeito a ξ temos: d F (g(ξ))dξ = f (g(ξ))g (ξ)dξ dξ Das propriedades da integral temos: F (g(ξ)) + C = f (g(ξ))g (ξ)dξ Como x = g(ξ) temos: F (x) + C = 34 f (g(ξ))g (ξ)dξ
  • 31. Cálculo III AULA Como F (x) é uma primitiva de f (x) a primeira expressão é a in- 2 tegral indefinida de f (x) com respeito a x e temos: f (x)dx = f (g(ξ))g (ξ)dξ Que representa a mudança de variáveis em uma integral simples. Para integrais definidas, se c = g(a) e d = g(b) então: d b f (g(ξ))g (ξ)dξ f (x)dx = c a A expressão acima funciona bem quando g(ξ) é crescente neste caso a < b e c < d. Porém, no caso de g(ξ) decrescente (g (ξ) < 0) pois neste caso a < b e d < c e portanto o limite inferior da segunda integral não conhecide com o limite inferior do intervalo da imagem de g(ξ) o mesmo acontecendo com o limite superior. Neste caso, usando as propriedades da integral simples temos: b c f (x)dx = − f (g(ξ))g (ξ)dξ a d De outra forma escrevemos: b c f (x)dx = a f (g(ξ))|g (ξ)|dξ. d e operaremos os limites inferiores e superiors das integrais como os limites inferiores e superiores dos domínios (intervalos) e a expressão acima vale tanto pra g(ξ) crescente quanto decrescente. Vamos agora diretamente ao assunto dando uma argumentação heurística para a expressão da mudança de variáveis em integrais duplas. f (x, y)dxdy sobre Para isto, consideremos a integral dupla D uma região D ∈ R2 do plano (x, y) e a transformação (x, y) = T (u, v) tal que o domínio D do plano (x, y) seja a imagem do domínio D do plano (u, v) (podemos expressar este fato como D = T (D )). Mais especificamente podemos escrever: x = x(u, v) ˆ e y = y (u, v) tomando uma partição para o domínio D no plano ˆ OBSERVAÇÃO heurística heu.rís.ti.ca sf (gr heuristiké) 1 Ciência ou arte do procedimento heurístico. 2 Método de ensino que consiste em que o educando chegue à verdade por seus próprios meios. 3 Ramo da ciência histórica que consiste na pesquisa dos documentos do passado. (u, v) cobrindo-o com pequenos retângulos e usando a transformação T podemos levar o pequeno retângulo Ajk na pequena figura 35
  • 32. Mudança de Variáveis em Integrais Duplas plana Ajk = T (Ajk ) (ver Fig 2.1 e Fig 2.2). A área do pequeno retângulo no plano (u, v) é ∆Ajk a área da pequena figura Ajk no plano (x, j), e ai é que reside a argumentação heurística, será apro∂T ximada pela área do paralelogramo formado pelos vetores ∆vk ∂v ∂T e ∆uj e pelas linhas tracejadas (paralelas aos respectivos veto∂u res). Do calculo vetorial temos: ∂T ∂x ˆ ∂y ˆ ∆uj = ∆uj i + ∆uj j + 0k. ∂u ∂u ∂u ∂T ∂x ˆ ∂y ˆ ∆vk = ∆vk i + ∆vk j + 0k ∂v ∂v ∂v Vistos como vetores de R3 e a área do paralelogramo (ver Vetores e Geometria Analítica) dada pelo módulo do seguinte produto vetorial: ∂T ∂T ∆uj × ∆vk . ∂u ∂v Fazendo o cálculo do produto vetorial temos:  ∆Ajk =  i j k    ∂x  ˆ ∂y ˆ ∂T ∂T  ∆uj × ∆vk = det   ∂u ∆uj ∂u ∆uj 0  ∂u ∂v  ∂x  ∂y ˆ ˆ ∆vk ∆vk 0 ∂v ∂v Fazendo os cálculos temos: ∂T ∂T ∂x ∂y ∂x ∂y ˆ ˆ ˆ ˆ ∆uj × ∆vk = − ∆uj ∆vk k. ∂u ∂v ∂u ∂v ∂v ∂u Tomando o módulo da expressão acima, para a área de Ajk , temos: ˆ ˆ ∂x ∂y ∂x ∂y ˆ ˆ − ∆uj ∆vk . ∆Ajk ≈ ∂u ∂v ∂v ∂u A expressão dentro do módulo é o determinante de uma matrix 2 × 2 conhecida como jacobiano da transformação x = x(u, v) e ˆ y = y (u, v) e denotado: ˆ   ∂x ∂y ˆ ˆ ˆ ˆ ˆ ˆ ∂(x, y)   ∂x ∂y ∂x ∂y = det  ∂u ∂u  = − . ∂x ∂y ˆ ˆ ∂(u, v) ∂u ∂v ∂v ∂u ∂v ∂v Como a área do pequeno retângulo Ajk é dada por ∆Ajk = ∆uj ∆vk temos: ∆Ajk ≈ 36 ∂(x, y) ∆Ajk . ∂(u, v)
  • 33. Cálculo III AULA 2 Figura 2.1: Plano (u, v) Figura 2.2: Plano (x, y) O que nos leva a considerar a seguinte fórmula para a mudança de variáveis em integrais duplas: ∂(x, y) dudv. ∂(u, v) D D Que representa a mudança de variáveis na integral dupla pela f (x, y)dxdy = f (ˆ(u, v), y (u, v)) x ˆ transformação (x, y) = T (u, v). OBS 2.1. Para o caso particular da mudança de variáveis do sistema de coordenadas cartesianas para o sistema de coordenadas polares (x, y) = T (r, ϑ) = (r cos(ϑ), r sin(ϑ)) onde x = x(r, ϑ) = ˆ r cos(ϑ) e y = (r, ϑ) = r sin(ϑ), o jacobiano é dado por: y ˆ    ∂x ∂y ˆ ˆ cos(ϑ) sin(ϑ) ∂(x, y)  ∂r ∂r   = r. = det  ∂ x ∂ y  = det  ˆ ˆ ∂(r, ϑ) −r sin(ϑ) r cos(ϑ) ∂ϑ ∂ϑ ∂(x, y) Portanto o jacobiano da transformação = r a mudança de ∂(r, ϑ) variáveis na integral dupla toma a forma: f (x, y)dxdy = D f (r cos(ϑ), r sin(ϑ))rdrdϑ. D OBS 2.2. Daremos aqui um método prático para determinar os limites de integração em uma integral dupla sobre domínio não retangular da forma: D em coordenadas polares. 37
  • 34. Mudança de Variáveis em Integrais Duplas Passo 1 Fazer um desenho da região D (Fig. 2.3), identificando as curvas que limitam a região D. Passo 2 Atravessar toda a região D com um raio r (ϑ) orientado na direção positiva (Fig. 2.3) Passo 3 Deslocar o raio r (ϑ) na direção negativa do ângulo ϑ (diNOTA reção horária) até tocar o ponto mais à negativa de D marcando Por convenção a medida de ângulo tem sinal positivo quando o deslocamento é feito na direção anti-horária, direção contrária ao movimento dos ponteiros do relógio e tem sinal negativo quando o deslocamento é feito na direção horária, direção do movimento dos ponteiros do relógio. Figura 2.3: Determinação prática dos limites para D o limite inferior de ϑ (ângulo α na Fig. 2.3). Passo 4 Deslocar o raio r (ϑ) na direção positiva do ângulo ϑ (direção anti-horária) até tocar o ponto mais à positiva de D marcando o limite inferior de ϑ (ângulo β na Fig. 2.3). Passo 5 Tomando um ponto qualquer ϑ ∈ (α, β) passamos o raio r (ϑ) através de D o limite inferior para a variável r será a função α(ϑ), ponto da curva onde o raio r (ϑ) entra na região D e o limite superior para a variável r será β(ϑ), ponto da curva onde o raio r (ϑ) sai da região D. Nossa integral será efetuada assim: β β(ϑ) f (x, y)dxdy = D 38 f (r cos(ϑ), r sin(ϑ))rdrdϑ α α(ϑ)
  • 35. AULA Cálculo III 2.3 2 Alguns Exemplos Caros alunos, nesta seção ilustraremos, com dois exemplos, a mudança de variáveis em integrais duplas. A rigor, trataremos apenas de exemplos em coordenadas polares. f (x, y)dxdy onde Exemplo 2.1. Determinar a integral dupla D D = {(x, y) ∈ R2 |x ≥ 0 ∧ y ≥ 0 ∧ x2 + y 2 ≤ 1} e f (x, y) = exp(−x2 − y 2 ). O domínio da função representa um quarto de disco (Fig 2.4). Figura 2.4: Gráfico do exemplo 1 SOLUÇÃO: Passo 1 Como o domínio D é um quarto de disco, o mais adequado é utilizar o sistema de coordenadas polares. Podemos usar o método prático de determinação dos limites da integral dupla π em coordenadas polares (Fig 2.5) e verificar que: α = 0, β = , 2 α(ϑ) = 0 e β(ϑ) = 1. Neste caso podemos descrever o domínio como: D = {(r, ϑ) ∈ R2 |0 ≤ r ≤ 1 ∧ 0 ≤ ϑ ≤ π/2}. E como x = r cos(ϑ) e y = r sin(ϑ) e 39
  • 36. Mudança de Variáveis em Integrais Duplas Figura 2.5: Gráfico do exemplo 1 ∂(x, y) = r. ∂(r, ϑ) Quanto a variável r varia no intervalo [0, 1] independentemente de o módulo do jacobiano da transformação é dado por: ϑ e a variável ϑ varia no intervalo [0, π/2] ( a variação de ângulo no primeiro quadrante). Podemos reescrever a integral dupla como: 1 I= π/2 f (x, y)dxdy = D f (r cos(ϑ), r sin(ϑ))rdϑdr Subs0 0 tituindo f (x, y) temos: 1 π/2 exp(−(r cos(ϑ))2 − (r sin(ϑ))2 )rdϑdr I= 0 0 Efetuando as simplificações temos: 1 π/2 exp(−r2 )rdϑdr I= 0 0 Passo 2 Integrando primeiramente na variável ϑ e como o integrando não depende de ϑ temos: 1 exp(−r2 )ϑ I= 0 π/2 0 rdr Substituindo os limites de integração temos: 1 exp(−r2 )rdr I = π/2 0 Passo 3 A última integral (variável r) podemos efetuar por mudança de variáveis pondo ξ = r2  deste modo temos: dξ = 2rdr   1  1 1 ou seja rdr = − dξ e os limites r eξ . Daí, a integral  0  0 2 40
  • 37. Cálculo III 2 passará a forma: 1 I = π/4 AULA exp(−ξ)dξ 0 Cuja integração é fácil e da forma: I = π/4 − exp(−ξ) 1 0 Efetuando os cálculo temos: π I = (1 − exp(−1)) 4 Vamos agora, diretamente ao nosso segundo exemplo. Trata-se de uma curva já conhecida de vocês (Cálculo II) a lemniscata. Exemplo 2.2. Determinar a área da região D, a parte da lemniscata, r = cos(2ϑ), que situa-se no primeiro quadrante. ver parte cinza da (Fig 2.6). Figura 2.6: Gráfico do exemplo 2 SOLUÇÃO: Passo 1 Como o domínio D é um quarto de uma lemniscata, o mais adequado é utilizar o sistema de coordenadas polares. Podemos usar o método prático de determinação dos limites da integral dupla em coordenadas polares (Fig 2.7) e verificar que: α = 0, 41
  • 38. Mudança de Variáveis em Integrais Duplas β= π , α(ϑ) = 0 e β(ϑ) = 4 cos(2ϑ). Figura 2.7: Gráfico do exemplo 2 Neste caso podemos descrever o domínio como: D = {(r, ϑ) ∈ R2 |0 ≤ ϑ ≤ π/4 ∧ 0 ≤ r ≤ cos(2ϑ)}. E como, neste exemplo, queremos calcular área temos que f (x, y) = 1 e em coordenadas polares podemos escrever na forma da seguinte integral dupla: √ cos(2ϑ) π/4 A= rdrdϑ dxdy = D 0 0 Integrando em r temos: √ π/4 2 cos(2ϑ) r A= dϑ 2 0 0 Substituindo os limites de integração temos: 2 π/4 cos(2ϑ) dϑ A= 2 0 Simplificando o integrando temos: π/4 cos(2ϑ) A= dϑ 2 0 Integrando na variável ϑ temos: sin(2ϑ) π/4 A= 4 0 Substituindo os limites de integração temos: sin(π/2) − sin(0) A= 4 Portanto: 1 A= 4 42
  • 39. Cálculo III AULA 2 OBS 2.3. Caros alunos, é muito importante neste ponto uma revisão cuidadosa e detalhada dos dois exemplos dados acima. Efetuar uma mudança de varáveis em integrais duplas não é tão simples quanto efetuar uma mudança de variáveis em integrais simples. 2.4 Conclusão Na aula de hoje, vimos que a mudança de variáveis em integrais dupla, nos permite, facilitar o cálculo das ditas integrais quando trabalhamos com domínios de integração de geometrias específicas, como a induzida pelas coordenadas polares. RESUMO No nosso resumo da Aula 02 constam os seguintes tópicos: Mudança de Variáveis em Integrais Duplas Consideramos a transformação (x, y) = T (u, v) tal que o domínio D do plano (x, y) seja transformado no domínio D do plano (u, v) (D = T (D )) e mais especificamente x = x(u, v) e y = y (u, v). ˆ ˆ ∂(x, y) Definindo o jacobiano da transformação, denotado , por: ∂(u, v)   ˆ ∂x ∂y ˆ ∂(x, y) ˆ ˆ ˆ ˆ   ∂x ∂y ∂x ∂y = det  ∂u ∂u  = − . ∂x ∂y ˆ ˆ ∂(u, v) ∂u ∂v ∂v ∂u ∂v ∂v Vale então,a seguinte fórmula para a mudança de variáveis en integrais duplas: 43
  • 40. Mudança de Variáveis em Integrais Duplas f (ˆ(u, v), y (u, v)) x ˆ f (x, y)dxdy = D D ∂(x, y) dudv. ∂(u, v) Sistema de Coordenadas Polares Para o caso particular da mudança de variáveis do sistema de coordenadas cartesianas para o sistema de coordenadas polares no cálculo de integrais duplas temos: (x, y) = T (r, ϑ) = (r cos(ϑ), r sin(ϑ)) onde x = x(r, ϑ) = r cos(ϑ) e ˆ y = y (r, ϑ) = r sin(ϑ). ˆ Vale a seguinte transformação de variáveis: f (x, y)dxdy = D f (r cos(ϑ), r sin(ϑ))rdrdϑ. D Determinação dos Limites de Integração em Coordenadas Polares Daremos aqui um método prático para determinar os limites de integração em uma integral dupla sobre domínio não retangular da forma: D em coordenadas polares. Passo 1 Fazer um desenho da região D (Fig. 2.3), identificando as curvas que limitam a região D. Passo 2 Atravessar toda a região D com um raio r (ϑ) orientado na direção positiva (Fig. 2.3) Passo 3 Deslocar o raio r (ϑ) na direção negativa do ângulo ϑ (direção horária) até tocar o ponto mais à negativa de D marcando o limite inferior de ϑ (ângulo α na Fig. 2.3). Passo 4 Deslocar o raio r (ϑ) na direção positiva do ângulo ϑ (direção anti-horária) até tocar o ponto mais à positiva de D marcando o limite inferior de ϑ (ângulo β na Fig. 2.3). Passo 5 Tomando um ponto qualquer ϑ ∈ (α, β) passamos o raio 44
  • 41. Cálculo III AULA r (ϑ) através de D o limite inferior para a variável r será a função 2 α(ϑ), ponto da curva onde o raio r (ϑ) entra na região D e o limite superior para a variável r será β(ϑ), ponto da curva onde o raio r (ϑ) sai da região D. Nossa integral será efetuada assim: β β(ϑ) f (x, y)dxdy = D f (r cos(ϑ), r sin(ϑ))rdrdϑ α α(ϑ) PRÓXIMA AULA Em nossa próxima aula veremos algumas das inúmeras aplicações da integral dupla. Nossa atenção estará voltada para o cálculo do centro de massa de perfis planos bem como no cálculo de seus momentos de inércia. ATIVIDADES Deixamos como atividades as seguintes questões. ATIV. 2.1. Determine a área da parte da cardióide r(ϑ) = 1 + cos(ϑ) que fica acima do eixo dos x (Fig 2.8) que está em cinza. Comentário: Volte ao texto e reveja com calma e atenção os exemplos acima, eles lhe servirão de guia. ATIV. 2.2. Determine a área entre a cardióide r(ϑ) = 1+cos(ϑ) e o círculo r(ϑ) = 1 acima do eixo do x (Fig 2.9) que está em cinza. Comentário: Volte ao texto e reveja com calma e atenção os exemplos acima, eles lhe servirão de guia. 45
  • 42. Mudança de Variáveis em Integrais Duplas Figura 2.8: Atividade 1 Figura 2.9: Atividade 2 LEITURA COMPLEMENTAR ÁVILA, Geraldo, Cálculo 3: Funções de Várias Variáveis, Livros Técnicos e Científicos Editora, São Paulo, 3a edição, 1982. LEITHOLD, Louis, O Cálculo com Geometria Analítica. Volume 2, Editora Harbra, 1994. STEWART, James,Cálculo. Volume 3, 5a edição, Editora CENGAGE Learning, 2009. SWOKOWSKI, Earl E., Cálculo com Geometria Analítica, Volume 2, 2a edição, Makron Books do Brásil SP, 1994. THOMAS, George B., Cálculo, Volume 2, 10a, Addilson Wesley, 2003. KAPLAN, Wilfred, Cálculo Avançado Vol.1 e vol.2 Editora Edgard Blücher 1991.// SPIEGEL, Murray R. Cálculo Avançado, Editora McGraw-Hill do Brasil, 1971. BOUCHARA, Jacques, Cálculo Integral Avançado, EDUSP, 2006. 46
  • 43. AULA Algumas Aplicações da Integral Dupla META: Apresentar algumas aplicações das integrais duplas de funções de valores reais e domínio em R2 . OBJETIVOS: Ao fim da aula os alunos deverão ser capazes de: Determinar área, massa, centro de massa, momento de massa e momento de inércia de figuras planas usando integrais duplas de funções de valores reais e domínio em R2 . PRÉ-REQUISITOS Os conhecimentos de integrais de funções de valores reais com domínio em R, da disciplina Cálculo I, curvas em R2 e coordenadas polares da disciplina Cálculo II e integrais duplas aula 01 e aula 02. 3
  • 44. Algumas Aplicações da Integral Dupla 3.1 Introdução Caros alunos nesta terceira aula do nosso curso de Cálculo III com o tema “Algumas Aplicações das Integrais Duplas”. Dentre as inúmeras aplicações da integral dupla, veremos apenas duas pelo pouco tempo que dispomos. Veremos apenas como usar as integrais duplas para calcular a massa de uma região plana dada sua distribuição de densidade e como calcular seu centro de gravidade. Para outras aplicações recomendo uma busca na INTERNET 3.2 Preliminares Consideraremos uma região D ⊂ R2 finita, com uma distribuição de densidade mássica superficial (massa por unidade de superfície) (x, y), ∀(x, y) ∈ D. Determinação da massa Para determinar a massa consideremos uma função Φ definida em um domínio retangular R = {(x, y)  R2 |a ≤ x ≤ b ∧ c ≤ ∈  (x, y) , (x, y) ∈ D y ≤ d} tal que D ⊂ R e Φ(x, y) = .  0 , (x, y) ∈ D / Considerando a uma partição para o retângulo R dada por P = P [R] = P [a, b] × P [c, d], o produto cartesiano das partições P [a, b] e P [c, d] onde P [a, b] = {x0 = a, x1 , . . . , xj , xj+1 , . . . , xm = b} e P [c, d] = {y0 = c, y1 , . . . , yk , yk+1 , . . . , yn = d}. Tomamos um ponto (ξj , ζk ) ∈ [xj−1 , xj ] × [yk−1 , yk ] em cada pequeno retângulo e definimos a seguinte soma de Riemann: m n Smn = Φ(ξj , ζk )∆Ajk . j=1 k=1 48
  • 45. Cálculo III AULA A massa da região D, denotada m(D), será a integral dupla da fun- 3 ção (x, y) sobre o domínio D ⊂ R2 , denotada (x, y)dxdy D será então definida como o seguinte limite: def (x, y)dxdy = lim Smn m(D) = |P |→0 D . OBS 3.1. Para a determinação do peso da região D toma-se a seguinte soma de Riemann: m n Smn = g(ξj , ζk )Φ(ξj , ζk )∆Ajk j=1 k=1 onde g(ξj , ζk ) é a aceleração da gravidade no ponto (ξj , ζk ). E o peso da região D, denotado p(D), será dado pela integral dupla: def p(D) = g(x, y) (x, y)dxdy = lim Smn |P |→0 D . Determinação do Momento de Massa Usando as mesmas considerações acima para o cálculo da massa de uma região D limitada com distribuição de densidade (x, y). Para calcular o momento de massa de um pequeno retângulo com relação ao eixo y tomamos o seguinte produto ξj Φ(ξj , ζk )∆Ajk . O momento de massa total em relação ao eixo y para a região D será aproximado pelo limite da soma de Riemann: m n Smn = ξj Φ(ξj , ζk )∆Ajk j=1 k=1 . O momento de massa da região D em relação ao eixo y será dada 49
  • 46. Algumas Aplicações da Integral Dupla pelo limite: def x (x, y)dxdy = lim Smn My (D) = |P |→0 D . De forma semelhante chega-se ao momento de massa da região D em relação ao eixo x tomando-se a seguinte soma de Riemann: m n Smn = ζk Φ(ξj , ζk )∆Ajk j=1 k=1 . O momento de massa da região D em relação ao eixo x será dada pelo limite: def Mx (D) = y (x, y)dxdy = lim Smn D |P |→0 . Determinação do Centro de Massa O centro de massa de uma região plana D ⊂ R2 finita, com uma distribuição de densidade mássica superficial (x, y), ∀(x, y) ∈ D, é o ponto (¯, y ) definido por: x ¯ x= ¯ My (D) = m(d) x (x, y)dxdy D (x, y)dxdy D Mx (D) y= ¯ = m(d) y (x, y)dxdy D (x, y)dxdy D Determinação do Momento de Inércia Usando as mesmas considerações acima para o cálculo da massa 50
  • 47. Cálculo III AULA de uma região D limitada com distribuição de densidade (x, y). 3 Para calcular o momento de inércia de um pequeno retângulo com 2 relação ao eixo y tomamos o seguinte produto ξj Φ(ξj , ζk )∆Ajk . O momento de inércia total em relação ao eixo y para a região D será aproximado pelo limite da soma de Riemann: m n 2 ξj Φ(ξj , ζk )∆Ajk Smn = j=1 k=1 . O momento de inércia da região D em relação ao eixo y será dada pelo limite: def x2 (x, y)dxdy = lim Smn Iy (D) = |P |→0 D . De forma semelhante chega-se ao momento de inércia da região D em relação ao eixo x tomando-se a seguinte soma de Riemann: m n 2 ζk Φ(ξj , ζk )∆Ajk Smn = j=1 k=1 . O momento da região D em relação ao eixo x será dada pelo limite: def y 2 (x, y)dxdy = lim Smn Ix (D) = D |P |→0 . O momento de inércia em relação a origem é dado pela seguinte integral dupla: 51
  • 48. Algumas Aplicações da Integral Dupla (x2 + y 2 ) (x, y)dxdy I0 (D) = D . 3.3 Algumas Aplicações da Integral Dupla Faremos duas aplicações da integral dupla ao cálculo do centro de massa de duas figuras planas. Na primeira usaremos o sistema de coordenadas cartesiano. Na segunda usaremos uma mudança de variáveis para o sistema de coordenadas polares. Vamos aos nossos exemplos. Exemplo 3.1. Para o primeiro exemplo desejamos determinar o centro de massa de uma região triangular D dada pela interseção das retas x = 0, y = 0 e a reta que passa pelos pontos (0, a) e (b, 0) com a, b > 0 (Fig 3.1), cuja densidade superficial de massa é constante (x, y) = . Figura 3.1: Gráfico do exemplo 1 52
  • 49. AULA Cálculo III 3 SOLUÇÃO: Começaremos por determinar os limites de integração inspeciox nando a (Fig 3.1) e verificando que 0 ≤ x ≤ a e 0 ≤ y ≤ b 1 − . a Em segundo calcularemos a massa da região D, m(D) e os respectivos momentos de massa com relação ao eixo x e ao eixo y Mx (D) e My (D) respectivamente. Passo 1 determinar a massa m(D), dada pela integral dupla: a m(D) = b(1−x/a) (x, y)dxdy = dydx 0 D 0 Integrando em y temos: a m(D) = b(1−x/a) y 0 0 dx Substituindo os limites de integração temos: a x m(D) = b 1 − dx a 0 Integrando em x temos: x2 a m(D) = b x − 2a 0 Substituindo os limites de integração temos: a2 m(D) = b a − 2a Simplificando temos: ab m(D) = 2 Passo 2 calcular o momento de massa Mx (D) dado pela integral dupla: Mx (D) = (x, y)ydxdy D Substituindo os limites temos: a Mx (D) = b(1−x/a) (x, y)ydxdy = D ydydx 0 0 Integrando em y teremos: a y 2 b(1−x/a) dx Mx (D) = 2 0 0 Substituindo os limites de integração temos: a (b(1 − x/a))2 Mx (D) = dx 2 0 Simplificando o integrando temos: 53
  • 50. Algumas Aplicações da Integral Dupla a b2 b2 x b2 x2 dx − + 2 a 2a2 0 Integrando em x teremos: b2 x b2 x2 b2 x3 a Mx (D) = − + 2 2a 6a2 0 Substituindo os limites de integração temos: b2 a b2 a2 b2 a3 Mx (D) = − + 2 2a 6a2 Simplificando as frações temos: b2 a Mx (D) = 6 Passo 3 calcular o momento de massa My (D) dado pela integral Mx (D) = dupla: My (D) = (x, y)xdxdy D My (D) = (x, y)xdxdy D Substituindo os limites temos: a My (D) = b(1−x/a) (x, y)xdxdy = D xdydx 0 0 Integrando em y teremos: a b(1−x/a) xy My (D) = 0 0 dx Substituindo os limites de integração temos: a x My (D) = bx 1 − dx a 0 Integrando em x teremos: x2 x3 a My (D) = b − 2 3a 0 Substituindo os limites de integração temos: a3 a2 − My (D) = b 2 3a Simplificando as frações temos: ba2 My (D) = 6 Passo 4 Determinar o centro de massa de D pelas fórmulas: My (D) Mx (D) x= ¯ ey= ¯ . m(D) m(D) Usando os resultados anteriores temos: 54
  • 51. Cálculo III ba2 b2 a x= 6 ey= 6 ¯ ¯ ab ab 2 2 Simplificando temos: a b x= ey= ¯ ¯ 3 3 AULA 3 Como segundo exemplo usaremos uma região em que o sistema de coordenadas polares facilita os cálculos. Exemplo 3.2. Para o segundo exemplo desejamos determinar o centro de massa de uma região D dada pelo quarto da coroa circular de raio interno a e raio externo b que situa-se no primeiro quadrante (Fig 3.2), cuja densidade superficial de massa é constante (x, y) = . Figura 3.2: Gráfico do exemplo 2 SOLUÇÃO: Começaremos por determinar os limites de integração inspecionando a (Fig 3.2) e verificando que 0 ≤ ϑ ≤ π/2 e a ≤ r ≤ b. Em segundo calcularemos a massa da região D, m(D) e os respectivos momentos de massa com relação ao eixo x e ao eixo y Mx (D) e My (D) respectivamente. 55
  • 52. Algumas Aplicações da Integral Dupla Passo 1 determinar a massa m(D), dada pela integral dupla: π/2 m(D) = b (x, y)dxdy = D rdrdϑ 0 a Integrando em r temos: π/2 r2 b m(D) = dϑ 2 a 0 Substituindo os limites de integração temos: π/2 b2 a2 m(D) = dϑ − 2 2 0 Integrando em ϑ temos: π/2 b2 a2 m(D) = − ϑ 2 2 0 Substituindo os limites de integração temos: 1 π(b2 − a2 ) m(D) = 4 Passo 2 calcular o momento de massa Mx (D) dado pela integral dupla: Mx (D) = (x, y)ydxdy D Substituindo os limites em coordenadas polares e sabendo que y = r sin(ϑ) temos: π/2 Mx (D) = b (x, y)ydxdy = D r sin(ϑ)rdrdϑ 0 a Integrando em r temos: π/2 r3 b dϑ 3 a 0 Substituindo os limites de integração temos: π/2 b3 a3 − dϑ Mx (D) = sin(ϑ) 3 3 0 Integrando em ϑ temos: π/2 b3 a3 Mx (D) = − (− cos(ϑ)) 3 3 0 Substituindo os limites de integração temos: b3 a3 Mx (D) = − (− cos(π/2) − − cos(0)) 3 3 Simplificando temos: 1 3 Mx (D) = (b − a3 ) 3 Passo 3 calcular o momento de massa My (D) dado pela integral Mx (D) = dupla: 56 sin(ϑ)
  • 53. AULA Cálculo III My (D) = 3 (x, y)xdxdy D My (D) = (x, y)xdxdy D Substituindo os limites em coordenadas polares e sabendo que x = r cos(ϑ) temos: π/2 b r cos(ϑ)rdrdϑ (x, y)ydxdy = Mx (D) = 0 D a Integrando em r temos: π/2 r3 b dϑ 3 a 0 Substituindo os limites de integração temos: π/2 b3 a3 Mx (D) = cos(ϑ) − dϑ 3 3 0 Integrando em ϑ temos: π/2 b3 a3 − (sin(ϑ)) Mx (D) = 3 3 0 Substituindo os limites de integração temos: b3 a3 − (sin(π/2) − sin(0)) Mx (D) = 3 3 Simplificando temos: 1 3 Mx (D) = (b − a3 ) 3 Passo 4 Determinar o centro de massa de D pelas fórmulas: My (D) Mx (D) x= ¯ ey= ¯ . m(D) m(D) Usando os resultados anteriores temos: 1 3 (b − a3 ) 3 x=y= ¯ ¯ 1 π(b2 − a2 ) 4 Levando em conta que b3 − a3 = (b − a)(b2 + ba + a2 ) e b2 − a2 = Mx (D) = cos(ϑ) (b − a)(b + a) temos: 1 (b − a)(b2 + ba + a2 ) 3 x=y= ¯ ¯ 1 π(b − a)(b + a) 4 Simplificando temos: 4 b2 + ba + a2 x=y= ¯ ¯ . 3π b+a 57
  • 54. Algumas Aplicações da Integral Dupla 3.4 Conclusão Na aula de hoje, vimos que dentre as inúmeras aplicações da integral dupla, dentro da área da física destacamos, entre outras, algumas das mais importantes que são: a determinação da massa de uma região plana limitada por curvas, dada sua distribuição de densidade, o cálculo do momento de massa de uma região plana limitada por curvas, dada sua distribuição de densidade, o momento de inércia de uma região plana limitada por curvas, dada sua distribuição de densidade e o cálculo do centro de massa de uma região plana limitada por curvas, dada sua distribuição de densidade. RESUMO Massa, Momento de Massa e Momento de Inércia Dada uma região D ∈ R2 plana limitada com distribuição de densidade superficial (x, y) podemos calcular a massa de D, o momento de massa em relação ao eixo x, o momento de massa relativo ao eixo y, o momento de inércia em relação ao eixo x, o momento de inércia relativo ao eixo y e momento de inércia relativo a origem, denotados respectivamente m(D), Mx (D), My (D), Ix (D), Iy (D) e I0 (D), pelas integrais duplas: m(D) = (x, y)dxdy D Mx (D) = (x, y)ydxdy D My (D) = (x, y)xdxdy D (x, y)y 2 dxdy Ix (D) = D 58
  • 55. Cálculo III (x, y)x2 dxdy e Iy (D) = AULA 3 D (x, y)(x2 + y 2 )dxdy I0 (D) = D Centro de Massa Podemos também calcular o centro de massa, denotado (¯, y ) usando x ¯ as seguintes fórmulas: My (D) x= ¯ = m(d) x (x, y)dxdy D (x, y)dxdy D y= ¯ Mx (D) = m(d) y (x, y)dxdy D (x, y)dxdy D PRÓXIMA AULA Em nossa próxima aula veremos as integrais triplas. Primeiramente definindo-as para funções de domínios retangulares através do limite de somas de riemann estendendo a definição para funções definidas em domínios não retangulares porém limitados. ATIVIDADES Deixamos como atividades dois problemas de determinação do centro de massa. ATIV. 3.1. Determine o centro de massa da região D dada pela interseção das retas y = 0, x = 1 e y = ax2 (Fig 3.3) região em cinza. Comentário: Volte ao texto e reveja com calma e atenção as 59
  • 56. Algumas Aplicações da Integral Dupla Figura 3.3: Atividade 1 Figura 3.4: Atividade 2 demonstrações acima, elas lhe servirão de guia. Use para este caso coordenadas cartesianas. ATIV. 3.2. Determine o centro de massa da região D dada pelo semi-círculo superior x2 + y 2 = a2 (Fig 3.4) região em cinza. Comentário: Volte ao texto e reveja com calma e atenção as demonstrações acima, elas lhe servirão de guia. Use para este caso coordenadas polares. LEITURA COMPLEMENTAR ÁVILA, Geraldo, Cálculo 3: Funções de Várias Variáveis, Livros Técnicos e Científicos Editora, São Paulo, 3a edição, 1982. LEITHOLD, Louis, O Cálculo com Geometria Analítica. Volume 2, Editora Harbra, 1994. STEWART, James,Cálculo. Volume 3, 5a edição, Editora CENGAGE Learning, 2009. SWOKOWSKI, Earl E., Cálculo com Geometria Analítica, Volume 2, 2a edição, Makron Books do Brásil SP, 1994. 60
  • 57. Cálculo III AULA THOMAS, George B., Cálculo, Volume 2, 10a, Addilson Wesley, 3 2003. KAPLAN, Wilfred, Cálculo Avançado Vol.1 e vol.2 Editora Edgard Blücher 1991.// SPIEGEL, Murray R. Cálculo Avançado, Editora McGraw-Hill do Brasil, 1971. BOUCHARA, Jacques, Cálculo Integral Avançado, EDUSP, 2006. 61
  • 58.
  • 59. AULA Integrais triplas META: Apresentar integrais triplas de funções de valores reais e domínio em R3 . OBJETIVOS: Ao fim da aula os alunos deverão ser capazes de: Definir integral tripla e calcular algumas integrais triplas de funções de valores reais e domínio em R3 . PRÉ-REQUISITOS Os conhecimentos de integrais de funções de valores reais com domínio em R, da disciplina Cálculo I. 4
  • 60. Integrais triplas 4.1 Introdução Caros alunos a quarta aula do nosso curso de Cálculo III com o tema “Integrais Triplas”. Bem como a integral dupla, vista na nossa primeira aula, a integração tripla, em essência, é uma extensão natural da integral simples vista em Cálculo I e definida como limite de somas de Riemann. Na prática, a integração tripla é dada HISTÓRIA A primeira técnica sistemática documentada para o cálculo de integrais triplas no cálculo de volume foi o método da exaustão de Eudoxus cerca de 370AC. O maior avanço no cálculo de integrais triplas veio do Iraque, no século 11, na figura de Ibn AL-Haythan (conhecido na Europa por Alhazen ). Enquanto resolvia o que ficou conhecido como “Problema de Alhazen” (um problema de ótica) ele calculou o volume de um parabolsóide usando um método de indução. Wikipédia. por três integrações simples, cada uma efetuada sobre uma variável e considerando as demais como constantes. É o que denominamos de integrais interadas. As características e detalhes próprios das integrais triplas serão vistas ao longo do nosso curso, nas próximas três aulas. 4.2 Integração Tripla: Domínios Paralelepípedais Começamos por considerar uma função φ definida em um domínio paralelepipedal R = {(x, y, z) ∈ R3 |a ≤ x ≤ b ∧ c ≤ y ≤ d ∧ e ≤ z ≤ f }. Formalmente φ : [a, b] × [c, d] × [e, f ] → R. Usando a imaginação, pensemos em R retalhada por uma rede de planos paralelos aos planos coordenados e que dividem R em pequenos paralelepípedos. Oficialmente, consideraremos três partições P [a, b] = {x0 = a, x1 , . . . , xi , xi+1 , . . . , xl = b}, P [c, d] = {y0 = c, y1 , . . . , yj , yj+1 , . . . , ym = d} e P [e, f ] = {z0 = e, z1 , . . . , zk , zk+1 , . . . , zn = f } onde como visto em Cálculo I temos: x0 < x1 < · · · < xi < xi+1 < · · · < xl , y0 < y1 < · · · < yj < yj+1 < · · · < ym e z0 < z1 < · · · < zk < zk+1 < · · · < zn . Desta forma cada um dos pequenos subintervalos Ii = [xi−1 , xi ], Jj = 64
  • 61. Cálculo III AULA [yj−1 , yj ] e Kk = [zk−1 , zk ] têm comprimentos ∆xi = xi − xi−1 , 4 ∆yj = yj − yj−1 e ∆zk = zk − zk−1 , respectivamente. Definimos, agora, a uma partição para o paralelepípedo R por P = P [R] = P [a, b]×P [c, d]×P [e, f ], o produto cartesiano das partições P [a, b], P [c, d] e P [e, f ]. Os planos retalham a região R em uma série de pequenos paralelepípedos Vijk = [xi−1 , xi ] × [yj−1 , yj ] × [zk−1 , zk ], 1 ≤ i ≤ l, 1 ≤ j ≤ m, 1 ≤ k ≤ n. O volume de cada pequeno paralelepípedo é dado por ∆Vijk = ∆xi ∆yj ∆zk . Como tanto ∆xi quanto ∆yj quanto ∆zk são diferentes de zero, o volume de cada pequeno paralelepípedo é também diferente de zero. Podemos então definir a norma da partição por: |P | = max (∆Vijk ), que corresponde 1≤i≤l 1≤j≤m 1≤k≤n ao maior volume entre todos os pequenos paralelepípedos. Pausa para respirar que já vamos definir a integral tripla sobre domínios paralelepípedais. Para isto tomemos um ponto (ξi , ζj , ηk ) ∈ [xi−1 , xi ] × [yj−1 , yj ] × [zk−1 , zk ] em cada pequeno paralelepípedo e definimos a seguinte soma de Riemann: l m n Slmn = φ(ξi , ζj , ηk )∆Vijk i=1 j=1 k=1 A integral tripla da função φ(x, y, z) sobre o paralelepípedo R, φ(x, y, z)dxdydz será então definida como o se- denotada R guinte limite: def φ(x, y, z)dxdydz = lim Slmn R |P |→0 65
  • 62. Integrais triplas 4.3 Integração Tripla: Domínios Não Paralelepípedais Limitados Para definir a integral tripla de uma função φ : D ⊂ R3 → R onde D é limitado não paralelepipedal, começamos por considerar uma função Φ definida em um domínio paralelepipedal R = {(x, y, z) ∈ R3 |a ≤ x ≤ b ∧ c ≤ y ≤ d ∧ e ≤ z ≤ f } tal   φ(x, y, z) , (x, y, z) ∈ D . Formalque D ⊂ R e Φ(x, y, z) =  0 , (x, y, z) ∈ D / mente Φ : [a, b] × [c, d] × [e, f ] → R é uma extensão da função φ(x, y, z). Usando a imaginação, pensemos em R coberta por uma rede de planos paralelos aos planos coordenados e que dividem R em pequenos paralelepípedos e procedemos como na integral tripla sobre domínios paralelepípedais, considerando a uma partição para o paralelepípedo R por P = P [R] = P [a, b] × P [c, d] × P [e, f ], o produto cartesiano das partições P [a, b], P [c, d] e P [e, f ] onde P [a, b] = {x0 = a, x1 , . . . , xi , xi+1 , . . . , xl = b}, P [c, d] = {y0 = c, y1 , . . . , yj , yj+1 , . . . , ym = d} e P [e, f ] = {z0 = e, z1 , . . . , zk , zk+1 , . . . , zn = f }. Do mesmo modo definimos a norma da partição por: |P | = max (∆Vijk ) onde ∆Vijk = ∆xi ∆yj ∆zk , 1≤i≤l 1≤j≤m 1≤k≤n ∆xi = xi − xi−1 , ∆yj = yj − yj−1 e ∆zk = zk − zk−1 . Tomamos um ponto (ξi , ζj , ηk ) ∈ [xi−1 , xi ] × [yj−1 , yj ] × [zk−1 , zk ] em cada pequeno paralelepípedo e definimos a seguinte soma de Riemann para a função estendida Φ(x, y, z): l m n Slmn = Φ(ξi , ζj , ηk )∆Vijk i=1 j=1 k=1 66
  • 63. Cálculo III AULA A integral tripla da função φ(x, y, z) sobre o domínio D ⊂ R3 , 4 φ(x, y, z)dxdydz será então definida como o se- denotada D guinte limite: def φ(x, y, z)dxdydz = lim Slmn . |P |→0 D Observem que, semelhante ao caso das integrais duplas, apenas os pequenos paralelepípedos cujo ponto escolhido pertence ao domínio D ⊂ R3 , contribuem para a soma de Riemann os demais têm contribuição nula visto que o ponto escolhido dentro destes estão fora de D ⊂ R2 e portanto Φ(ξi , ζj , ηk ) = 0. 4.4 Interpretação Geométrica Quando a função φ : D ⊂ R3 → R é constante e igual a um (φ(x, y, z) = 1, ∀(x, y, z) ∈ D) e a região domínio D é limitada, vemos que a soma de Riemann aproxima o volume da região D e quanto maior for o refinamento da partição de R3 ⊃ R ⊃ D melhor será a aproximação. Podemos então, interpretar a integral tripla dxdydz como o volume da região D ⊂ R3 . D 4.5 Integrais Iteradas Dada uma função φ : R → R onde R = [a, b] × [c, d] × [e, f ], do mesmo modo que na integral dupla, valem as integrais interadas: b d f φ(x, y, z)dxdydz = 1. R φ(x, y, z)dz dy dx a c b e f d φ(x, y, z)dxdydz = 2. R φ(x, y, z)dy dz dx a e c 67
  • 64. Integrais triplas d b f φ(x, y, z)dxdydz = 3. R φ(x, y, z)dz dx dy c a d e f b φ(x, y, z)dxdydz = 4. R φ(x, y, z)dx dz dy c e f a d b φ(x, y, z)dx dy dz φ(x, y, z)dxdydz = 5. e R c f a b d φ(x, y, z)dy dx dz φ(x, y, z)dxdydz = 6. R e a c Em outras palavras, quando o domínio da integral tripla é paralelepipedal a ordem de integração não importa. 4.6 Propriedades das Integrais Triplas Como nosso curso é de Cálculo, apenas listaremos, sem demonstração, alguma das propriedades das integrais triplas. Caso desejem conhecer a demonstração de algumas destas propriedades, recomendo livros de Cálculo Avançado como os citados na bibliografia abaixo. Propriedade 4.1. Sejam f : D ⊂ R3 → R uma função de valores reais integrável em D e c ∈ R, então vale: cf (x, y, z)dxdydz = c D f (x, y, z)dxdydz D Propriedade 4.2. Sejam f, g : D ⊂ R3 → R duas funções de valores reais integráveis em D, então vale: (f + g)(x, y, z)dxdydz = D f (x, y, z)dxdydz D + g(x, y, z)dxdydz D 68
  • 65. Cálculo III AULA Propriedade 4.3. Sejam f : D ⊂ R3 → R uma função de valores 4 reais integrável em D tal que f (x, y, z) ≥ 0, ∀(x, y, z) ∈ D, então vale: f (x, y, z)dxdydz ≥ 0 D Propriedade 4.4. Sejam f, g : D ⊂ R3 → R duas funções de valores reais integráveis em D tais que f (x, y, z) ≥ g(x, y, z), ∀(x, y, z) ∈ D, então vale: f (x, y, z)dxdydz ≥ g(x, y, z)dxdydz D D Propriedade 4.5. Seja f : D ⊂ R3 → R uma função de valores reais integrável em D onde D = A ∪ B e A ∩ B é a união de um número finito de superfícies em R3 , então vale: f (x, y, z)dxdydz = D f (x, y, z)dxdydz A + f (x, y, z)dxdydz B OBS 4.1. As duas primeiras propriedades diz respeito à “linearidade” do operador integral tripla. As terceira e quarta propriedades são denominadas “dominação” enquanto que a quinta propriedade é denominada “aditividade”. 4.7 Exemplos Nada mais natural que ilustrar um novo conceito com exemplos e, vamos aqui fazer exatamente isto. Ilustrar o conceito de integral tripla com dois exemplos. Antes porém, vale observar 69
  • 66. Integrais triplas que a na prática uma integral tripla equivale a três integrais simples e neste caso uma pergunta fica no ar. Qual das duas variáveis x, y ou z integraremos primeiro? Muito bem, a resposta é dada pela propria expressão da integral tripla. Isto é, na integral f (x, y, z)dxdydz primeiramente integramos na variável x, R depois na variável y e por último na variável z. Já na integral f (x, y, z)dzdydx primeiramente integramos na variável z, R depois na variável y e por último na variável x. Exemplo 4.1. Considere a função f : [0, 1] × [0, 1] × [0, 1] → R dada por f (x, y) = x2 + y 2 + z 2 e determine a integral tripla f (x, y, z)dxdydz sobre a região R = {(x, y, z) ∈ R3 |0 ≤ I= R x ≤ 1 ∧ 0 ≤ y ≤ 1 ∧ 0 ≤ z ≤ 1}. SOLUÇÃO: Passo 1 colocaremos os limites de integração que representam a região R dada, segundo a ordem de integração: 1 1 1 (x2 + y 2 + z 2 )dxdydz I= 0 0 0 Passo 2 integraremos na variável x considerando as variáveis y e z como constantes: 1 1 x3 I= + y 2 x + z 2 x dydz 3 0 0 Substituindo os limites de integração temos: 1 1 13 03 I= − + y 2 (1 − 0) + z 2 (1 − 0) dydz 3 3 0 0 Efetuando os cálculos temos: 1 1 1 I= + y 2 + z 2 dy 3 0 0 Passo 3 integraremos na variável y considerando a variável x como constante: 70
  • 67. Cálculo III 1 1 1 y3 y+ + z2y dz 3 3 0 0 Substituindo os limites de integração temos: 1 1 13 03 I= (1 − 0) + − + z 2 (1 − 0) dz 3 3 3 0 Efetuando os cálculos temos: 1 1 1 + + z 2 dz I= 3 3 0 Passo 4 último passo, integraremos na variável z: 1 1 z3 1 I= z+ z+ 3 3 3 0 Substituindo os limites de integração temos: 1 1 13 03 I= (1 − 0) + (1 − 0) + − 3 3 3 3 Efetuando os cálculos temos: 1 1 1 I= + + =1 3 3 3 I= AULA 4 Figura 4.1: Determinação prática dos limites para D OBS 4.2. Daremos aqui um método prático para determinar os limites de integração em uma integral tripla sobre domínio não retangular da forma: D. Passo 1 Fazer um desenho da região D. (Fig. 4.1) identificando as superfícies inferior a(x, y) e superior b(x, y) que limitam a região 71
  • 68. Integrais triplas D, bem como a sombra projetada no plano xy por D, denotada D∗ e identificar as curvas limites da região D∗ a(x) curva inferior e b(x) curva superior, como na AULA01. Passo 2 Atravessar toda a região D∗ e o eixo x com um segmento de reta paralelo e orientado na direção positiva ao eixo y (segmento r na Fig. 4.1) Passo 3 Deslocar o segmento de reta r paralelo ao eixo y na direção negativa do eixo x até tocar o ponto mais à esquerda de D∗ marcando o limite inferior de x (ponto a na Fig. 4.1). Passo 4 Deslocar o segmento de reta r paralelo ao eixo y na direção positiva do eixo x até tocar o ponto mais à direita de D∗ marcando o limite superior de x (ponto b na Fig. 4.1). Passo 5 Tomando um ponto qualquer x ∈ (a, b) passamos o segmento de reta r através da região D∗ paralelo ao eixo y na direção positiva do eixo x. O limite inferior para a variável y será a função a(x), ponto da curva onde o segmento entra na região D∗ e o limite superior para a variável y será b(x), ponto da curva onde o segmento de reta sai da região D∗ . Passo 6 Tomando um ponto qualquer (x, y) ∈ D∗ passamos o segmento de reta s através da região D, paralelo ao eixo z orientado na direção positiva de z. O limite inferior para a variável z será a função a(x, y), ponto da superfície onde o segmento entra na região D e o limite superior para a variável z será b(x, y), ponto da superfície onde o segmento de reta sai da região D. Nossa integral será efetuada assim: b b(x) b(x,y) f (x, y)dxdy = D 72 f (x, y, z)dzdydx a a(x) a(x,y)
  • 69. AULA Cálculo III 4 Vamos diretamente para um segundo exemplo de integral dupla sobre domínios não retangulares. A saber: Exemplo 4.2. Considere a função f : D ⊂ R3 → R dada por f (x, y, z)dxdydz f (x, y) = xyz e determine a integral dupla D sobre a região D = {(x, y, z) ∈ R3 |0 ≤ x ≤ 1 ∧ 0 ≤ y ≤ x2 ∧ 0 ≤ z ≤ 1}, (Fig. 4.2). Figura 4.2: Domínio D para o exemplo 2 SOLUÇÃO: Passo 1 faremos o desenho das superfícies que determinam os limites para a região D. A saber x = 0, x = 1, y = x2 , x = 0 e z = 1 (Fig. 4.2). Usando o processo prático exposto acima determinamos os limites de integração. A saber: a = 0, b = 1, a(x) = 0, b(x) = x2 , a(x, y) = 0 e b(x, y) = 1. x2 1 1 I= xyzdzdydx 0 0 0 Passo 2 integraremos na variável z considerando a variável y 73
  • 70. Integrais triplas e x como uma constante: 1 x2 z2 1 dydx I= xy 2 0 0 0 Substituindo os limites de integração temos: 1 x2 12 02 xy − xy ) dydx I= 2 2 0 0 Efetuando os cálculos temos: 2 1 1 x xydydx I= 2 0 0 Passo 3 integraremos na variável y considerando a variável x constante temos: 1 y 2 x2 I= x dx 2 0 0 Substituindo os limites de integração temos: 1 1 (x2 )2 02 I= x −x dx 2 0 2 2 Efetuando os cálculos temos: 1 1 5 x dx I= 4 0 Integrando , finalmente , na variável x temos: 1 x6 1 I= 4 6 0 Substituindo os limites de integração temos: 1 16 06 − I= 4 6 6 1 Efetuando os cálculos temos: I = 24 4.8 Conclusão Na aula de hoje, vimos que a integral tripla é uma extensão natural do conceito de integral simples visto em Cálculo I e também uma extensão natural do conceito de integral dupla, vista em nossa primeira aula do curso de Cálculo III. E se por um lado a integral simples pode ser interpretada como a área sob a curva descrita por 74
  • 71. Cálculo III AULA função positiva f (x) em um domínio [a, b] e a integral dupla pode 4 ser vista como o volume de um prisma reto limitado superiormente pela a superfície descrita por uma função positiva f (x, y) e limitado inferiormente pelo domínio [a, b] × [c, d], a integral tripla só tem interpretação geométrica no caso simples em que f (x, y, z) = 1. Neste caso a integral tripla representa o volume da região limitada D ⊂ R3 . RESUMO Integração Tripla: Domínios Paralelepipedais Considerando uma função φ definida em um domínio paralelepipedal R = {(x, y, z) ∈ R3 |a ≤ x ≤ b ∧ c ≤ y ≤ d ∧ e ≤ z ≤ f }. Podemos dividir R em pequenos paralelepípedos considerando os planos paralelos ao planos cartesianos gerados pela partição P = P [R] = P [a, b]×P [c, d]×P [e, f ], o produto cartesiano das partições P [a, b], P [c, d] e P [e, f ] onde P [a, b] = {x0 = a, x1 , . . . , xi , xi+1 , . . . , xl = b}, P [c, d] = {y0 = c, y1 , . . . , yj , yj+1 , . . . , ym = d} e P [e, f ] = {z0 = e, z1 , . . . , zk , zk+1 , . . . , zn = f }. Os planos retalham a região R em uma série de pequenos paralelepípedos Vijk = [xi−1 , xi ] × [yj−1 , yj ] × [zk−1 , zk ], 1 ≤ i ≤ l, 1 ≤ j ≤ m, 1 ≤ k ≤ n. O volume de cada pequeno paralelepípedo é dado por ∆Vijk = ∆xi ∆yj ∆zk . A norma da partição fica estabelecida como: |P | = max (∆Vijk ). 1≤i≤l 1≤j≤m 1≤k≤n Toma-se um ponto (ξi , ζj , ηk ) ∈ [xi−1 , xi ] × [yj−1 , yj ] × [zk−1 , zk ] em cada pequeno paralelepípedo e definimos a seguinte soma de Riemann: 75
  • 72. Integrais triplas l m n Slmn = φ(ξi , ζj , ηk )∆Vijk i=1 j=1 k=1 A integral tripla da função φ(x, y, z) sobre o paralelepípedo R, φ(x, y, z)dxdydz será então definida como o se- denotada R guinte limite: def φ(x, y, z)dxdydz = lim Slmn |P |→0 R Integração Tripla: Domínios Não Paralelepípedais Limitados Para definir a integral tripla de uma função φ : D ⊂ R3 → R onde D é não paralelepipedal limitado, começamos por considerar uma função Φ definida em um domínio paralelepipedal R = {(x, y, z) ∈ R3 |a ≤ x b ∧ c ≤ y ≤ d ∧ e ≤ z ≤ f } tal que ≤  φ(x, y, z) , (x, y, z) ∈ D D ⊂ R e Φ(x, y, z) = . Formalmente  0 , (x, y, z) ∈ D / Φ : [a, b] × [c, d] × [e, f ] → R é uma extensão da função φ(x, y, z). A partir daqui todo o procedimento é semelhante ao da definição da integral tripla em domínios paralelepipedais. Podemos definir a integral tripla de uma função φ(x, y, z) em um domínio não retangular D por: def φ(x, y, z)dxdydz = lim Slmn D onde Slmn = l i=1 m j=1 |P |→0 n k=1 Φ(ξi , ζj , ηk )∆Vijk é a soma de Rie- mann para Φ(x, y, z. Integrais Iteradas As integrais iteradas dizem que em um domínio retangular R = [a, b] × [c, d] × [e, f ] a ordem de execução das integrais simples não 76
  • 73. Cálculo III AULA alteram o valor da integral tripla, que pode ser representada por: 4 b d f φ(x, y, z)dz dy dx φ(x, y, z)dxdydz = 1. a R c b e f d φ(x, y, z)dy dz dx φ(x, y, z)dxdydz = 2. a R e d c b f φ(x, y, z)dxdydz = 3. R φ(x, y, z)dz dx dy c a d e f b φ(x, y, z)dxdydz = 4. R φ(x, y, z)dx dz dy c e f a d b φ(x, y, z)dxdydz = 5. R φ(x, y, z)dx dy dz e c f a b d φ(x, y, z)dxdydz = 6. R φ(x, y, z)dy dx dz e a c Propriedades das Integrais triplas As integrais triplas são, de certo modo, semelhantes às propriedades das integrais simples que vimos em Cálculo I sendo quase que uma extensão natural destas. As integrais triplas têm, entre outras, as seguintes propriedades: Propriedade 1 Sejam f : D ⊂ R3 → R uma função de valores reais integrável em D e c ∈ R, então vale: cf (x, y, z)dxdydz = c D f (x, y, z)dxdydz D Propriedade 2 Sejam f, g : D ⊂ R3 → R duas funções de valores reais integráveis em D, então vale: (f + g)(x, y, z)dxdydz = D f (x, y, z)dxdydz D + g(x, y, z)dxdydz D 77
  • 74. Integrais triplas Propriedade 3 Sejam f : D ⊂ R3 → R uma função de valores reais integrável em D tal que f (x, y, z) ≥ 0, ∀(x, y, z) ∈ D, então vale: f (x, y, z)dxdydz ≥ 0 D Propriedade 4 Sejam f, g : D ⊂ R3 → R duas funções de valores reais integráveis em D tais que f (x, y, z) ≥ g(x, y, z), ∀(x, y, z) ∈ D, então vale: f (x, y, z)dxdydz ≥ D g(x, y, z)dxdydz D Propriedade 5 Seja f : D ⊂ R3 → R uma função de valores reais integrável em D onde D = A ∪ B e A ∩ B é a união de um número finito de superfícies em R3 , então vale: f (x, y, z)dxdydz = D f (x, y, z)dxdydz A + f (x, y, z)dxdydz B Determinação dos Limites de Integração para Integrais Triplas Daremos aqui um método prático para determinar os limites de integração em uma integral tripla sobre domínio não retangular da forma: D. Passo 1 Fazer um desenho da região D. (Fig. 4.1) identificando as superfícies inferior a(x, y) e superior b(x, y) que limitam a região D, bem como a sombra projetada no plano xy por D, denotada 78
  • 75. Cálculo III AULA D∗ e identificar as curvas limites da região D∗ a(x) curva inferior 4 e b(x) curva superior, como na AULA01. Passo 2 Atravessar toda a região D∗ e o eixo x com um segmento de reta paralelo e orientado na direção positiva ao eixo y (segmento r na Fig. 4.1) Passo 3 Deslocar o segmento de reta r paralelo ao eixo y na direção negativa do eixo x até tocar o ponto mais à esquerda de D∗ marcando o limite inferior de x (ponto a na Fig. 4.1). Passo 4 Deslocar o segmento de reta r paralelo ao eixo y na direção positiva do eixo x até tocar o ponto mais à direita de D∗ marcando o limite superior de x (ponto b na Fig. 4.1). Passo 5 Tomando um ponto qualquer x ∈ (a, b) passamos o segmento de reta r através da região D∗ paralelo ao eixo y na direção positiva do eixo x. O limite inferior para a variável y será a função a(x), ponto da curva onde o segmento entra na região D∗ e o limite superior para a variável y será b(x), ponto da curva onde o segmento de reta sai da região D∗ . Passo 6 Tomando um ponto qualquer (x, y) ∈ D∗ passamos o segmento de reta s através da região D, paralelo ao eixo z orientado na direção positiva de z. O limite inferior para a variável z será a função a(x, y), ponto da superfície onde o segmento entra na região D e o limite superior para a variável z será b(x, y), ponto da superfície onde o segmento de reta sai da região D. 79
  • 76. Integrais triplas Nossa integral será efetuada assim: b b(x) b(x,y) f (x, y)dxdy = D f (x, y, z)dzdydx a a(x) a(x,y) PRÓXIMA AULA Em nossa próxima aula veremos mudança de variáveis na integração tripla. O objetivo da mudança de variáveis em uma integral tripla será a de facilitar esta integração de uma de duas formas. A primeira será tornando o integrando mais simples. A segunda transformando o domínio D do integrando em um domínio de forma geométrica mais simples. ATIVIDADES Deixamos como atividades o cálculo de algumas integrais tríplas. ATIV. 4.1. Seja f : [−1, +1] × [−1, +1] × [−1, +1] → R dada por f (x, y, z) = x2 + y 2 + z 2 . Determine a integral tripla: f (x, y, z)dxdydz. R Comentário: Volte ao texto e reveja com calma e atenção o cálculo de integrais duplas dos exemplos acima, elas lhe servirão de guia. ATIV. 4.2. Seja f : D ⊂ R3 → R dada por f (x, y, z) = 1, onde D = {(x, y, z) ∈ R3 |x ≥ 0 ∧ 0 ≤ y ≤ 1 − x2 ∧ 0 ≤ z ≤ 1 − x2 }. 80
  • 77. Cálculo III AULA 4 • Esboce a região de integração • Determine os limites da integral tripla: f (x, y, z)dxdydz D • Calcule a integral tripla f (x, y, z)dxdydz. D Comentário: Volte ao texto e reveja com calma e atenção o cálculo de integrais duplas dos exemplos acima, elas lhe servirão de guia. LEITURA COMPLEMENTAR ÁVILA, Geraldo, Cálculo 3: Funções de Várias Variáveis, Livros Técnicos e Científicos Editora, São Paulo, 3a edição, 1982. LEITHOLD, Louis, O Cálculo com Geometria Analítica. Volume 2, Editora Harbra, 1994. STEWART, James,Cálculo. Volume 3, 5a edição, Editora CENGAGE Learning, 2009. SWOKOWSKI, Earl E., Cálculo com Geometria Analítica, Volume 2, 2a edição, Makron Books do Brásil SP, 1994. THOMAS, George B., Cálculo, Volume 2, 10a, Addilson Wesley, 2003. KAPLAN, Wilfred, Cálculo Avançado Vol.1 e vol.2 Editora Edgard Blücher 1991.// SPIEGEL, Murray R. Cálculo Avançado, Editora McGraw-Hill do Brasil, 1971. BOUCHARA, Jacques, Cálculo Integral Avançado, EDUSP, 2006. 81
  • 78.
  • 79. AULA Mudança de Variáveis em Integrais tríplas META: Introduzir mudança de variáveis em integrais triplas de funções de valores reais e domínio em R3 . OBJETIVOS: Ao fim da aula os alunos deverão ser capazes de: Calcular integrais triplas de funções de valores reais e domínio em R3 utilizando mudança de variáveis. PRÉ-REQUISITOS Os conhecimentos de integrais de funções de valores reais com domínio em R, da disciplina Cálculo I, superfícies em R3 , de coordenadas polares da disciplina Cálculo II e integrais triplas aula 04. 5
  • 80. Mudança de Variáveis em Integrais tríplas 5.1 Introdução Caros alunos o problema da mudança de variáveis em integrais triplas é inteiramente análogo ao problema de mudança de variáveis em integrais duplas. Analogias a parte, o fato de do espaço R3 ter uma dimensão a mais que o R2 , traz um esforço algébrico adicional ao tratamento geral da mudança de variáveis em integrais HISTÓRIA O teorema de mudança de variáveis em integrais tríplas foi primeiro proposto por Lagrange em 1773 e usado por Legendre, Laplace e Gauss, e primeiramente generalizado para n variáveis por Mikhail Ostrogradski em 1836, resistiu a uma demonstração mais rigorosa por longo tempo (cerca de 125 anos). E foi satisfatóriamente demonstrado por Elie Cartan em uma série de artigos nos anos 1890. triplas. Veremos dois casos particulares de mudança de variáveis em integrais tripla que correspondem aos: sistemas de coordenadas cilíndricos e sistema de coordenadas esféricos. 5.2 Mudança de Variáveis em Integrais Triplas Vamos considerar a integração de uma função f : D ⊂ R3 → R onde (x, y, z) ∈ D e conseqüentemente, ∀(x, y, z) ∈ D temos f (x, y, z) ∈ R. Consideraremos também, uma transformação T : D ⊂ R3 → D ⊂ R3 , biunívoca de modo que D = T −1 (D ), ∀(u, v, w) ∈ D , (x, y, z) = T −1 (u, v, w) ∈ D. Trocando em miúdos: x = x(u, v, w), y = y (u, v, w) e z = z (u, v, w). E suponhamos ˆ ˆ ˆ as funções contínuas e deriváveis e seu ∂(x, y, z) x, y, z ou : finido por: J u, v, w ∂(u, v, w)  ∂x ˆ  ∂u  ∂x ∂(x, y, z) x, y, z ˆ J = = det   ∂v u, v, w ∂(u, v, w)  ∂x ˆ ∂w jacobiano, denotado J, de- ∂y ˆ ∂u ∂y ˆ ∂v ∂y ˆ ∂w ∂z ˆ ∂u ∂z ˆ ∂v ∂z ˆ ∂w    .   Suponhamos uma partição de D feita partindo de planos paralelos aos planos coordenados vw (u constante), uw (v constante) e uv (w constante). Denotando ui+1 = ui + ∆ui , vj+1 = vj + ∆vj e 84
  • 81. Cálculo III AULA wk+1 = wk + ∆wk , destacamos o pequeno paralelepípedo indexado 5 por ijk, (Fig 5.1). Suponhamos que Figura 5.1: Elemento de vo- Figura 5.2: Elemento de vo- lume em D lume em D este pequeno paralelepípedo de volume ∆Vijk = ∆ui ∆vj ∆wk seja mapeado por T −1 em um subdomínio em D de volume ∆Vijk (Fig 5.2). Seja: P = P (u, v, w) = (ˆ(u, v, w), y (u, v, w), z (u, v, w)). x ˆ ˆ Os segmentos de reta (u, vj , wk ), (ui , v, wk ) e (ui , vj , w) começando no ponto (ui , vj , wk ) são mapeados por T −1 em P (u, vj , wk ) P (ui , v, wk ) P (ui , vj , w) ver (Fig 5.2). ∂P No subdomínio Vijk ⊂ D traçamos os vetores tangentes ∆ui , ∂u ∂P ∂P ∆vj e ∆wk , ver (Fig 5.3). Em seguida traçamos segmentos ∂v ∂w de reta paralelos aos vetores tangentes completando um paralelepípedo em D, ver (Fig 5.4), cujo volume admitiremos aproximadamente igual ao ∆Vijk (esta é a argumentação heurística). Este volume é dado por: ∆Vijk ≈ ∂P ∂P ∂P ∆ui × ∆vj • ∆wk . ∂u ∂v ∂w 85
  • 82. Mudança de Variáveis em Integrais tríplas Levando em conta que: ∂P ∂x ˆ ∂y ˆ ∂z ˆ = i+ j+ k ∂u ∂u ∂u ∂u ∂P ∂x ˆ ∂y ˆ ∂z ˆ = i+ j+ k ∂v ∂v ∂v ∂v ∂P ∂x ˆ ∂y ˆ ∂z ˆ = i+ j+ k ∂w ∂w ∂w ∂w e calculando o produto vetorial mixto teremos: Figura 5.3: Elemento de vo- Figura 5.4: Elemento de vo- lume em D lume em D    ∂P ∂P ∂P ∆ui × ∆vj • ∆wk = det   ∂u ∂v ∂w  ∂x ˆ ∂u ∂x ˆ ∂v ∂x ˆ ∂w ∂y ˆ ∂u ∂y ˆ ∂v ∂y ˆ ∂w ∂z ˆ ∂u ∂z ˆ ∂v ∂z ˆ ∂w     ∆ui ∆vj ∆wk   Daí, levando em conta a expressão do jacobiano em R3 , dada acima, temos: ∆Vijk ≈ ∂(ˆ, y , z ) x ˆ ˆ ∆ui ∆vj ∆wk ∂(u, v, w) O que nos leva à seguinte expressão para a mudança de variáveis em integrais triplas: F (u, v, w) |J| dudvdw f (x, y, z)dxdydz = D 86 D
  • 83. Cálculo III onde: F (u, v, w) = f (ˆ(u, v, w), y (u, v, w), z (u, v, w)) e J é o jacox ˆ ˆ x, y , z ˆ ˆ ˆ . biano J = J u, v, w 5.3 AULA 5 Alguns Exemplos Nesta seção veremos dois exemplos de integrais triplas com mudança de variáveis. No primeiro aplicaremos a mudança de variáveis dada pelo sistema de coordenadas cilíndricas e no segundo o sistema de coordenadas esféricas Primeiramente veremos um exemplo em coordenadas cilíndricas. Antes porém, veremos como determinar os limites de integração em coordenadas cilíndricas. Figura 5.5: Coordenadas ci- Figura 5.6: Coordenadas ci- líndricas 1 líndricas 2 Passo 1 Esboçar o domínio D bem como sua projeção D∗ no plano xy (ver Fig. 5.5). Passo 2 Identificar as curvas que limitam a região D∗ . Atravessar a região D∗ com uma reta r começando na origem (ver Fig. 5.6). 87
  • 84. Mudança de Variáveis em Integrais tríplas À medida em que a reta r percorre a região D∗ o ângulo ϑ que ela forma com o eixo x positivo varia do mínimo α que será o limite inferior da variável ϑ ao máximo β que será o limite superior da variável ϑ. Daí, a variável ϑ ∈ [α, β]. Passo 3 Para cada valor fixo da variável ϑ ∈ [α, β] atravessar a região D∗ com a reta r (ver Fig. 5.7). O ponto onde a reta r entra na região D∗ é o limite inferior α(ϑ) para a variável r e o ponto onde a reta r sai da região D∗ é o limite inferior β(ϑ) para a variável r. Daí, r ∈ [α(ϑ), β(ϑ)]. Passo 4 Para cada valor fixo da variável ϑ ∈ [α, β] e da variável r ∈ [α(ϑ), β(ϑ)] tomar o ponto (r, ϑ) ∈ D∗ em coordenadas polares e levantar a reta s atravessando a região D (ver Fig. 5.8). O ponto onde a reta s entra na região D é o limite inferior α(r, ϑ) para a variável z e o ponto onde a reta s sai da região D é o limite superior β(r, ϑ) para a variável z. Daí, z ∈ [α(r, ϑ), β(r, ϑ)]. Figura 5.7: Coordenadas ci- Figura 5.8: Coordenadas ci- líndricas 3 líndricas 4 Podemos agora encarar o nosso primeiro exemplo onde colocaremos 88
  • 85. Cálculo III AULA em prática a determinação dos limites de integração em coordena- 5 das cilíndricas. Exemplo 5.1. Considere o sólido gerado pela intersecção das superfícies: z = y + a, (plano) x2 + y 2 − 2ay = 0, (cilindro) e z = 0, (plano) (Fig 5.9) e determine seu volume. Figura 5.9: exemplo 1 Superfícies do Figura 5.10: Interseção das superfícies do exemplo 1 SOLUÇÃO: Para uma melhor compreensão mostramos na (Fig 5.10) o sólido gerado pela interseção das superfícies dadas e na (Fig 5.11) as superfícies que compõem o sólido separadas no espaço. Usaremos para o caso o sistema de coordenadas cilíndricas, dada pela transformação: (x, y, z) → (r, ϑ, z) onde x = r cos(ϑ), y = r sin(ϑ) e z = z. O jacobiano da transformação é dado por: 89
  • 86. Mudança de Variáveis em Integrais tríplas Figura 5.11: Domínio D para o exemplo 2 ∂x ˆ  ∂r  ∂x x, y, z ˆ = det  J =J  ∂ϑ r, ϑ, z  ∂x ˆ ∂z Efetuando as derivadas parciais temos:   cos(ϑ) ∂y ˆ ∂r ∂y ˆ ∂ϑ ∂y ˆ ∂z sin(ϑ) ∂z ˆ ∂r ∂z ˆ ∂ϑ ∂z ˆ ∂z 0            J = det  −r sin(ϑ) r cos(ϑ) 0    0 0 1 Fazendo as contas do determinante temos: J =r Aproveitaremos o exemplo para aplicar os passos, vistos acima, para determinação dos limites de integração de uma integral tripla no sistema de coordenadas cilíndricas. Passo 1: Esboçar a interseção das superfícies (sólido D), bem como sua projeção sobre o plano xy (superfície D∗ ), (ver Fig 5.10). A projeção sobre o plano xy (superfície D∗ ), conhecide com a superfície inferior do sólido, sendo o disco dado por x2 +y 2 −2ay ≤ 0. 90
  • 87. Cálculo III AULA Passo 2: Os limites para r e ϑ são determinados em D∗ do mesmo 5 modo que para coordenadas polares em R2 . Neste caso 0 ≤ ϑ ≤ 2π e para r temos: que r vai de zero até a borda de D∗ que é dada por x2 + y 2 − 2ay = (r cos(ϑ))2 + (r sin(ϑ))2 − 2ar sin(ϑ) = 0. Daí, r2 − 2ar sin(ϑ) = r(r − 2a sin(ϑ)) = 0 Simplificando temos: 0 ≤ r ≤ 2a sin(ϑ). Passo 3: Para determinar os limites para z. Por cada par (r, ϑ) ∈ D∗ , traçamos uma reta paralela ao eixo z orientada no sentido positivo do eixo z atravessando o sólido. O limite inferior de z é o ponto onde a reta entra no sólido e o limite superior o ponto onde a reta sai do sólido. Neste caso: 0 ≤ z ≤ a + x ou como x = r cos(ϑ) temos: 0 ≤ z ≤ a + r cos(ϑ). Daí, o cálculo do volume de D será dado pela integral: 2π 2a sin(ϑ) a+r sin(ϑ) V ol(D) = rdzdrdϑ 0 0 0 Integrando na variável z temos: 2π 2a sin(ϑ) V ol(D) = a+r sin(ϑ) rz 0 0 drdϑ 0 Substituindo os limites de integração temos: 2π 2a sin(ϑ) r(a + r sin(ϑ) − 0)drdϑ V ol(D) = 0 0 Fazendo as contas temos: 2π 2a sin(ϑ) (ar + r2 sin(ϑ))drdϑ V ol(D) = 0 0 Integrando em na variável r temos: 2π V ol(D) = (a 0 r2 r3 + sin(ϑ)) 2 3 2a sin(ϑ) 0 dϑ 91
  • 88. Mudança de Variáveis em Integrais tríplas Substituindo o limite superior pois, o limite inferior por ser r = 0 não contribui, temos: 2π V ol(D) = (a 0 (2a sin(ϑ))2 (2a sin(ϑ))3 + sin(ϑ))dϑ 2 3 Simplificando temos: 2π (2a3 sin2 (ϑ) + V ol(D) = 0 8a3 sin4 (ϑ) )dϑ 3 Reescrevendo temos: 2π sin2 (ϑ)dϑ + V ol(D) = 2a3 0 8a3 3 2π sin4 (ϑ)dϑ 0 Das tabelas de integrais temos: α sinn−1 (αu) cos(αu) an n−1 + sinn−2 (αu)du n u sin(2αu) − sin2 (αu)du = 2 4α sinn (αu)du = − Dai, temos: ϑ sin(2ϑ) − 2 4 sin3 (ϑ) cos(ϑ) 3 sin4 (ϑ)dϑ = − + 4 4 3 sin (ϑ) cos(ϑ) 3 = − + 4 4 sin2 (ϑ)dϑ = sin2 (ϑ)dϑ ϑ sin(2ϑ) − 2 4 Podemos agora calcular as integrais. Para a integral de sin2 (ϑ) temos: 2π ϑ sin(2ϑ) 2π − 2 4 0 2π sin(4π) = + − 2 4 0 sin(0) − + 2 4 = π sin2 (ϑ)dϑ = 0 92
  • 89. AULA Cálculo III 5 Para a integral de sin4 (ϑ) temos: 2π 2π sin3 (ϑ) cos(ϑ) 3 ϑ sin(2ϑ) + − 4 4 2 4 0 3 sin (2π) cos(2π) 3 2π sin(4π) = + − + − 4 4 2 4 3 sin (0) cos(0) 3 0 sin(0) − − + − 4 4 2 4 3π = 4 sin4 (ϑ)dϑ = 0 − Substituindo no cálculo de V ol(D) temos: V ol(D) = 2πa3 + 8a3 3π 3 4 = 4πa3 Em nosso segundo exemplo utilizaremos coordenadas esféricas, Antes porém, veremos como determinar os limites de integração em coordenadas esféricas. Figura 5.12: Coordenadas es- Figura 5.13: Coordenadas es- féricas 1 féricas 2 Passo 1 Esboçar o domínio D bem como sua projeção D∗ no plano xy (ver Fig. 5.12). 93
  • 90. Mudança de Variáveis em Integrais tríplas Passo 2 Identificar as curvas que limitam a região D∗ . Atravessar a região D∗ com uma reta r começando na origem (ver Fig. 5.13). Á medida em que a reta r percorre a região D∗ o ângulo ϑ que ela forma com o eixo x positivo varia do mínimo α que será o limite inferior da variável ϑ ao máximo β que será o limite superior da variável ϑ. Daí, a variável ϑ ∈ [α, β]. Passo 3 Para cada valor fixo da variável ϑ ∈ [α, β] atravessar Figura 5.14: Coordenadas es- Figura 5.15: Coordenadas es- féricas 3 féricas 4 a região D com o plano P que contem o eixo z e forma ângulo ϑ com o eixo x positivo (ver Fig. 5.14). Traçamos uma reta r que começa na origem e está contida no plano que corta D. À medida em que a reta r percorre a região D o ângulo ϕ que ela forma com o eixo z positivo varia do mínimo α(ϑ) que será o limite inferior da variável ϕ ao máximo β(ϑ) que será o limite superior da variável ϕ. Daí, a variável ϕ ∈ [α(ϑ), β(ϑ)]. Passo 4 Para cada valor fixo da variável ϑ ∈ [α, β] e da variável r ∈ [α(ϑ), β(ϑ)] plano P que contem o eixo z e forma ângulo ϑ com o eixo x positivo. No plano P traçar a reta s que forma ângulo ϕ 94
  • 91. Cálculo III AULA com o eixo z positivo atravessando a região D (ver Fig. 5.15). O 5 ponto onde a reta s entra na região D é o limite inferior α(ϑ, ϕ) para a variável r e o ponto onde a reta s sai da região D é o limite superior β(ϑ, ϕ) para a variável r. Daí, r ∈ [α(ϑ, ϕ), β(ϑ, ϕ)]. Podemos agora encarar o nosso segundo exemplo onde colocaremos em prática a determinação dos limites de integração em coordenadas esféricas. Exemplo 5.2. Considere o sólido gerado pela interseção das superfícies: z = x2 + y 2 (cone), z = a2 − x2 − y 2 (esfera)(Fig 5.16) e determine seu volume. Figura 5.16: exemplo 2 Superfícies de Figura 5.17: Interseção das superfícies de exemplo 2 SOLUÇÃO: Para uma melhor compreensão mostramos na (Fig 5.17) o sólido gerado pela interseção das superfícies dadas e na (Fig 5.18) as superfícies que compõem o sólido separadas no espaço. 95
  • 92. Mudança de Variáveis em Integrais tríplas Figura 5.18: Domínio D para o exemplo 2 Usaremos para o caso o sistema de coordenadas esféricas, dada pela transformação: (x, y, z) → (r, ϑ, ϕ) onde x = r cos(ϑ) cos(ϕ), y = r sin(ϑ) cos(ϕ) e z = r sin(ϕ). O jacobiano da transformação é dado por: J =J x, y, z r, ϑ, z  ∂x ˆ  ∂r  ∂x ˆ = det   ∂ϑ  ∂x ˆ ∂ϕ ∂y ˆ ∂r ∂y ˆ ∂ϑ ∂y ˆ ∂ϕ ∂z ˆ ∂r ∂z ˆ ∂ϑ ∂z ˆ ∂ϕ       Efetuando as derivadas parciais temos:  cos(ϑ) cos(ϕ) sin(ϑ) cos(ϕ) sin(ϕ)   J = det  −r sin(ϑ) cos(ϕ) r cos(ϑ) cos(ϕ) 0  −r cos(ϑ) sin(ϕ) −r sin(ϑ) sin(ϕ) r cos(ϕ)      Fazendo as contas do determinante temos: J = r2 sin(ϕ) Aproveitaremos o exemplo para aplicar os passos na determinação dos limites de integração de uma integral tripla no sistema de 96
  • 93. Cálculo III AULA 5 Figura 5.19: Domínio D para o exemplo 2 coordenadas esféricas expostos acima. Passo 1: Esboçar a interseção das superfícies (sólido D), bem como sua projeção sobre o plano xy (superfície D∗ ), ver (Fig 5.19). A projeção sobre o plano xy (superfície D∗ ), é dada por x2 + y 2 ≤ a2 . Passo 2: Os limites para a variável ϑ são determinados em D∗ como em um sistema de coordenadas polares. No caso como D∗ é √ a temos que: 0 ≤ ϑ ≤ 2π. um disco de raio 2 Passo 3: Os limites para a variável ϕ são determinados em D do seguinte modo: para cada valor fixo de ϑ, em D∗ , cortamos o domínio D por um plano que passa no eixo z e forma ângulo ϑ com o eixo x. Traçamos uma reta M que passa na origem, pertence ao plano ϑ e atravessa o domínio D. O ângulo ϕ é o ângulo formado por M e o eixo z positivo. Para o caso o menor valor é ϕ = 0, quando M conhecide com o eixo Z e o maior valor de ϕ em D é quando M conhecide com a geratriz do cone z = x2 + y 2 e π ϕ= . 4 Passo 4: Os limites para a variável r são determinados em D 97
  • 94. Mudança de Variáveis em Integrais tríplas do seguinte modo: para cada par fixo ϑ, ϕ) percorremos a reta M partindo da origem. O limite inferior de r é o ponto onde a reta entra em D e o limite superior o ponto onde M sai de D. Para o nosso caso: 0 ≤ r ≤ a (a reta sai na superfície da esfera z= a2 − x2 − y 2 . Podemos determinar o volume de D pela integral tripla: 2π π/4 a r2 sin(ϕ)drdϕdϑ V ol(D) = 0 0 0 Integrando primeiramente na variável r temos: 2π π/4 V ol(D) = 0 0 r3 3 a 0 sin(ϕ)dϕdϑ Substituindo os limites de integração temos: 2π π/4 a3 03 − 3 3 V ol(D) = 0 0 sin(ϕ)dϕdϑ Simplificando temos: V ol(D) = a3 3 2π π/4 sin(ϕ)dϕdϑ 0 0 Integrando na variável ϕ temos: V ol(D) = a3 3 2π − cos(ϕ) 0 π/4 0 dϑ Substituindo os limites de integração temos: V ol(D) = a3 3 2π (− cos(π/4) + cos(0)) dϑ 0 Simplificando temos: √ a3 2 − 2 V ol(D) = 3 2 2π dϑ 0 Integrando na variável ϑ temos: √ a3 2 − 2 V ol(D) = ϑ 3 2 98 2π 0
  • 95. AULA Cálculo III Substituindo os limites de integração temos: 5 √ a3 2 − 2 V ol(D) = (2π − 0) 3 2 Finalmente, simplificando temos: πa3 (2 − V ol(D) = 3 5.4 √ 2) Conclusão Na aula de hoje, vimos que algumas vezes é conveniente fazer uma mudança nas variáveis de integração em uma integral tripla, para facilitar o cálculo da mesma. Vimos em particularmente duas mudanças de variáveis são muito importantes e correspondem aos: sistema de coordenadas cilíndricas e sistema de coordenadas esféricas. RESUMO Para o nosso resumo da aula 05 necessitamos algumas considerações iniciais para tratar da mudança de variáveis em integrais triplas. A saber: Consideramos a transformação (x, y, z) = T (u, v, w) tal que o domínio um ponto do domínio D ⊂ R3 , (x, y, z) seja transformado no ponto (u, v, w) do domínio D ⊂ R3 , (D = T (D )) e mais especificamente x = x(u, v, w), y = y (u, v, w) e z = z (u, v, w). Deˆ ˆ ˆ x, y, z ou finindo o jacobiano da transformação, denotado J, J u, v, w ∂(x, y, z) , por: ∂(u, v, w) 99